Você está na página 1de 121

MABELLE J.

ARELLANO PROPERTY
L02 Saturday, 8:00 AM-12:00 PM

Luis Marcos P. Laurel vs Hon. Zeus C. Abrogar


GR No. 155076
January 13, 2009

FACTS

Laurel was charged with Theft under Art. 308 of the Revised Penal Code (RPC) for
allegedly taking, stealing, and using PLDT's international long distance calls by conducting
International Simple Resale (ISR) – “a method of outing and completing international long-
distance calls using lines, cables, antennae, and/or air wave frequency which connect
directly to the local/domestic exchange facilities of the country where the call is destined”.
PLDT alleged that this service was stolen from them using their own equipment and caused
damage to them amounting to P20,370,651.92. PLDT alleges that the international calls
and business of providing telecommunication or telephone service are personal properties
capable of appropriation and can be objects of theft.

ISSUE

Whether or not the international calls as well as the business of providing


telecommunication or telephone service are personal properties capable of appropriation
and can be the objects of theft.

RULING

Art.308, RPC: Theft is committed by any person who, with intent to gain but without
violence against, or intimidation of persons nor force upon things, shall take personal
property of another without the latter’s consent.

The term “personal property” as used in Art.308, RPC should be interpreted in the context
of the Civil Code's definition of real and personal property. Consequently, any personal
property, tangible or intangible, corporeal or incorporeal, capable of appropriation may be
the subject of theft so long as the same is not included in the enumeration of Real
Properties under the Civil Code.

The only requirement for personal property to be the object of theft under the penal code
is that it be capable of appropriation.

Art. 416 (3) of the Civil Code deems “Forces of Nature” which are brought under the control
of science, as Personal Property.

In the instant case, the act of conducting ISR operations by illegally connecting various
equipment or apparatus to PLDT’s telephone system, through which petitioner is able to
resell or re-route international long distance calls using PLDT’s facilities constitute
Subtraction.

Moreover, interest in business should be classified as personal property since it is capable


of appropriation, and not included in the enumeration of real properties. Therefore, the
business of providing telecommunication or telephone service are personal property which
can be the object of theft under Art. 308 of the RPC.

PLDT merely encodes, augments, enhances, decodes and transmits said calls using its
complex communications infrastructure and facilities. Since PLDT does not own the said
telephone calls, then it could not validly claim that such telephone calls were taken without
its consent.

What constitutes Theft is the use of the PLDT's communications facilities without PLDT's
consent. The theft lies in the unlawful taking of the telephone services and businesses. The
Amended Information should be amended to show that the property subject of the theft
were services and business of the offended party.

1|Page
MABELLE J. ARELLANO PROPERTY
L02 Saturday, 8:00 AM-12:00 PM

Antonio Bicerra, Domingo Bicerra, Bernardo Bicerra, Cayetano Bicerra, Linda


Bicerra, Pio Bicerra and Eufricina Bicerra, vs. Tomasa Teneza and Benjamin
Barbosa
G.R. No. L-16218, November 29, 1962

FACTS

Appellants were the owners of the house worth P200.00, built on and owned by them and
situated in the said municipality Lagangilang; that sometime in January 1957, appellees
forcibly demolished the house, claiming to be the owners thereof; that the materials of the
house, after it was dismantled, were placed in the custody of the barrio lieutenant of the
place; and that as a result of appellees’ refusal to restore the house or to deliver the
material to the appellants, the latter have suffered damages.

ISSUE

Whether or not the action involves title to real property.

RULING

A house is classified as immovable property by reason of its adherence to the soil which it
is built (Art. 415, par. 1, Civil Code). This classification holds true regardless of the fact that
the house may be situated on land belonging to a different owner. But once the house is
demolished, as in this case, it ceases to exist as such and hence its character as an
immovable likewise ceases.

2|Page
MABELLE J. ARELLANO PROPERTY
L02 Saturday, 8:00 AM-12:00 PM

Antonio Punsalan, Jr. vs. Remedios Vda. De Lacsamana and the Honorable Judge
Rodolfo A. Ortiz
G.R. No. L-55729, March 28, 1983

FACTS

The petitioner was the former registered owner of a parcel of land situated in Bamban,
Tarlac. In 1963, petitioner mortgaged said land to respondent PNB (Tarlac Branch) but for
failure to pay said amount, the property was foreclosed on December 16, 1970.
Respondent PNB was the highest bidder in said foreclosure proceedings. However, the
bank secured title thereto only on December 14, 1977. In the meantime, in 1974, while the
property was still in the alleged possession of petitioner and with the acquiescence of
respondent PNB, petitioner constructed a warehouse on said property.

On July 26, 1978, a Deed of Sale was executed between respondent PNB and respondent
Lacsamana over the property. This contract was amended particularly to include in the
sale, the building and improvement thereon. By virtue of said instruments, respondent
Lacsamana secured title over the property in her name as well as separate tax declarations
for the land and building. On November 22, 1979, petitioner commenced suit for
“Annulment of Deed of Sale with Damages” against herein respondents PNB and
Lacsamana before the Court of First Instance of Rizal, Quezon City, essentially impugning
the validity of the sale of the building as embodied in the Amended Deed of Sale.

Respondent PNB filed a motion to Dismiss on the ground that venue was improperly laid
considering that the building was real property under article 415 (1) of the New Civil Code
and therefore section 2(a) of Rule 4 should apply. The respondent Court granted PNB’s
Motion to Dismiss the plaintiff’s complaint for improper venue.

ISSUE

Whether or not the judgement rendered by the Court is proper.

RULING

The warehouse claimed to be owned by petitioner is an immovable or real property as


provided in article 415(l) of the Civil Code. Buildings are always immovable under the Code.
A building treated separately from the land which it stood is immovable property and the
mere fact that the parties to a contract seem to have dealt with it separate and apart from
the land on which it stood in no wise changed its character as immovable property.

While it is true that petitioner does not directly seek the recovery of title or possession of
the property in question, his action for annulment of sale and his claim for damages are
closely intertwined with the issue of ownership of the building which, under the law, is
considered immovable property, the recovery of which is petitioner’s primary objective. The
prevalent doctrine is that an action for the annulment or rescission of a sale of real property
does not operate to efface the fundamental and prime objective and nature of the case,
which is to recover said real property. It is a real action.

Respondent Court, therefore, did not err in dismissing the case on the ground of improper
venue (Section 2, Rule 4), which was timely raised (Section 1, Rule 16).

3|Page
MABELLE J. ARELLANO PROPERTY
L02 Saturday, 8:00 AM-12:00 PM

Ruby L. Tsai vs. Hon. Court of Appeals, Ever Textile Mills, Inc. and Mamerto R.
Villaluz
G.R. No. 120098, October 2, 2001

FACTS

Ever Textile Mills, Inc. (EVERTEX) obtained a loan from petitioner Philippine Bank of
Communications (PBCom). As security for the loan, EVERTEX executed in favor of
PBCom, a deed of Real and Chattel Mortgage over the lot where its factory stands, and
the chattels located therein as enumerated in a schedule attached to the mortgage
contract. PBCom granted a second loan to EVERTEX. The loan was secured by a Chattel
Mortgage over personal properties enumerated in a list attached thereto. The listed
properties were similar to those listed in the first mortgage deed. Due to business reverses,
EVERTEX filed insolvency proceedings docketed. The CFI issued an order on declaring
the corporation insolvent. All its assets were taken into the custody of the Insolvency Court,
including the collateral, real and personal, securing the two mortgages as abovementioned.

Upon EVERTEX’s failure to meet its obligation to PBCom, the latter commenced
extrajudicial foreclosure proceedings against EVERTEX. PBCom was the highest
bidder. Thus, PBCom consolidated its ownership over the lot and all the properties in it
and leased the entire factory premises to petitioner Ruby L. Tsai. PBCom sold the factory,
lock, stock and barrel to Tsai, including the contested machineries. EVERTEX filed a
complaint for annulment of sale, reconveyance, and damages with the Regional Trial Court
against PBCom, alleging inter alia that the extrajudicial foreclosure of subject mortgage
was in violation of the Insolvency Law. EVERTEX claimed that no rights having been
transmitted to PBCom over the assets of insolvent EVERTEX, therefore Tsai acquired no
rights over such assets sold to her, and should reconvey the assets. EVERTEX averred
that PBCom, without any legal or factual basis, appropriated the contested properties,
which were not included in the Real and Chattel Mortgages.

ISSUE

Whether or not the foreclosure on after acquired properties of EVERTEX is valid.

RULING

Inasmuch as the subject mortgages were intended by the parties to involve chattels, insofar
as equipment and machinery were concerned, the Chattel Mortgage Law applies, which
provides in Section 7 thereof that: “a chattel mortgage shall be deemed to cover only the
property described therein and not like or substituted property thereafter acquired by the
mortgagor and placed in the same depository as the property originally mortgaged,
anything in the mortgage to the contrary notwithstanding.” And, since the disputed
machineries were acquired in 1981 and could not have been involved in the 1975 or 1979
chattel mortgages, it was consequently an error on the part of the Sheriff to include subject
machineries with the properties enumerated in said chattel mortgages. As the auction sale
of the subject properties to PBCom is void, no valid title passed in its favor. Consequently,
the sale thereof to Tsai is also a nullity under the elementary principle of nemo dat quod
non habet, one cannot give what one does not have

Assuming arguendo that the properties in question are immovable by nature, nothing
detracts the parties from treating it as chattels to secure an obligation under the principle
of estoppel. An immovable may be considered a personal property if there is a stipulation
as when it is used as security in the payment of an obligation where a chattel mortgage is
executed over it, as in the case at bar.

4|Page
MABELLE J. ARELLANO PROPERTY
L02 Saturday, 8:00 AM-12:00 PM

CALTEX (Philippines) Inc., vs. Central Board of Assessment Appeals and City
Assessor
G.R. No. L-50466, May 31, 1982

FACTS

Caltex loaned its machines and equipment to gas station operators under an appropriate
lease agreement or receipt. The machines and equipment consists of underground tanks,
elevated tank, elevated water tanks, water tanks, gasoline pumps, computing pumps, water
pumps, car washer, car hoists, truck hoists, air compressors and tireflators. It is stipulated
in the lease contract that the operators, upon demand, shall return to Caltex the machines
and equipment in good condition as when received, ordinary wear and tear excepted. The
lessor of the land, where the gas station is located, does not become the owner of the
machines and equipment installed therein. Caltex retains the ownership thereof during the
term of the lease.

The City Assessor of Pasay City characterized the said items of gas station equipment and
machinery as taxable realty. However, the City Board of Tax Appeals ruled that they are
personalty. The assessor appealed to the Central Board of Assessment Appeals. The
Board held on June 3, 1977 that the said machines are real property within the meaning of
Sections 3(k) & (m) and 38 of the Real Property Tax Code, P.D. No. 464, which took effect
on June 1, 1974, and that the definitions of real property and personal property in articles
415 and 416 of the Civil Code are not applicable in this case.

ISSUE

Whether or not the said machines and equipment are real property subject to realty tax

RULING

The said machines and equipment are considered real property. Section 2 of the
Assessment Law provides that the realty tax is due on “real property, including land,
buildings, machinery, and other improvements” not specifically exempted in section 3
thereof. This provision is reproduced with some modification in the Real Property Tax Code
which provides:

SEC. 38 Incidence of Real Property Tax.- There shall be levied, assessed and collected in
all provinces, cities and municipalities an annual ad valorem tax on real property not
hereinafter specifically exempted.

The Code contains the following definitions in section 3:

k) Improvements- is a valuable addition made to property or an amelioration in its condition,


amounting to more than mere repairs or replacement of waste, costing labor or capital and
intended to enhance its value, beauty or utility or to adapt it for new or further purposes.

m) Machinery- shall embrace machines, mechanical contrivances, instruments, appliances


and apparatus attached to the real estate. It includes the physical facilities available for
production, as well as the installations and appurtenant service facilities, together with all
other equipment designed for or essential to its manufacturing, industrial or agricultural
purposes.

The machines and equipment are necessary to the operation of the gas station, for without
them the gas station would be useless, and which have been attached or fixed permanently
to the gas station site or embedded therein, are taxable improvements and machinery
within the meaning of the Assessment Law and the Real Property Tax Code. Therefore,
the machines and equipment are real property subject to realty tax.

5|Page
MABELLE J. ARELLANO PROPERTY
L02 Saturday, 8:00 AM-12:00 PM

Manila Electric Company, petitioner, vs. Central Board of Assessment Appeals,


Board of Assessment Appeals of Batangas and Provincial Assessor of Batangas,
respondents
G.R. No. L-47943, May 31, 1982

FACTS

This case is about the imposition of the realty tax on two oil storage tanks installed in 1969
by Manila Electric Company on a lot in San Pascual, Batangas which it leased in 1968 from
Caltex (Phil.), Inc. The tanks are within the Caltex refinery compound. They are used for
storing fuel oil for Meralco’s power plants. According to Meralco, the storage tanks are
made of steel plates welded and assembled on the spot. Their bottoms rest on a foundation
consisting of compacted earth as the outermost layer, a sand pad as the intermediate layer
and a two-inch thick bituminous asphalt stratum as the top layer. The bottom of each tank
is in contact with the asphalt layer. The steel sides of the tank are directly supported
underneath by a circular wall made of concrete, eighteen inches thick, to prevent the tank
from sliding. Hence, according to Meralco, that tank is not attached to its foundation. It is
not anchored or welded to the concrete circular wall. Its bottom plate is not attached to any
part of the foundation by bolts, screws or similar devices. The tank merely sits on its
foundation. Each empty tank can be floated by flooding its dike-inclosed location with water
four feet deep.

Meralco filed this special civil action of certiorari to annul the Board’s decision and
resolution. It contends that the Board acted without jurisdiction and committed a grave error
of law in holding that its storage tanks are taxable property.

ISSUE

Whether or not the said fuel tanks are real properties subject to Realty Tax.

RULING

The tanks are considered real properties subject to Realty Tax. While the two storage tanks
are not embedded in the land, they may, nevertheless, be considered as improvements on
the land, enhancing its utility and rendering it useful to the oil industry. It is undeniable that
the two tanks have been installed with some degree of permanence as receptacles for the
considerable quantities of oil needed by Meralco for its operations.

Oil storage tanks were held to be taxable realty in Standard Oil Co., of New Jersey vs.
Atlantic City, 15 Atl. 2nd 271.

For purposes of taxation, the term “real property” may include things which should generally
be regarded as personal property. It is a familiar phenomenon to see things classed as real
property for purposes of taxation which on general principle might be considered personal
property (Standard Oil Co. of New York vs. Jaramillon, 44 Phil. 630,633).

The case of Board of Assessment Appeals vs. Manila Electric Company, 119 Phil. 328,
wherein Meralco’s steel towers were held not to be subject to realty tax, is not in point
because in that case the steel towers were regarded as poles and under its franchise
Meralco’s poles are exempt from taxation. Moreover, the steel towers were not attached to
any land or building. They were removable from their metal frames.

Nor is there any parallelism between this case and Mindanao Bus Co. vs. City Assessor,
116 Phil. 501, where the tools and equipment in the repair, carpentry and blacksmith shops
of a transportation company were held not subject to realty tax because they were personal
property.

6|Page
MABELLE J. ARELLANO PROPERTY
L02 Saturday, 8:00 AM-12:00 PM

Metropolitan Bank & Trust Company vs. Hon. Floro T. Alejo


G.R. No. 141970
September 10, 2001

FACTS

On November 21, 1995 and January 30, 1996, spouses Raul and Christina Acampado
obtained loans from petitioner in the amounts of P5,000,000.00 and P2,000,000.00,
respectively. As security for the payment of these credit accomodations, the spouses
executed in favor of petitioner a Real Estate Mortgage and an Amendment of Real Estate
Mortgage over a parcel of land registered in their names. The land was covered by TCT
No. V-41319 in the Registry of Deeds of Valenzuela City. On June 3, 1996, Complaint for
Declaration of Nullity of TCT No. V-41319 was filed by respondent Sy Tan Se against the
spouses. It was docketed as Civil Case No. 4930-V-96 in the Regional Trial Court (RTC)
of Valenzuela. Despite being the registered mortgagee of the real property covered by the
title sought to be annulled, petitioner was not made a party to the case nor was she notified
of its existence. The spouses defaulted in the payment of their loan and extrajudicial
foreclosure proceedings were initiated wherein the petitioner submitted the highest and
winning bid. A Certificate of Sale was issued in their favor. Upon presentation to the
Register of Deeds of the Affidavit of Consolidation of Ownership, petitioner was informed
of the decision in the aforementioned civil case (complaint for declaration of nullity of TCT)
declaring the spouses’ TCT null and void. The petitioner filed with the CA a petition for
annulment of the RTC Decision. The CA dismissed their petition and ruled that the bank
should have filed a petition for relief from judgment or an action of quieting of title.

ISSUES

1. Whether or not a petition for annulment of judgment is the proper remedy of the
petitioner
2. Whether or not the judgment of the trial court in Civil Case No. 4930-V-96 should
be annulled

RULING

1. YES. Petition for annulment of judgment was the proper remedy of the petitioner. It
precisely alleged that private respondent purposely concealed the case by
excluding petitioner as a defendant to the civil case even if he was an indispensable
party. This deprived the petitioner of its duly registered property without due
process of law. The allegation of extrinsic fraud may be the basis for annulling a
judgment. Petition for relief (what the CA recommended) was not available to the
petitioner since it was never a party to the civil case. An action for quieting of title is
also not available to the petitioner. An action for quieting of title is filed only when
there is a cloud on title to real property or any interest therein. The subject judgment
cannot be considered as a cloud on petitioner’s title or interest over real property
covered by TCT, which does not even have a semblance of being a title. Moreover,
the petitioner cannot intervene to a case to which he has no knowledge of.

2. YES. The judgement of the trial court should also be declared null and void because
the petitioner, who is an indispensable party, was not impleaded in the civil case.
The absence of an indispensable party renders all subsequent actuations of the
court null and void, for want of authority to act, not only as to the absent parties but
even as to those present.

7|Page
MABELLE J. ARELLANO PROPERTY
L02 Saturday, 8:00 AM-12:00 PM

Francisco I. Chavez vs. Public Estates Authority and AMARI Coastal Bay
Development Corporation
G.R. No. 133250, November 11, 2003

FACTS

From the time of Marcos until Estrada, portions of Manila Bay were being reclaimed. A law
was passed creating the Public Estate Authority which was granted with the power to
transfer reclaimed lands. Now in this case, PEA entered into a Joint Venture Agreement
with AMARI, a private corporation. Under the Joint Venture Agreement between AMARI
and PEA, several hectares of reclaimed lands comprising the Freedom Islands and several
portions of submerged areas of Manila Bay were going to be transferred to AMARI.

ISSUE

Whether or not the stipulations in the Amended JVA for the transfer to AMARI of lands,
reclaimed or to be reclaimed, violate the Constitution.

RULING

Under the Public Land Act (CA 141, as amended), reclaimed lands are classified as
alienable and disposable lands of the public domain Section 3 of the Constitution: Alienable
lands of the public domain shall be limited to agricultural lands. Private corporations or
associations may not hold such alienable lands of the public domain except by lease The
157.84 hectares of reclaimed lands comprising the Freedom Islands, now covered by
certificates of title in the name of PEA, are alienable lands of the public domain. PEA may
lease these lands to private corporations but may not sell or transfer ownership of these
lands to private corporations.

PEA may only sell these lands to Philippine citizens, subject to the ownership limitations in
the 1987 Constitution and existing laws. Clearly, the Amended JVA violates glaringly
Sections 2 and 3, Article XII of the 1987 Constitution. Under Article 1409 of the Civil Code,
contracts whose “object or purpose is contrary to law,” or whose “object is outside the
commerce of men,” are “inexistent and void from the beginning.” The Court must perform
its duty to defend and uphold the Constitution, and therefore declares the Amended JVA
null and void ab initio.

8|Page
MABELLE J. ARELLANO PROPERTY
L02 Saturday, 8:00 AM-12:00 PM

Republic of the Philippines vs. Michael C. Santos, et al.


G.R. No. 180027, July 18, 2012

FACTS
Respondents purchased three parcels of unregistered land formerly owned by Generosa
Asuncion, Teresita Sernal, and spouses Antona. The three parcels of land were
consolidated into a single lot (Lot 3). Respondents filed with the RTC an application for
original registration over Lot 3. The RTC directed the DENR to report on the status of Lot
3. DENR submitted a report saying that Lot 3 is an alienable and disposable land since
March 15, 1982. Respondents submitted a Certification by the DENR-Community
Environment and Natural Resources Office (CENRO) that Lot 3 is classified as alienable
and disposable since March 15, 1982. During the trial, respondents presented the
testimonies of Generosa, Teresita, and spouses Antona all saying that they have been in
possession of the lots for more than 30 years before the sale. The Government maintains
that the land in question still forms part of the public domain.

ISSUE
Whether or not the respondents’ application for registration should be granted.

RULING
No. Jura Regalia means that the State is the original proprietor of all lands and the source
of all private titles. Being an unregistered land, Lot 3 is presumed to belong to the state.
Those who seek the entry of such land into the Torrens system of registration must first
establish valid title thereto as against the state. Respondents failed to establish valid title
in this case. They anchored their claim on prescription provided under Section 14(2) of PD
1529; for prescription to run against the state, the land must be proven to be patrimonial in
character. To be patrimonial, there must be an express declaration by the state that the
land is no longer needed for public service or the development of national wealth, or that
the property has been converted to patrimonial. Until then, the period of prescription
against the state will not commence to run. The express declaration contemplated is
separate and distinct from mere classification that the land is alienable and disposable.
Respondents were not able to prove that prescription has begun to run against the state.

9|Page
MABELLE J. ARELLANO PROPERTY
L02 Saturday, 8:00 AM-12:00 PM

Teofilo C. Villarico vs. Vivencio Sarmiento, et al.


G. R. No. 136438, November 11, 2004

FACTS

 Villarico here is an owner of a lot that is separated from the Ninoy Aquino Avenue
Highway by a strip of land belonging to the government.
 Vivencio Sarmiento had a building constructed on a portion of the said
government land and a part thereof was occupied by Andok’s Litson Corp.
 In 1993, by means of a Deed of Exchange of Real Property, Villarico acquired a
portion of the same area owned by the government.
 He then filed an accion publiciana alleging that respondents (Vivencio) on the
government land closed his right of way to the Ninoy Aquino Avenue and
encroached on a portion of his lot.

ISSUE

Whether or not Villarico has a right of way to Ninoy Aquino Avenue.

RULING

No. It is not disputed in this case that the alleged right of way to the lot belongs to the state
or property of public dominion. It is intended for public use meaning that it is not confined
to privileged individuals but is open to the indefinite public. Records show that the lot on
which the stairways were built is for the use of the people as passageway hence, it is a
property for public dominion.

Public dominion property is outside the commerce of man and hence, it cannot be:

1. Alienated or leased or otherwise be the subject matter of contracts


2. Acquired by prescription against the state
3. Cannot be the subject of attachment and execution
4. Be burdened by any voluntary easement

It cannot be burdened by a voluntary easement of right of way in favor of the petitioner and
petitioner cannot appropriate it for himself and he cannot claim any right of possession over
it.

10 | P a g e
MABELLE J. ARELLANO PROPERTY
L02 Saturday, 8:00 AM-12:00 PM

Jose Menchavez, et al vs. Florentino Teves, Jr.


G.R. No. 153201, January 26, 2005

FACTS

On February 28, 1986, Menchavez (lessors) and Forentino Teves, Jr. (lessee) executed a
contract of lease. On June 2, 1988, Cebu RTC Sheriffs demolished the fishpond dikes
constructed by the respondent and delivered the possession of the subject property to the
plaintiffs. Respondent filed a complaint for damages and alleged that the lessors have
violated their contract, specifically the peaceful and adequate enjoyment of the property for
the entire duration of the contract. RTC declared the contract of lease between the parties
as void ab initio as it is owned by the State based on the Regalian Doctrine. RTC ruled in
favor of the petitioners.

Respondent elevated the case to the Court of Appeals (CA). The CA disagreed with RTC’s
finding that petitioners and respondents were in pari delicto. It contended that while there
was negligence on the part of the respondent for failing to verify the ownership of the
property, there was no evidence that he had knowledge of petitioners’ lack of ownership.
Hence, this petition.

ISSUE

Whether or not the subject property (fishponds) can be leased by the petitioners.

RULING

No. It was the state who owned the fishpond. The 1987 Constitution specifically declares
that all lands of the public domain, waters, fisheries and other natural resources belong to
the State. Included here are fishponds, which may not be alienated but only leased.
Possession thereof, no matter how long, cannot ripen into ownership.

Being merely applicants or the lease of the fishponds, petitioners had no transferable right
over them. And even if the State were to grant their application, the law expressly
disallowed sublease of the fishponds to respondent. The contract of lease is void ab initio.
No damages may be recovered from a void contract.

Wherefore, the petition is granted and the assailed Decision and Resolution set aside. The
Decision of the trial court is hereby reinstated.

11 | P a g e
MABELLE J. ARELLANO PROPERTY
L02 Saturday, 8:00 AM-12:00 PM

Francisco U. Dacanay vs. Mayor Macario Asistio, et al.


G.R. No. 93654, May 6, 1992

FACTS

An ordinance was issued designated certain city and municipal streets, roads, and other
public areas for sites of public markets. Pursuant to this, licenses were issued to market
stall owners to put up their stalls in certain streets. Thereafter, the OIC mayor of Caloocan
has caused the demolition
of the stalls, which was upheld by the trial court, saying that the public streets are
part of the public dominion and is not open to the commerce of man.

Then there come about a change in administration of the city. The


next mayor did not continue the demolition of the stalls. Using the trial court’s decision,
here now comes petitioner asking for the demolition of the stalls.

ISSUE

Whether or not public streets be leased or licensed to market stallholders by virtue of a city
ordinance or resolution of the Metro Manila Commission.

RULING

There is no doubt that the disputed areas from which the private respondents' market stalls
are sought to be evicted are public streets. A public street is property for public use hence
outside the commerce of man. Being outside the commerce of man, it may not be the
subject of lease or other contract.

As the stallholders pay fees to the City Government for the right to occupy portions of the
public street, the City Government, contrary to law, has been leasing portions of the streets
to them. Such leases or licenses are null and void for being contrary to law. The right of
the public to use the city streets may not be bargained away through contract. The interests
of a few should not prevail over the good of the greater number in the community whose
health, peace, safety, good order and general welfare, the respondent city officials are
under legal obligation to protect.

12 | P a g e
MABELLE J. ARELLANO PROPERTY
L02 Saturday, 8:00 AM-12:00 PM

Manila Lodge No. 761, Benevolent and Protective Order of the Elks, Inc. vs. The
Honorable Court of Appeals, et al.
G.R. No. L-41001, September 30, 1976

FACTS

The Philippine Commission enacted Act No. 1306 which authorized the City of Manila to
reclaim a portion of Manila Bay. The reclaimed area was to form part of the Luneta
extension. The act provided that the reclaimed area shall be the property of the City of
Manila, and the city is authorized to set aside a tract of the reclaimed land for a hotel site
and to lease or to sell the same. Later, the City of Manila conveyed a portion of the
reclaimed area to Petitioner. Then Petitioner sold the land, together with all the
improvements, to the Tarlac Development Corporation (TDC).

ISSUE
Whether or not the subject property was patrimonial property of the City of Manila.

RULING

The petitions were denied for lack of merit. The court found it necessary to analyze all the
provisions of Act No. 1360, as amended, in order to unravel the legislative intent. The grant
made by Act No. 1360 of the reclaimed land to the City of Manila is a grant of a “public”
nature. Such grants have always been strictly construed against the grantee because it is
a gratuitous donation of public money or resources, which resulted in an unfair advantage
to the grantee. In the case at bar, the area reclaimed would be filled at the expense of the
Insular Government and without cost to the City of Manila. Hence, the letter of the statute
should be narrowed to exclude matters which, if included, would defeat the policy of
legislation.

13 | P a g e
MABELLE J. ARELLANO PROPERTY
L02 Saturday, 8:00 AM-12:00 PM

Republic of the Philippines (Director of Lands) vs. Honorable Court of Appeals, et


al.
G.R. No. L-61647, October 12, 1984

FACTS

The respondents (Tancinco’s) were registered owners of a parcel of land in Bulacan,


bordering on the Maycauayan and Bocaue Rivers. They filed an application for the
registration of three lots adjacent to their fishpond, but because of the recommendation of
the Commissioner, they only pushed for the registration of two. The RTC and CA granted
the petition despite the opposition of the Bureau of Lands.

The respondents based their claim on accretions to their fishponds. They presented a lone
witness (their overseer). The Bureau of Lands argue that the lands in dispute are not
accretions. They assert that what actually happened was that the respondents simply
transferred their dikes simply further down the river bed of the Meycauayan River. Thus, if
there was any accretion to speak of, it was man-made.

Respondents counter that their evidence shows that accretion happened without human
intervention and that the transfer of the dikes occurred only after.

ISSUE

Whether or not accretion took place.

RULING

Alluvion must be the exclusive work of nature. There is no evidence that the addition to
said property was made gradually through the effects of the currents of the two rivers. The
lands in question total almost 4 hectares of land, which are highly doubtful to have been
caused by accretion. The lone witness testified that she observed an increase in the area
in 1939, but the lots in question were not included in the survey of their adjacent property
conducted in 1940. They were also not included in the Cadastral Survey of the entire
Municipality of Maycauayan between the years 1958-1960. If the overseer was indeed
telling the truth, the accretion was sudden, not gradual. When the respondents transferred
their dikes towards the river beds, the dikes were meant for reclamation purposes and not
to protect their property from the destructive force of the waters of the river. The lots in
question were portions of the bed of the Meycauayan River and are therefore classified as
public property.

Registration denied, decisions appealed are reversed. Note: The lands sought were not
even dry land. The entire area was under one to two meters of water.

14 | P a g e
MABELLE J. ARELLANO PROPERTY
L02 Saturday, 8:00 AM-12:00 PM

MANILA INTERNATIONAL AIRPORT AUTHORITY, petitioner,


vs.
COURT OF APPEALS, CITY OF PARAÑAQUE, CITY MAYOR OF PARAÑAQUE,
SANGGUNIANG PANGLUNGSOD NG PARAÑAQUE, CITY ASSESSOR OF
PARAÑAQUE, and CITY TREASURER OF PARAÑAQUE, respondents
G.R. No. 155650, July 20, 2006

DOCTRINE: The term “ports” includes seaports and airports. The MIAA Airport Lands and
Buildings constitute a “port” constructed by the State. Under Article 420 of the Civil Code,
the MIAA Airport Lands and Buildings are properties of public dominion and thus owned by
the State or the Republic of the Philippines.

FACTS

Manila International Airport Authority (MIAA) operates the Ninoy Aquino International
Airport Complex in Parañaque City. As operator of the international airport, MIAA
administers the land, improvements and equipment within the NAIA Complex.

The MIAA Charter transferred to MIAA approximately 600 hectares of land including the
runways and buildings (“Airport Lands and Buildings”) then under the Bureau of Air
Transportation. The MIAA Charter further provides that no portion of the land transferred
to MIAA shall be disposed of through sale or any other mode unless specifically approved
by the President of the Philippines.

OGCC (Office of the Government Corporate Counsel) issued Opinion No. 061, in which it
said that the Local Government Code of 1991 withdrew the exemption for real estate tax
granted to MIAA under Section 21 of the MIAA charter.

Therefore, MIAA was held to be delinquent in paying its taxes. The City of Parañaque
Levied upon the properties of MIAA, and posted invitations for public biddings of MIAA’s
properties. MIAA filed with CA an action for prohibition / injunction. The City of Parañaque
averred that Section 193 of the Local Government code expressly withdrew tax exemptions
from government owned and controlled corporations (GOCCs).

CA dismissed the petition for filing beyond the 60 day reglementary period

ISSUE

Whether properties of the MIAA are subject to real estate taxes.

RULING

No. In the first place, MIAA is not a GOCC, it is an instrumentality of the government. MIAA
is a government instrumentality vested with corporate powers to perform efficiently its
governmental functions. MIAA is like any other government instrumentality, the only
difference is that MIAA is vested with corporate powers. As operator of the international
airport, MIAA administers the land, improvements and equipment within the NAIA
Complex. The MIAA Charter transferred to MIAA approximately 600 hectares of land,
including the runways and buildings (“Airport Lands and Buildings”) then under the Bureau
of Air Transportation. The MIAA Charter further provides that no portion of the land
transferred to MIAA shall be disposed of through sale or any other mode unless specifically
approved by the President of the Philippines.

Furthermore, Airport Lands and Buildings of MIAA are property of public dominion and
therefore owned by the State or the Republic of the Philippines. Article 419 of the Civil
Code provides, The Airport Lands and Buildings of MIAA are property of public dominion
and therefore owned by the State or the Republic of the Philippines.

The Civil Code provides:


15 | P a g e
MABELLE J. ARELLANO PROPERTY
L02 Saturday, 8:00 AM-12:00 PM

ARTICLE 419. Property is either of public dominion or of private ownership.

ARTICLE 420. The following things are property of public dominion:


(1) Those intended for public use, such as roads, canals, rivers, torrents,
ports and bridges constructed by the State, banks, shores, roadsteads, and
others of similar character;
(2) Those which belong to the State, without being for public use, and are
intended for some public service or for the development of the national
wealth.
ARTICLE 421. All other property of the State, which is not of the character stated
in the preceding article, is patrimonial property.

ARTICLE 422. Property of public dominion, when no longer intended for public use
or for public service, shall form part of the patrimonial property of the State.

No one can dispute that properties of public dominion mentioned in Article 420 of the Civil
Code, like “roads, canals, rivers, torrents, ports and bridges constructed by the State,” are
owned by the State. The term “ports” includes seaports and airports. The MIAA Airport
Lands and Buildings constitute a “port” constructed by the State. Under Article 420 of the
Civil Code, the MIAA Airport Lands and Buildings are properties of public dominion and
thus owned by the State or the Republic of the Philippines.

The Airport Lands and Buildings are devoted to public use because they are used by the
public for international and domestic travel and transportation. The fact that the MIAA
collects terminal fees and other charges from the public does not remove the character of
the Airport Lands and Buildings as properties for public use. The operation by the
government of a tollway does not change the character of the road as one for public use.
Someone must pay for the maintenance of the road, either the public indirectly through the
taxes they pay the government, or only those among the public who actually use the road
through the toll fees they pay upon using the road. The tollway system is even a more
efficient and equitable manner of taxing the public for the maintenance of public roads.

The charging of fees to the public does not determine the character of the property whether
it is of public dominion or not. Article 420 of the Civil Code defines property of public
dominion as one “intended for public use.” Even if the government collects toll fees, the
road is still “intended for public use” if anyone can use the road under the same terms and
conditions as the rest of the public. The charging of fees, the limitation on the kind of
vehicles that can use the road, the speed restrictions and other conditions for the use of
the road do not affect the public character of the road.

The terminal fees MIAA charges to passengers, as well as the landing fees MIAA charges
to airlines, constitute the bulk of the income that maintains the operations of MIAA. The
collection of such fees does not change the character of MIAA as an airport for public use.
Such fees are often termed user’s tax. This means taxing those among the public who
actually use a public facility instead of taxing all the public including those who never use
the particular public facility. A user’s tax is more equitable — a principle of taxation
mandated in the 1987 Constitution.

The Airport Lands and Buildings of MIAA, which its Charter calls the “principal airport of
the Philippines for both international and domestic air traffic,” are properties of public
dominion because they are intended for public use. As properties of public dominion, they
indisputably belong to the State or the Republic of the Philippines.

Being a property of public dominion, the properties of MIAA are beyond the commerce of
man.

16 | P a g e
MABELLE J. ARELLANO PROPERTY
L02 Saturday, 8:00 AM-12:00 PM

Vergo D. Tufexis vs. Francisco Olaguera


G.R. No. L-9865, December 14, 1915

FACTS

It was alleged that on September 30, 1911, plaintiff acquired at a public sale held in
execution of a judgment rendered against Ricardo Pardo y Pujol, a piece of property
situated in the municipality of Guinobatan, consisting of a frame building of strong materials
with a galvanized-iron roof, erected on a parcel of land belonging to that municipality and
intended for a public market. The plaintiff also acquired at the sale all the right, interest,
title, and participation in the said property that appertained or might appertain to Pardo y
Pujol. The said building was constructed by virtue of a concession granted by the former
Spanish government to Ricardo Pardo y Cabañas, father of the judgment debtor.

On January 2, 1912, the said building was totally destroyed by an accidental fire. For
several months thereafter the municipal council of Guinobatan negotiated with plaintiff for
the purchase of his rights in the said concession but such could not be brought to a
conclusion because the municipal council had allegedly acted deceitfully, fraudulently, and
in bad faith for the sole purpose of beguiling, deceiving, and prejudicing plaintiff in order to
prevent him from exercising his right to reconstruct the burned market building and utilize
it in accordance with the terms of the said concession. The defendant municipal council
with the other defendant, Francisco Olaguera, had authorized the latter to take possession
of all the land and to occupy the same with booths or stores for the sale of groceries and
other merchandise, for billiard tables, and other analogous. The plaintiff proposed to
construct another public market building on the same land, but that the defendants had
prevented him from using the land and reconstructing thereon the said public market
building, and refused to recognize plaintiff's right and to vacate the land that had been
occupied by the burned edifice.

After filing a petition before the CFI of Albay, the provincial fiscal alleged as a ground for
the demurrer that in no part of the instrument of concession did it appear that the privilege
granted to the father of the judgment debtor had likewise been granted to his successors
or assignees, and that therefore such rights and actions could not be conveyed to nor be
acquired by any other person. It was alleged that the building was completely destroyed by
fire and that if plaintiff's right to the possession of the land was conditioned by the existence
thereon of the said market building, such right had terminated by the disappearance of the
building.

ISSUE

Whether a the subject building on land belonging to the municipality of Guinobatan which
was intended for a public market, by virtue of a concession could be attached and sold for
the payment of a certain debt owed by Ricardo Pardo y Pujol to a third person who had
obtained a final judgment.
RULING
No. The land on which the building was erected and which is referred to in the foregoing
articles of the concession granted by the Government of the former sovereignty belongs to
the municipality of Guinobatan. Although the building was constructed at the expense and
with the money of the grantee, Ricardo Pardo y Cabañas, it is, nevertheless, the property
of the state or of the said municipality, and was temporarily transferred to the grantee,
Pardo y Cabañas, in order that he might enjoy the usufruct of its floor space for forty years,
but on the termination of this period the said right of usufruct was to cease and the building
was to belong finally and absolutely to the state or the municipality in representation
thereof.
For these reasons, there is no question that the building and the land, on which it was
erected, since they did not belong to the grantee, nor do they belong to his son and heir,
Ricardo Pardo y Pujol, could not be attached or sold for the payment of a debt contracted

17 | P a g e
MABELLE J. ARELLANO PROPERTY
L02 Saturday, 8:00 AM-12:00 PM

by the latter. The concession granted by the former Spanish Government is personal and
transferable only by inheritance, and in no manner could it be conveyed as a special
personal privilege to another and a third person.

18 | P a g e
MABELLE J. ARELLANO PROPERTY
L02 Saturday, 8:00 AM-12:00 PM

ROMAN R. SANTOS, petitioner-appellee, 
vs.
HON. FLORENCIO MORENO, as


Secretary of Public Works and Communications and JULIAN C.
CARGULLO, respondents-appellants.
G.R. No. L-15829

FACTS

The Zobel family of Spain formerly owned vast track of marshland in the municipality of
Macabebe, Pampanga province called Hacienda San Esteban. It was administered and
managed by the Ayala y Cia. From the year 1860 to about the year 1924 Ayala y Cia.,
devoted the hacienda to the planting and cultivation of nipa palms from which it gathered
nipa sap or "tuba." It operated a distillery plant in barrio San Esteban to turn nipa tuba into
potable alcohol, which was in turn manufactured into liquor.

Accessibility through the nipa palms deep into the hacienda posed as a problem. Ayala y
Cia., therefore dug canals leading towards the hacienda's interior where most of them
interlinked with each other. The canals facilitated the gathering of tuba and the guarding
and patrolling of the hacienda by security guards called "arundines." By the gradual
process of erosion these canals acquired the characteristics and dimensions of rivers.

In 1924, Ayala y Cia shifted from the business of alcohol production to bangus culture. It
converted Hacienda San Esteban from a forest of nipa groves to a web of fishponds. To
do so, it cut down the nipa palm, constructed dikes and closed the canals criss-crossing
the hacienda.

Sometime in 1925 or 1926 Ayala y Cia., sold a portion of Hacienda San Esteban to Roman
Santos who also transformed the swamp land into a fishpond. In so doing, he closed and
built dikes across Sapang Malauling Maragul, Quiñorang Silab, Pepangebunan, Bulacus,
Nigui and Nasi.

The closing of the man-made canals in Hacienda San Esteban drew complaints from
residents of the surrounding communities. Claiming that the closing of the canals caused
floods during the rainy season, and that it deprived them of their means of transportation
and fishing grounds, said residents demanded re-opening of those canals.

Subsequently, Mayor Lazaro Yambao of Macabebe, accompanied by policemen and some


residents went to Hacienda San Esteban and opened the closure dikes at Sapang
Malauling Maragul Nigui and Quiñorang Silab.

Whereupon, Roman Santos filed Civil Case No. 4488 in the Court of First Instance of
Pampanga which preliminarily enjoined Mayor Yambao and others from demolishing the
dikes across the canals. The municipal officials of Macabebe countered by filing a
complaint (docketed as Civil Case No. 4527) in the same court. The Pampanga Court of
First Instance rendered judgment in both cases against Roman Santos who immediately
elevated the case to the Supreme Court.

ISSUE

Whether or not the streams involved in this case belong to the public domain or to the
owner of Hacienda San Esteban according to law and the evidence submitted to the
Department of Public Works and Communications.

RULING

A private person may take possession of a watercourse if he constructed the same within
his property.

One and all, the evidence, oral and documentary, presented by Roman Santos in the
administrative proceedings supports the conclusion of the lower court that the streams

19 | P a g e
MABELLE J. ARELLANO PROPERTY
L02 Saturday, 8:00 AM-12:00 PM

involved in this case were originally man-made canals constructed by the former owners
of Hacienda San Esteban and that said streams were not held open for public use. This
same conclusion was reached 27 years earlier by an investigator of the Bureau of Public
Works whose report and recommendations were approved by the Director of Public Works
and submitted to the Secretary of Commerce and Communications.

The streams in question were artificially made, hence of private ownership.

Pursuant to Article 71 of the Spanish Law of Waters of August 3, 1866, and Article 408(5)
of the Spanish Civil Code, channels of creeks and brooks belong to the owners of estates
over which they flow. The channels, therefore, of the streams in question, which may be
classified creeks, belong to the owners of Hacienda San Esteban.

With the exception of Sapang Cansusu, being a natural stream and a continuation of the
Cansusu River, admittedly a public stream, belongs to the public domain. Its closure
therefore by the predecessors of Roman Santos was illegal.

All the other streams, being artificial and devoted exclusively for the use of the hacienda
owner and his personnel, are declared of private ownership. Hence, the dams across them
should not he ordered demolished as public nuisances.

20 | P a g e
MABELLE J. ARELLANO PROPERTY
L02 Saturday, 8:00 AM-12:00 PM

Almagro vs. Kwan


G.R. NO. 175806

FACTS

The heirs of spouses Kwan filed with the MTC an action for recovery of possession and
damages against spouses Duran among others. However, the MTC dismissed the
complaint on the ground that the remaining dry portion the subject lot has become
foreshore land and should be returned to the public domain. It explained that a big portion
of the said lot is presently underwater or submerged under the sea. When the sea moves
towards the estate and the tide invades it, the invaded property becomes foreshore land
and passes to the realm of public domain. The subject land, being foreshore land, should
therefore be returned to the public domain. On appeal to the RTC, the court observed that
the small portion of the subject lot actually remained dry even during high tide. Thus, the
small dry portion of the land is not within the scope of the well-settled definition of foreshore
and foreshore land as it is not adjacent to the sea and it is not alternately wet and dry by
the ordinary flow of the tides as it is dry land.
ISSUE

Whether the disputed portion of the subject has become foreshore land.
RULING

No. The small dry portion of the subject lot is not foreshore because "it is already dry land"
and is "away from the shoreline." The land's proximity to the waters alone does not
automatically make it a foreshore land. The disputed land remained dry even during high
tide. It is not foreshore land and remains private land.

21 | P a g e
MABELLE J. ARELLANO PROPERTY
L02 Saturday, 8:00 AM-12:00 PM

SIMPLICIO BINALAY, PONCIANO GANNABAN, NICANOR MACUTAY, DOMINGO


ROSALES, GREGORIO ARGONZA, EUSTAQUIO BAUA, FLORENTINO ROSALES,
TEODORO MABBORANG, PATRICIO MABBORANG and FULGENCIO MORA,
petitioners
vs.
GUILLERMO MANALO and COURT OF APPEALS, respondents
G.R. No. 92161, March 18, 1991

FACTS

Manalo acquired 2 lots which were originally owned by Judge Taccad from 2 different
people (the latter’s daughter and from an earlier purchaser). These lots were later
consolidated into Lot 307, a total of 10.45 hectares. The lot was beside the Cagayan River,
which, due to flooding, would place a portion of the land underwater during the rainy season
(September to December). On sunny days, however, the land would be dried up for the
entire dry season (January to August). When a survey of the land was conducted on a rainy
month, a portion of the land that Manalo bought was then underwater and was thus left
unsurveyed and excluded from Lot 307.

The big picture is this: Cagayan River running from south to north, forks at a certain point
to form two braches (western and eastern) and then unites at the other end, further north,
to form a narrower strip of land. The eastern branch of the river cuts through Lot 307, and
is flooded during the rainy season. The unsurveyed portion, on the other hand, is the bed
of the eastern branch. Note that the fork exists only during the rainy season while the
“island”/elongated strip of land formed in the middle of the forks becomes dry and perfect
for cultivation when the Cagayan river is at its ordinary depth. The strip of land in the middle
of the fork totaled 22.7 hectares and was labeled Lot 821-822. Lot 821 is directly opposite
Lot 307 and is separated by the eastern branch of the river’s fork.

Manalo claims that Lot 821 belongs to him by way of accretion to the submerged portion
of the land to which it is adjacent. Petitioners (Binalay, et al) who possess the Lot 821, on
the other hand, insist that they own it. They occupy the other edges of the lot along the
river bank (i.e. the fertile portions on which they plant tobacco and other agricultural
products) and also cultivate the western strip during the summer.

Manalo filed 2 cases for forcible entry which were both dismissed. Later on, he filed a
complaint for quieting of title, possession, and damages against petitioner. The trial court
and the CA ruled in favor of Manalo, saying that Lot 821 and Lot 307 cannot be considered
separate and distinct from each other. They reasoned that when the land dries up for the
most part of the year, the two are connected. [Note: The CA applied the ruling in Gov’t of
the Phil Islands vs. Colegio de San Jose, which was actually inappropriate because the
subject matter in this case was a lake so that the definition of a “bed” was different.]

ISSUE

Whether or not Manalo owns Lot 821 by way of accretion.

RULING

No. The disputed property is not an accretion. It is the action of the heavy rains that cause
the highest ordinary level of waters of the Cagayan River during the rainy season. The
depressed portion is a river bed and is thus considered property of public domain. The SC
observed the following:

a) The pictures identified by Manalo during his direct examination depict the
depressed portion as a river bed. The dried up portion had dike-like slopes (around
8m) on both sides connecting it to Lot 307 and Lot 821 that are vertical and very
prominent.
b) The eastern bed already existed even before Manalo bought the land. It was called
“Rio Muerte de Cagayan.”
c) Manalo could not have acquire ownership of the land because article 420 of the
civil code states that rivers are property of public dominion. The word “river”
22 | P a g e
MABELLE J. ARELLANO PROPERTY
L02 Saturday, 8:00 AM-12:00 PM

includes the running waters, the bed, and the banks. [The seller never actually
owned that part of the land since it was public property]
d) The submerged area (22.72 ha) is twice the area of the land he actually bought. It
is difficult to suppose that such a sizable area could have been brought about by
accretion.

More importantly, the requisites of accretion in article 457 were not satisfied. These are: 1)
that the deposition of the soil or sediment be gradual and imperceptible; 2) that it be the
result of the action of the waters of the river (or sea); and 3) the land where the accretion
takes place is adjacent to the banks of the rivers (or the sea coast).

The accretion should’ve been attached to Lot 307 for Manalo to acquire its ownership.
BUT, the claimed accretion lies on the bank of the river; not adjacent to Lot 307 but directly
opposite it – across the river. Aside from that, the dike-like slopes which were very steep
may only be formed by a sudden and forceful action like flooding. The steep slopes could
not have been formed by the river in a slow and gradual manner.

23 | P a g e
MABELLE J. ARELLANO PROPERTY
L02 Saturday, 8:00 AM-12:00 PM

JOSE V. HILARIO, JR., plaintiff-appellant,


vs.
THE CITY OF MANILA, defendant-appellee,
DIRECTOR OF PUBLIC WORKS, CITY ENGINEER OF MANILA, FERNANDO
BUSUEGO and EUGENIO SESE, defendants-appellants,
G.R. No. L-19570, April 27, 1967

FACTS

 Dr. Jose Hilario was the registered owner of a 49 hectares land in


Barrio Guinayang, San Mateo, Rizal.
 Upon his death, it was inherited by his son, Hilario, Jr..
 Hilario estate was bounded on the western side by the San Mateo River.
 To prevent its entry into the land, a bamboo and lumber post dike or ditch was
constructed on the northwestern side. This was further fortified by a stonewall built on
the northern side. For years, these safeguards served their purpose.
 1937 - an extraordinary flood occurred in the entire place including the neighboring
barrios and municipalities. The River destroyed the dike on the northwest, left its
original bed and meandered into the Hilario estate, segregating from the rest thereof
a lenticular piece of land. The disputed area is on the eastern side of
this lenticular strip which now stands between the old riverbed site and
the newcourse."
 1945 - U.S. Army opened a sand and gravel plant within the premises and started
scraping, excavating and extracting soil, gravel and sand from the nearby areas along
the River. The operations eventually extended northward into this strip of land.
 A claim for damages was filed with the U.S. War Department by Luis Hidalgo,
administrator of Dr. Hilario’s estate. U.S. Army paid.
 1947 - the plant was turned over to City of Manila, who took over its operations and
continued the extractions and excavations of gravel and sand from the strip of land
along an area near the River.
 Hilario filed a complaint for injunction and damages against City Engineer of Manila,
District Engineer of Rizal, the Director of Public Works, and Engr. Bosuego, the
Engineer-in-charge of the plant.
 Hilario prayed that:
1. To stop the excavating, bulldozing and extracting gravel, sand and soil from his
property;
2. To solidarily pay to him P5,000.00 as damages.
 Defendants' affirmed that the extractions were made from the riverbed.
 Intervenors, Bureau of Mines and Atty. Maximo Calalang, were allowed to join the
litigation.
 Bureau of Mines complained that the disputed area was within the bed of the River so
that plaintiff should not only be enjoined from making extractions there from but should
also be ordered to pay the fees and penalties for the materials taken by him.
 Atty. Calalang claimed that he was authorized by plaintiff to extract materials from the
disputed area.
 March 14, 1954 - Defendants filed a petition for injunction against plaintiff and
intervenor Calalang alleging and prayed that:
1. the latter have fence off the disputed area in contravention of an agreement had
between the latter and the Director of Public Works where¬in the defendants were
allowed to continue their operations but subject to the final outcome of the pending
suit.
2. To remove the fence and allow them to continue their operations.
 A counter injunction was filed.
 March 23, 1954 - Lower court issued an order
1. Maintaining the status quo;
2. Allowing them to continue their extractions from the disputed area provided a
receipt in plaintiff's favor be issued for all the materials taken.
 May 13, 1954 - Plaintiff amended his complaint. Impleaded as additional defendants
were the City of Manila, the Provincial Treasurer of Rizal, and Engr. Sese, the new
Engineer-in-charge of the plant.
24 | P a g e
MABELLE J. ARELLANO PROPERTY
L02 Saturday, 8:00 AM-12:00 PM

 Plaintiff also converted his claim to one purely for damages directed against the City
of Manila and the Direc¬tor of Public Works, in the amount of P1,000,000.00, as the
cost of materials taken since 1949, as well as those to be extracted there from until
they stop their operations.
 Manila City denied ownership of the plant and claimed that the City Engineer acted
merely as a deputy of the Public Works Director.
 The other defendants put up, as special defense, the agreement between plaintiff and
the Public Works Director, and asserted a P1.2 million counterclaim for damages
against plaintiff.
 The rest renewed the same defense: that the disputed area was part of the public
domain, since it was situated on the riverbanks.
 November 3, 1954 - City Engineer of Manila filed a petition to delimit the area of
excavation and asked the lower court to authorize his men to extend their operations
west of the camachile tree in the disputed area. But, later DENIED
 Lower Court’s Decision: (against defendants)
1. City of Manila and the Director of Public Works, to pay solidarily, P376,989.60,
for the cost of gravel and sand extracted from the land.
2. Provincial Treasurer of Rizal, to reimburse to intervenor Calalang, P236.80 for
gravel fees illegally collected.
2. They are perpetually enjoined from extracting any sand or gravel from plaintiff's
property which is two-fifths northern portion of the disputed area.
 None of the parties seemed to be satisfied so they all sought a reconsideration of the
same.
 Lower Court’s Decision: (MR)
1. Denies the MR filed by plaintiff and intervenor Calalang;
2. Dismisses the complaint with respect to City of Manila; 3. The northern 2/5
portion of the area belongs to the plaintiff with right to the immediate possession;
4. Enjoins the defendants and intervenor Bureau of Mines to vacate the same and
to stop from extracting gravel thereon.
5. Dismisses the case against the Bureau of Public Works and its agents and
employees with regard to the claim for money.
 Hence, this appeal.

ISSUE
WON when a river, leaving its old bed, changes its original course and opens a new one
through private property, would the new riverbanks be considered as public ownership?
RULING

 SC set aside the decision and orders appealed from, and entered another judgment
to the effect that the City of Manila and the Director of Public Works, and his agent
and employees, are absolved of liability from extracting materials from subject
property (of public domain);
 Portion within the strip of land question declared not part of public domain and
confirmed as part of Hilario’s private property.
 No Costs.

1. Old Civil Code and Law of Waters of 1866 controlling law


Since the change in the course of the River took place in 1937, long before the present
Civil Code took effect, the question should be determined in accordance with the provisions
of the old Civil Code and those of the Law of Waters of 3 August 1866.
2. All riverbanks, as part of the riverbeds, are of public ownership. Under the old
Civil Law and the Law of Waters, all riverbanks are of public ownership,
Including those formed when a river leaves its old bed and opens a new course through a
private estate. Article 339 of the old Civil Code is very clear. Without any qualifications, it
provides that “that devoted to public use, such as roads, canals, rivers, torrents, ports and
25 | P a g e
MABELLE J. ARELLANO PROPERTY
L02 Saturday, 8:00 AM-12:00 PM

bridges constructed by the State, riverbanks, shores, roadsteads, and that of a similar
character” are property of public ownership. Further, the riverbank is part of the riverbed.
Article 73 of the Law of Waters which provides that the phrase “banks of a river” is
understood those lateral strips of zones of its beds which are washed by the stream only
during such high floods as do not cause inundations. The use of the words “of its bed [de
sus alveos] “ clearly indicates the intent of the law to consider the banks for all legal
purposes, as part of the riverbed. Thus, the banks of the River are part of its bed. Since
undeniably all beds of river are of public ownership, it follows that the banks, which form
part of them, are also of public ownership.
3. New bed, when river changes course, is of public ownership; Means to recover
Article 372 of the old Civil Code which provides that “whenever a navigable or floatable
river changes its course from natural causes and opens a new bed through a private estate,
the new bed shall be of public ownership, but the owner of the estate shall recover it in the
event that the waters leave it dry again either naturally or as the result of any work legally
authorized for this purpose.” Banks are not mentioned in the provision, as the nature of
banks follows that of the bed and the running water of the river.
4. A river is a compound concept consisting of running waters, bed, and banks
A river is a compound concept consisting of three elements;
(1) the running waters,
(2) the bed and
(3) the banks.
All these constitute the river. American authorities are in accord with this view, as that
“‘River’ consists of water, bed and banks”; and that “A ‘river’ consists of water, a bed and
banks, these several parts constituting the river, the whole river. It is a compound idea; it
cannot exist without all its parts. Evaporate the water, and you have a dry hollow. If you
could sink the bed, instead of a river you would have a fathomless gulf. Remove the banks,
and you have, a boundless flood”
5. River is of public ownership, elements follow same nature of ownership; Law
explicit
Since a river is but one compound concept, it should have only one nature, i.e., it should
either be totally public or completely private. Since rivers are of public ownership, it is
implicit that all the three component elements be of the same nature also. Still, the law
expressly makes all three elements public. Thus, riverbanks and beds are public under
Articles 339 and 407, respectively, of the Code, while the flowing waters are declared so
under Articles 33, par. 2 of the Law of Waters of 1866.
6. Legal definition applies with the legal order, distinction due to physical order
cannot prevail
The conclusion made by the lower court that only the northern 2/5 of the disputed area
remained as plaintiff’s private property is predicated from the findings that the portion where
rice and corn were found in the ocular inspection of 15 June 1951, was on the northern 2/5
of the disputed area; that this cannot be a part of the bed because of the existence of
vegetation which could not have grown underwater, and that this portion is manmade. This
is bereft of evidence, as the unexcavated portion of the land is the southwestern ¼. Further,
American cases cannot be applied as these do not involve a similar statutory provision,
unlike in the Law of Waters, which defined “beds” and “banks” and considered the latter as
part of the former. That plants can and do grow on the banks which otherwise could not
have grown on the bed which is constantly subjected to the flow of the waters proves the
distinction between “beds” and “banks” in the physical order. However, in dealing with the
legal order, legal definitions prevail.

26 | P a g e
MABELLE J. ARELLANO PROPERTY
L02 Saturday, 8:00 AM-12:00 PM

7. Limits of banks of rivers


Article 73 of the Law of Waters which defines the limits of banks of rivers “By the phrase
‘banks of a river’ is understood those lateral strips or zones of its bed which are washed by
the stream only during such high floods as do not cause inundations. The farthest extremity
of the bank on the west side would, therefore, be that lateral line or strip which is reached
by the waters during those high floods that do not cause inundations. In other words, the
extent reached by the waters when the River is at high tide.
8. Banks of river different in topography
There is a difference between the topography of the two sides immediately adjoining the
River. The line indicated as “primary bank,” which is on the east, is about 3 meters high
and has a steep grade right at the edge where it drops almost vertically to the watercourse
level. The opposite side, on the other hand, has no such steep acclivity. The bank near the
water edge, is about 30 to 50 cms. high only, and gradually slopes up to a height of about
2 to 2-1/2 meters along the line indicated as “secondary bank”, which is quite far from the
waterline. Considering the peculiar characteristics of the two sides banking the river, the
rise in the waterlevel would not have the same effect on the two sides. Thus, on the east,
the water would rise vertically, until the top of the “primary bank” is reached, but on the
west, there would be a low angled inclined rise, the water covering more ground until the
“secondary bank” line is reached. In other words, while the water expansion on the east is
vertical, that on the west is more or less lateral, or horizontal.
9. Ordinary and extraordinary flood
There are two types of floods in the area during the rainy season. One is the so-called
“ordinary” flood, when the river is swollen but the flowing water is kept within the confines
of the “primary” and “secondary” banks. This occurs annually, about three to four times
during the period. Then there is the “extraordinary” flood, when the waters overflow beyond
the said banks, and even inundate the surrounding areas. However, this flood does not
happen regularly. From 1947 to 1955, there were only three such floods.
10. Movement of the river not due to excavation and extraction of materials
The excavations and extractions of materials, even from the American period, have been
made only on the strip of land west of the River. Under the “following-the nature-of-things”
argument advanced by plaintiff, the River should have moved westward, where the level
of the ground had been lowered. But the movement has been in the opposite direction
instead. Therefore, it cannot be attributed to defendants’ operations. Moreover, Hilario’s
own evidence indicates that the movement eastward was all due to natural causes. The
movement eastward of the channel by as much as 31 meters, from 1950 to 1953, was due
to two typhoons which caused the erosion of the east bank and the depositing of materials
on the west side which increased its level from as much as .93 to 2 meters.
11. Plaintiff not denied of property without just compensation
The Court does not declare that the entire channel, i.e., all that space between the
“secondary bank” line and the “primary bank” line, has permanently become part of the
riverbed. What is held is that at the time the defendants made their extractions, the
excavations were within the confines of the riverbanks then. All that space to the west of
said receding line” would still be part of plaintiff’s property and also whatever portion
adjoining the river is, at present, no longer reached by the non-inundating ordinary floods.
Further, it is not correct to say that plaintiff would be deprived of his property without any
compensation at all. Under Article 370 of the old Civil Code, the abandoned bed of the old
river belongs to the riparian owners either fully or in part with the other riparian owners.
And had the change occurred under the Civil Code of the Philippines, plaintiff would even
be entitled to all of the old bed in proportion to the area he has lost.

27 | P a g e
MABELLE J. ARELLANO PROPERTY
L02 Saturday, 8:00 AM-12:00 PM

MANILA INTERNATIONAL AIRPORT AUTHORITY, petitioner,


vs.
COURT OF APPEALS, CITY OF PARAÑAQUE, CITY MAYOR OF PARAÑAQUE,
SANGGUNIANG PANGLUNGSOD NG PARAÑAQUE, CITY ASSESSOR OF
PARAÑAQUE, and CITY TREASURER OF PARAÑAQUE, respondents
G.R. No. 155650, July 20, 2006

DOCTRINE: The term “ports” includes seaports and airports. The MIAA Airport Lands and
Buildings constitute a “port” constructed by the State. Under Article 420 of the Civil Code,
the MIAA Airport Lands and Buildings are properties of public dominion and thus owned by
the State or the Republic of the Philippines.

FACTS

Manila International Airport Authority (MIAA) operates the Ninoy Aquino International
Airport Complex in Parañaque City. As operator of the international airport, MIAA
administers the land, improvements and equipment within the NAIA Complex.

The MIAA Charter transferred to MIAA approximately 600 hectares of land including the
runways and buildings (“Airport Lands and Buildings”) then under the Bureau of Air
Transportation. The MIAA Charter further provides that no portion of the land transferred
to MIAA shall be disposed of through sale or any other mode unless specifically approved
by the President of the Philippines.

OGCC (Office of the Government Corporate Counsel) issued Opinion No. 061, in which it
said that the Local Government Code of 1991 withdrew the exemption for real estate tax
granted to MIAA under Section 21 of the MIAA charter.

Therefore, MIAA was held to be delinquent in paying its taxes. The City of Parañaque
Levied upon the properties of MIAA, and posted invitations for public biddings of MIAA’s
properties. MIAA filed with CA an action for prohibition / injunction. The City of Parañaque
averred that Section 193 of the Local Government code expressly withdrew tax exemptions
from government owned and controlled corporations (GOCCs).

CA dismissed the petition for filing beyond the 60 day reglementary period

ISSUE

Whether properties of the MIAA are subject to real estate taxes.

RULING

No. In the first place, MIAA is not a GOCC, it is an instrumentality of the government. MIAA
is a government instrumentality vested with corporate powers to perform efficiently its
governmental functions. MIAA is like any other government instrumentality, the only
difference is that MIAA is vested with corporate powers. As operator of the international
airport, MIAA administers the land, improvements and equipment within the NAIA
Complex. The MIAA Charter transferred to MIAA approximately 600 hectares of land,
including the runways and buildings (“Airport Lands and Buildings”) then under the Bureau
of Air Transportation. The MIAA Charter further provides that no portion of the land
transferred to MIAA shall be disposed of through sale or any other mode unless specifically
approved by the President of the Philippines.

Furthermore, Airport Lands and Buildings of MIAA are property of public dominion and
therefore owned by the State or the Republic of the Philippines. Article 419 of the Civil
Code provides, The Airport Lands and Buildings of MIAA are property of public dominion
and therefore owned by the State or the Republic of the Philippines.

28 | P a g e
MABELLE J. ARELLANO PROPERTY
L02 Saturday, 8:00 AM-12:00 PM

The Civil Code provides:

ARTICLE 419. Property is either of public dominion or of private ownership.

ARTICLE 420. The following things are property of public dominion:


(1) Those intended for public use, such as roads, canals, rivers, torrents,
ports and bridges constructed by the State, banks, shores, roadsteads, and
others of similar character;
(2) Those which belong to the State, without being for public use, and are
intended for some public service or for the development of the national
wealth.
ARTICLE 421. All other property of the State, which is not of the character stated
in the preceding article, is patrimonial property.

ARTICLE 422. Property of public dominion, when no longer intended for public use
or for public service, shall form part of the patrimonial property of the State.

No one can dispute that properties of public dominion mentioned in Article 420 of the Civil
Code, like “roads, canals, rivers, torrents, ports and bridges constructed by the State,” are
owned by the State. The term “ports” includes seaports and airports. The MIAA Airport
Lands and Buildings constitute a “port” constructed by the State. Under Article 420 of the
Civil Code, the MIAA Airport Lands and Buildings are properties of public dominion and
thus owned by the State or the Republic of the Philippines.

The Airport Lands and Buildings are devoted to public use because they are used by the
public for international and domestic travel and transportation. The fact that the MIAA
collects terminal fees and other charges from the public does not remove the character of
the Airport Lands and Buildings as properties for public use. The operation by the
government of a tollway does not change the character of the road as one for public use.
Someone must pay for the maintenance of the road, either the public indirectly through the
taxes they pay the government, or only those among the public who actually use the road
through the toll fees they pay upon using the road. The tollway system is even a more
efficient and equitable manner of taxing the public for the maintenance of public roads.

The charging of fees to the public does not determine the character of the property whether
it is of public dominion or not. Article 420 of the Civil Code defines property of public
dominion as one “intended for public use.” Even if the government collects toll fees, the
road is still “intended for public use” if anyone can use the road under the same terms and
conditions as the rest of the public. The charging of fees, the limitation on the kind of
vehicles that can use the road, the speed restrictions and other conditions for the use of
the road do not affect the public character of the road.

The terminal fees MIAA charges to passengers, as well as the landing fees MIAA charges
to airlines, constitute the bulk of the income that maintains the operations of MIAA. The
collection of such fees does not change the character of MIAA as an airport for public use.
Such fees are often termed user’s tax. This means taxing those among the public who
actually use a public facility instead of taxing all the public including those who never use
the particular public facility. A user’s tax is more equitable — a principle of taxation
mandated in the 1987 Constitution.

The Airport Lands and Buildings of MIAA, which its Charter calls the “principal airport of
the Philippines for both international and domestic air traffic,” are properties of public
dominion because they are intended for public use. As properties of public dominion, they
indisputably belong to the State or the Republic of the Philippines.

Being a property of public dominion, the properties of MIAA are beyond the commerce of
man.

29 | P a g e
MABELLE J. ARELLANO PROPERTY
L02 Saturday, 8:00 AM-12:00 PM

REPUBLIC OF THE PHILIPPINES, represented by the PHILIPPINE RECLAMATION


AUTHORITY (PRA), Petitioner,
vs.
CITY OF PARANAQUE, Respondent
G.R. No. 191109, July 18, 2012

FACTS

Philippine Reclamation Authority (PEA before) performs all the powers and functions of the
PEA relating to reclamation activities. PRA reclaimed several portions of the foreshore and
offshore areas of Manila Bay, including those located in Parañaque City, and was issued
Original Certificates of Title over the reclaimed lands.

On February 19, 2003, then Parañaque City Treasurer Carabeo issued Warrants of Levy
on PRA’s reclaimed properties (Central Business Park and Barangay San Dionisio) located
in Parañaque City based on the assessment for delinquent real property taxes made by
then Parañaque City Assessor Soledad Medina Cue for tax years 2001 and 2002.

On March 26, 2003, PRA filed a petition for prohibition with prayer for TRO and/or writ of
preliminary injunction against Carabeo before the RTC. RTC denied the petition for TRO
as well as the issuance of the writ of preliminary injunction.

Due to the failure of both parties to arrive at a compromise agreement, PRA filed a Motion
for Leave to File and Admit Attached Supplemental Petition which sought to declare as null
and void the assessment for real property taxes, the levy based on the said assessment,
the public auction sale conducted on April 7, 2003, and the Certificates of Sale issued
pursuant to the auction sale. RTC dismissed PRA’s petition. It ruled that PRA was not
exempt from payment of real property taxes.The RTC reasoned out that it was a GOCC
under Section 3 of P.D. No. 1084.

Thus, PRA claims that based on Section 133(o) of the LGC, local governments cannot tax
the national government which delegate to local governments the power to tax. It explains
that reclaimed lands are part of the public domain, owned by the State, thus, exempt from
the payment of real estate taxes. Hence, the assessment of real property taxes made on
said lands, as well as the levy thereon, and the public sale thereof on April 7, 2003,
including the issuance of the certificates of sale in favor of the respondent Parañaque City,
are invalid and of no force and effect.

On the other hand, the City of Parañaque argues that PRA since its creation consistently
represented itself to be a GOCC. Section 193 of the LGC of 1991 has withdrawn tax
exemption privileges granted to or presently enjoyed by all persons, whether natural or
juridical, including GOCCs. Hence, since PRA is a GOCC, it is not exempt from the
payment of real property tax.

ISSUE

Whether or not the reclaimed lands are part of the public domain and, hence, exempt from
real property tax.

RULING

Yes. Reclaimed lands such as the subject lands in issue are reserved lands for public use.
They are properties of public dominion. The ownership of such lands remains with the State
unless they are withdrawn by law or presidential proclamation from public use.

Under Section 2, Article XII of the 1987 Constitution, the foreshore and submerged areas
of Manila Bay are part of the "lands of the public domain, waters x x x and other natural
resources" and consequently "owned by the State." As such, foreshore and submerged
areas "shall not be alienated," unless they are classified as "agricultural lands" of the public
30 | P a g e
MABELLE J. ARELLANO PROPERTY
L02 Saturday, 8:00 AM-12:00 PM

domain. The mere reclamation of these areas by PEA does not convert these inalienable
natural resources of the State into alienable or disposable lands of the public domain. There
must be a law or presidential proclamation officially classifying these reclaimed lands as
alienable or disposable and open to disposition or concession. Moreover, these reclaimed
lands cannot be classified as alienable or disposable if the law has reserved them for some
public or quasi-public use.

The Court agrees with PRA that the subject reclaimed lands are still part of the public
domain, owned by the State and, therefore, exempt from payment of real estate taxes.

31 | P a g e
MABELLE J. ARELLANO PROPERTY
L02 Saturday, 8:00 AM-12:00 PM

VICENTE YU CHANG AND SOLEDAD YU CHANG v. REPUBLIC OF THE PHILIPPINES


G.R. No. 171726, February 23, 2011

FACTS

L. Yu Chang entered into a barter with the Municipality of Pili, Camarines Sur whereby the
former exchanged his 400 square meter land located in the barrio of San Roque for another
400 square meter land owned by the latter located in the barrio of San Juan. When L. Yu
Chang died, his wife and children inherited the land. A Deed of Transfer and Renunciation
of their rights over the land was executed by the heirs of L. Yu Chang in favour of Vicente
Yu Chang and Soledad Yu Chang. They filed a petition for registration for the land. They
declared that they are the co-owners of the subject land; that they and their predecessors-
in-interest have been in actual, physical, material, exclusive, open, occupation and
possession of the above described parcels of land for more than 100 years; and that they
have continuously, peacefully, and adversely possessed the property in the concept of
owners. The Republic of the Philippines opposed the petition claiming that the land forms
part of the public domain and not subject to private appropriation. The RTC granted the
petition but the CA reversed it, saying that there must be a positive act from the government
declassifying the land as forest land before it could be deemed alienable or disposable land
for agricultural or other purposes. Petitioners insist that the subject land could no longer be
considered and classified as forest land since there are buildings, residential houses, and
government structures existing in the land.

ISSUE

Whether the application for land registration should be granted.

RULING

No. In order that petitioners’ application for registration of title may be granted, they must
first establish the following: (1) that the subject land forms part of the disposable and
alienable lands of the public domain and (2) that they have been in open, continuous,
exclusive and notorious possession and occupation of the same under a bona fide claim
of ownership, since June 12, 1945, or earlier. Applicants must overcome the presumption
that the land they are applying for is part of the public domain and that they have an interest
therein sufficient to warrant registration in their names arising from an imperfect title.

The classification of land is descriptive of its legal nature or status and does not have to be
descriptive of what the land actually looks like. Unless and until the land classified as forest
land is released in an official proclamation to that effect so that it may form part of the
disposable agricultural lands of the public domain, the rules on confirmation of imperfect
title do not apply.

The fact that the area within which the subject parcels of land are located is being used for
residential and commercial purposes does not serve to convert the subject parcels of land
into agricultural land. It is fundamental that before any land may be declassified from the
forest group and converted into alienable or disposable land for agricultural or other
purposes, there must be a positive act from the government. The Government must first
declare the forest land to be alienable and disposable agricultural land before the year of
entry, cultivation and exclusive and adverse possession can be counted for purposes of an
imperfect title.

To reiterate, it is well settled that possession of forest land, prior to its classification as
alienable and disposable land, is ineffective since such possession may not be considered
as possession in the concept of owner.33 The adverse possession which can be the basis
of a grant of title in confirmation of imperfect title cases cannot commence until after forest
land has been declared and alienable.
32 | P a g e
MABELLE J. ARELLANO PROPERTY
L02 Saturday, 8:00 AM-12:00 PM

Salvador H. Laurel vs. Ramon Garcia, et al.


G.R. No. 92013, July 25, 1990

FACTS

Petitioners seek to stop the Philippine Government to sell the Roppongi Property, which is
located in Japan. It is one of the properties given by the Japanese Government as
reparations for damage done by the latter to the former during the war.
Petitioner argues that under Philippine Law, the subject property is property of public
dominion.

As such, it is outside the commerce of men. Therefore, it cannot be alienated.


Respondents aver that Japanese Law, and not Philippine Law, shall apply to the case
because the property is located in Japan. They posit that the principle of lex situs applies.

ISSUE

Whether or not the subject property cannot be alienated.

RULING

The answer is in the affirmative.

Under Philippine Law, there can be no doubt that it is of public dominion unless it is
convincingly shown that the property has become patrimonial. This, the respondents have
failed to do. As property of public dominion, the Roppongi lot is outside the commerce of
man. It cannot be alienated.

33 | P a g e
MABELLE J. ARELLANO PROPERTY
L02 Saturday, 8:00 AM-12:00 PM

Del Fierro v. Seguiran


G.R. NO. 152141,Aug. 8, 2011

FACTS

Plaintiffs-petitioners were the alleged owners and possessors of a parcel of land identified
as Lot Nos. 1625 and 1626. That Lodelfo and Narciso Marcial unlawfully entered the land
occupied by plaintiffs. Plaintiffs sued them for forcible entry before the Municipal Court of
Palauig. The court ruled in favor of plaintiffs, which decision was affirmed on appeal by the
CFI.

Meanwhile, Marcial had mortgaged the lots to the Rural Bank of San Marcelino, Inc., which
foreclosed the real estate mortgage on December 26, 1972, and consolidated ownership
over the lots on April 22, 1982. On October 28,1981, defendant Rene S. Seguiran
purchased from Lodelfo Marcial (deceased) the subject lots. On November 9,1981,
defendant purchased the subject lots again from the Rural Bank of San Marcelino, Inc.
Moreover, plaintiffs alleged that Lodelfo Marcial, predecessor-in-interest of defendant, had
no legal right to convey the said lots to plaintiffs, since he was merely a deforciant in the
said lots. Further, defendant, with evident bad faith, fraudulently applied with the Bureau of
Lands for a free patent over the said lots, alleging that he was the actual possessor thereof,
which constitutes a false statement, since the plaintiffs were the ones in actual possession.
Despite knowing that the said lots were the subject of legal controversy before the CFI of
Iba,Zambales, Branch II, defendant fraudulently secured a certification from the Court of
Olongapo to prove that the said parcels of land were not subject of any court action. As a
consequence of the foregoing illegal and fraudulent acts, defendant was able to secure
OCT Nos. P-7013 and P-7014 for Lot Nos. 1625 and 1626,respectively.

On May 20, 1988, defendant filed his Answer, claiming that when he bought the land in
dispute on October, 28, 1981, Lodelfo Marcial was no longer its owner, but the Rural Bank
of San Marcelino, Inc., since Marcial failed to redeem the land within the one-year period
of redemption. His only purpose for buying the land from the mortgagor, Lodelfo Marcial in
November 1981 was for the peaceful turn-over of the property to him by Marcial. Defendant
denied any fraud, illegality or bad faith in securing OCT Nos. P-7013 and P-7014. He
asserted that when he secured a certification from the RTC on June 6, 1983, there was in
truth no pending case involving the subject properties in any court in Zambales; hence, no
bad faith could be attributed to him.
On April 23, 1998, the trial court rendered judgment in favor of defendant, respondent
dismissed for insufficiency of evidence as to the identity of the properties sought to be
recovered. The complaint-in-intervention dated February 24, 1987 is dismissed for
prematurity and insufficiency of evidence

On October 2, 2001, the Court of Appeals upheld the decision of the trial court.

ISSUE

Whether petitioners are entitled to reconveyance of Lot Nos. 1625 and 1626.

RULING

No. The requisites of reconveyance are provided for in Article 434 of the Civil Code, thus:

Art. 434. In an action to recover, the property must be identified, and the plaintiff must rely
on the strength of his title and not on the weakness of the defendants claim.

Article 434 of the Civil Code provides that to successfully maintain an action to recover the
ownership of a real property, the person who claims a better right to it must prove two (2)
things: first, the identity of the land claimed; and second, his title thereto.

In regard to the first requisite, in an accion reinvindicatoria, the person who claims that he
has a better right to the property must first fix the identity of the land he is claiming by
describing the location, area and boundaries thereof. Anent the second requisite, i.e., the
34 | P a g e
MABELLE J. ARELLANO PROPERTY
L02 Saturday, 8:00 AM-12:00 PM

claimant's title over the disputed area, the rule is that a party can claim a right of ownership
only over the parcel of land that was the object of the deed.
In this case, petitioners failed to prove the identity of the parcels of land sought to be
recovered and their title thereto.
In fine, petitioners failed to prove the identity of the properties over which they claimed
ownership and sought to be reconvened to them, and they also failed to prove their title
over Lot Nos. 1625 and 1626; WHEREFORE, the petition is DENIED.

35 | P a g e
MABELLE J. ARELLANO PROPERTY
L02 Saturday, 8:00 AM-12:00 PM

Spouses Manuel and Florentina Del Rosario, petitioners,


vs.
Gerry Roxas Foundation, Inc., respondent
G.R. No. 170575, June 8, 2011

FACTS

Manuel del Rosario appears to be the registered owner of Lot 3-A of Psd-301974 located
in Roxas City which is described in and covered by a Transfer Certificate of Title (TCT).
Sometime in 1991, the Gerry Roxas Foundation, Inc., (GRFI) as a legitimate foundation,
took possession and occupancy of said land by virtue of a memorandum of agreement it
entered with Roxas City. Its possession and occupancy of said land is in the character of
being a lessee thereof.

In February and March 2003, the Spouses Manuel and Florentina del Rosario (Spouses)
served notices upon the GRFI to vacate the premises of said land. GRFI, however, did not
heed such notices because it still has the legal right to continue its possession and
occupancy of said land.

On July 2003, the spouses filed a Complaint for Unlawful Detainer against GRFI before the
Municipal Trial Court in Cities.

Sometime in 1991, without the consent and authority of the plaintiffs, defendant took full
control and possession of the subject property, developed the same and used it for
commercial purposes. Plaintiffs have allowed the defendant for several years, to make use
of the land without any contractual or legal basis. Hence, defendant’s possession of the
subject property is only by tolerance. But plaintiffs’ patience has come to its limits. Hence,
sometime in the last quarter of 2002, plaintiffs made several demands upon said defendant
to settle and/or pay rentals for the use of the property.

Notwithstanding receipt of the demand letters, defendant failed and refused, as it continues
to fail and refuse to pay reasonable monthly rentals for the use and occupancy of the land,
and to vacate the subject premises despite the lapse of the fifteen-day period specified in
the said demand letters.
Consequently, defendant is unlawfully withholding possession of the subject property from
the plaintiffs, who are the owners thereof.

ISSUE

Whether or not the allegations in the Complaint establish a cause of action for forcible entry
and not an unlawful detainer.

RULING

Yes. In forcible entry, one is deprived of physical possession of any land or building by
means of force, intimidation, threat, strategy, or stealth. Where the defendant’s possession
of the property is illegal ab initio, the summary action for forcible entry (detentacion) is the
remedy to recover possession. In their Complaint, the spouses maintained that the GRFI
took possession and control of the subject property without any contractual or legal basis.
Assuming that these allegations are true, it hence follows that GRFI’s possession was
illegal from the very beginning. Therefore, the foundation of the spouses Complaint is one
for forcible entry – that is, the forcible exclusion of the original possessor by a person who
has entered without right. Thus, there can be no tolerance as the spouses alleged that
GRFI’s possession was illegal at the inception. Corollarily, since the deprivation of physical
possession, as alleged in the spouses’ Complaint was attended by strategy and force, this
Court finds that the proper remedy for the spouses was to file a Complaint for Forcible
Entry and not the instant suit for unlawful detainer.

36 | P a g e
MABELLE J. ARELLANO PROPERTY
L02 Saturday, 8:00 AM-12:00 PM

Fiorello R. Jose, petitioner,


vs.
Roberto Alfuerto, respondent
G.R. No. 169380, November 26, 2012

FACTS
The dispute involves a parcel of land registered in the name of Rodolfo Chua Sing under
Transfer Certificate of Title No. 52594, with an area of 1919 square meters, located in
Barangay San Dionisio, Parañaque City. Chua Sing purchased the land in 1991. On April
1, 1999, Chua Sing leased the property to the petitioner. Their contract of lease was
neither notarized nor registered with the Parañaque City Registry of Deeds.

On April 28, 1999, soon after Chua Sing and the petitioner signed the lease contract, the
petitioner demanded in writing that the respondents vacate the property within 30 days and
that they pay a monthly rental of P1,000.00 until they fully vacate the property.

The respondents refused to vacate and to pay rent. On October 20, 1999, the petitioner
filed an ejectment case against the respondents before Branch 77 of the Parañaque City
MeTC. In this complaint, no mention was made of any proceedings before the barangay.
Jose then brought the dispute before the barangay for conciliation. The barangay issued a
Certification to File Action on March 1, 2000. Jose was then able to file an amended
complaint, incorporating the proceedings before the barangay before the summons and
copies of the complaint were served upon the named defendants.

In the Amended Complaint dated March 17, 2000, the petitioner claimed that as lessee of
the subject property, he had the right to eject the respondents who unlawfully occupy the
land.

The respondents likewise pointed out that they have been in possession of the land long
before Chua Sing acquired the property in 1991 and that the lease contract between the
petitioner and Chua Sing does not affect their right to possess the land. The respondents
also presented a Deed of Assignment. They argued that the MeTC had no jurisdiction over
the case as the issue deals with ownership of the land, and sought the dismissal of the
complaint for lack of cause of action and for lack of jurisdiction.
On January 27, 2003, the MeTC held that the respondents had no right to possess the land
and that their occupation was merely by the owner's tolerance. The MeTC held that it is
not divested of jurisdiction over the case because of the respondents' assertion of
ownership of the property

On appeal before the RTC, the respondents raised the issue, among others, that no legal
basis exists for the petitioner's claim that their occupation was by tolerance, "where the
possession of the defendants was illegal at the inception as alleged in the complaint there
can be no tolerance."

The RTC affirmed the MeTC decision of January 27, 2003.

On March 14, 2005, the Court of Appeals reversed the RTC and MeTC decisions. It ruled
that the respondents' possession of the land was not by the petitioner or his lessor's
tolerance.

The petitioner filed a motion for reconsideration.

ISSUE

Whether or not an action for unlawful detainer is the proper remedy.

RULING

Unlawful detainer is a summary action for the recovery of possession of real


property. This action may be filed by a lessor, vendor, vendee, or other person against

37 | P a g e
MABELLE J. ARELLANO PROPERTY
L02 Saturday, 8:00 AM-12:00 PM

whom the possession of any land or building is unlawfully withheld after the expiration or
termination of the right to hold possession by virtue of any contract, express or implied. In
unlawful detainer, the possession of the defendant was originally legal, as his possession
was permitted by the plaintiff on account of an express or implied contract between
them. However, the defendant's possession became illegal when the plaintiff demanded
that the defendant vacate the subject property due to the expiration or termination of the
right to possess under the contract, and the defendant refused to heed such demand. A
case for unlawful detainer must be instituted one year from the unlawful withholding of
possession.

The allegations in the complaint determine both the nature of the action and the jurisdiction
of the court. The complaint must specifically allege the facts constituting unlawful
detainer. In the absence of these allegations of facts, an action for unlawful detainer is not
the proper remedy and the municipal trial court or the MeTC does not have jurisdiction over
the case.

The petitioner's allegations in the amended complaint run counter to the requirements for
unlawful detainer. In an unlawful detainer action, the possession of the defendant was
originally legal and his possession was permitted by the owner through an express or
implied contract.

The Court has consistently adopted this position: tolerance or permission must have
been present at the beginning of possession; if the possession was unlawful from
the start, an action for unlawful detainer would not be the proper remedy and should
be dismissed.

The Judges DENY the petition, and AFFIRM the Court of Appeals' decision.

38 | P a g e
MABELLE J. ARELLANO PROPERTY
L02 Saturday, 8:00 AM-12:00 PM

Bienvenido Barrientos, petitioner,


vs.
Mario Rapal, respondent
G.R. No. 169594, July 20, 2011

FACTS

On April 15, 1998, respondent Mario Rapal acquired a parcel of land at Quezon city via a
notarized Deed of Transfer of Possessory Right. The parcel of land was said to be a portion
of the estate of the late Don Mariano San Pedro y Esteban. Thereafter, respondent
constructed a semi-concrete house on the lot and took actual possession of the property
by himself and through his caretaker, Benjamin Tamayo.

In 1993, Petitioner Bienvinido Barrientos and his family were allowed to stay in the subject
property as caretakers with a condition that they shall vacate the premises when the
respond would need it. Upon demand to vacate, petitioner refused to leave the subject
property. Thus, respondent filed a complaint of Unlawful Detainer against the petitioner.

On February 21, 2000, the trial court rendered a decision in favor of the respondent. It
ordered petitioner to vacate the premises and to pay respondent a compensation for the
use of the structure.

On appeal, (RTC) reversed the Decision of the MeTC and resolved in favor of petitioner,
reasoning that respondent has not shown any prior lawful possession of the property in
question.

On April 29, 2005, the CA rendered the assailed Decision reversing the decision of the
RTC and reinstating the decision of the MeTC. The CA found that both parties presented
weak evidence of ownership. In determining who between the parties was first in
possession, the CA concluded that respondent was, indeed, first in possession of the lot.

Hence, petitioner elevated the case to the SC.

ISSUES

WHETHER THE ISSUE OF OWNERSHIP CAN BE INITIALLY RESOLVED FOR THE


PURPOSE OF DETERMINING THE ISSUE OF POSSESSION.

WHETHER THE RESPONDENT'S DOCUMENT PURPORTING TO BE A TRANSFER OF


POSSESSORY RIGHT CAN PREVAIL OVER THE PETITIONER'S CLAIM OF
OWNERSHIP AND THE LATTER'S ACTUAL POSSESSORY RIGHT OVER THE
PROPERTY.

RULING

The petition is without merit.

Ejectment cases forcible entry and unlawful detainer are summary proceedings designed
to provide expeditious means to protect actual possession or the right to possession of the
property involved. The only question that the courts resolve in ejectment proceedings is:
who is entitled to the physical possession of the premises, that is, to the possession de
facto and not to the possession de jure. It does not even matter if a party's title to the
property is questionable. In an unlawful detainer case, the sole issue for resolution is
physical or material possession of the property involved, independent of any claim of
ownership by any of the parties. Where the issue of ownership is raised by any of the
parties, the courts may pass upon the same in order to determine who has the right to
possess the property. The adjudication is, however, merely provisional and would not bar
or prejudice an action between the same parties involving title to the property.

In the case at bar, both petitioner and respondent were claiming ownership over the subject
property. Hence, the CA correctly touched upon the issue of ownership only to determine
39 | P a g e
MABELLE J. ARELLANO PROPERTY
L02 Saturday, 8:00 AM-12:00 PM

who between the parties has the right to possess the subject property. True, as found by
the CA, both petitioner and respondent presented weak evidence of ownership.

Thus, based on the evidence presented by the respondent, it can be deduced that
petitioner's occupation of the subject lot was by mere tolerance only. Petitioner was initially
permitted by respondent to occupy the lot as a caretaker. Petitioner even admitted this fact
in his Beneficiary Evaluation and Qualification Form. Moreover, all other supporting
evidence, such as the Census Survey Certificate and construction material receipts, bolster
the fact that respondent was in prior possession of the property before petitioner entered
the same by mere tolerance of the respondent.
Perusing respondent's complaint, respondent clearly makes out a case for unlawful
detainer, since petitioner's occupation of the subject property was by mere tolerance. A
person who occupies the land of another at the latter's tolerance or permission, without any
contract between them, is necessarily bound by an implied promise that he will vacate the
same upon demand, failing which a summary action for ejectment is the proper remedy
against them

It should be stressed that unlawful detainer and forcible entry suits, under Rule 70 of the
Rules of Court, are designed to summarily restore physical possession of a piece of land
or building to one who has been illegally or forcibly deprived thereof, without prejudice to
the settlement of the parties' opposing claims of juridical possession in appropriate
proceedings. These actions are intended to avoid disruption of public order by those who
would take the law in their hands purportedly to enforce their claimed right of possession.
In these cases, the issue is pure physical or de facto possession, and pronouncements
made on questions of ownership are provisional in nature. The provisional determination
of ownership in the ejectment case cannot be clothed with finality.

40 | P a g e
MABELLE J. ARELLANO PROPERTY
L02 Saturday, 8:00 AM-12:00 PM

GERMAN MANAGEMENT & SERVICES, INC., petitioner,


vs.
HON. COURT OF APPEALS and ERNESTO VILLEZA, respondents
G.R. No. 76217 September 14, 1989
GERMAN MANAGEMENT & SERVICES, INC., petitioner,
vs.
HON. COURT OF APPEALS and ORLANDO GERNALE, respondents
G.R. No. L-76216 September 14, 1989

FACTS

In February 1982, the spouses Manuel and Cynthia Jose contracted with German
Management and Services, Inc. for the latter to develop their landholdings into a
residential subdivision. The spouses also executed a special power of attorney to that
effect.

German Management started the project in February 1983, however, German


Management discovered that the land was being possessed by Ernest0 Villeza et al
who were the farmers tilling the said land at that time. German Management spoke
with Villeza et al but the farmers refused to vacate the land as the farmers claimed that
they have been occupying the land for twelve years.

Nevertheless, German Management went on to develop the property and demolished


the properties of the farmers without acquiring a court order. In turn, Villeza et al filed
a case of forcible entry against German Management. In its defense, German
Management invoked the Doctrine of Self-help which provides that:

The owner or lawful possessor of a thing has the right to exclude any person from the
enjoyment and disposal thereof.For this purpose,he may use such force as may be
reasonably necessary to repel or prevent an actual or threatened unlawful physical
invasion or usurpation of his property(Art.429,CC)

ISSUE

Whether or not the doctrine of self-help is applicable in this case.

RULING

No. The Doctrine of Self-help is not applicable because at the time when German
Management excluded the farmers, there’s no longer an actual or threatened unlawful
physical invasion or usurpation. That actual or threatened unlawful physical invasion
by the farmers have already lapsed 12 years ago when they began occupying the said
land. In fact, they were already peaceably farming the land.

What should have been the remedy by German Management?


German Management should have filed either accion publiciana or accion
reivindicatoria to lawfully eject the farmers.

But the farmers are not the real owners and in fact, the spouses Jose have a lawful title
over the land?
Regardless of the actual condition of the title to the property, the party in peaceable
quiet possession shall not be turned out by a strong hand, violence or terror. Further,
there is now a presumption of ownership in favor of the farmers since they are the ones
occupying the said property. They can only be ejected either by accion
publicianaor accion reivindicatoria through which the spouses Jose’s better right may
be proven.

41 | P a g e
MABELLE J. ARELLANO PROPERTY
L02 Saturday, 8:00 AM-12:00 PM

Republic of the Philippines v. Court of Appeals,


G.R. No. L-43938, April 15, 1988
160 SCRA 228

Cruz, J.

FACTS
An application for registration of a parcel of land was filed on February 11, 1965, by Jose
de la Rosa on his own behalf and on behalf of his three children. The land, situated in
Tuding, Itogon, Benguet Province, was divided into 9 lots. According to the application,
Lots 1-5 were sold to Jose de la Rosa and Lots 6-9 to his children by Mamaya Balbalio and
Jaime Alberto, respectively, in 1964. In support of the application, both Balbalio and Alberto
testified that they had acquired the subject land by virtue of prescription Balbalio claimed
to have received Lots 1-5 from her father shortly after the Liberation. Alberto said he
received Lots 6-9 in 1961 from his mother, Bella Alberto. She was corroborated by Felix
Marcos, who recalled the earlier possession of the land by Alberto's father. Benguet
opposed on the ground that the June Bug mineral claim covering Lots 1-5 was sold to it on
September 22, 1934, by the successors-in-interest of James Kelly, who located the claim
in September 1909 and recorded it on October 14, 1909. From the date of its purchase,
Benguet had been in actual, continuous and exclusive possession of the land in concept
of owner. Atok alleged that a portion of Lots 1-5 and all of Lots 6-9 were covered by the
Emma and Fredia mineral claims located by Harrison and Reynolds on December 25,
1930, and recorded on January 2, 1931, in the office of the mining recorder of Baguio.
These claims were purchased from these locators on November 2, 1931, by Atok, which
has since then been in open, continuous and exclusive possession of the said lots. The
Bureau of Forestry Development also interposed its objection, arguing that the land sought
to be registered was covered by the Central Cordillera Forest Reserve under Proclamation
No. 217 dated February 16, 1929. Moreover, by reason of its nature, it was not subject to
alienation under the Constitutions of 1935 and 1973.
The trial court denied the application, holding that the applicants had failed to prove their
claim of possession and ownership of the land sought to be registered. The applicants
appealed to the respondent court, which reversed the trial court and affirmed the surface
rights of the de la Rosas over the land while at the same time reserving the sub-surface
rights of Benguet and Atok by virtue of their mining claims. Both Benguet and Atok
appealed to the Supreme Court, invoking their superior right of ownership. The Republic
filed its own petition for review and reiterated its argument that neither the private
respondents nor the two mining companies had any valid claim to the land because it was
not alienable and registerable.

ISSUE
Whether or not Benguet and Atok have a better right over the property in question.

RULING

Yes. It is true that the subject property was considered forest land and included in the
Central Cordillera Forest Reserve, but this did not impair the rights already vested in
Benguet and Atok at that time. The perfection of the mining claim converted the property
to mineral land and under the laws then in force removed it from the public domain. By
such act, the locators acquired exclusive rights over the land, against even the government,
without need of any further act such as the purchase of the land or the obtention of a patent
over it. As the land had become the private property of the locators, they had the right to
transfer the same, as they did, to Benguet and Atok.

It is true, as the Court of Appeals observed, that such private property was subject to the
"vicissitudes of ownership," or even to forfeiture by non-user or abandonment or, as the
42 | P a g e
MABELLE J. ARELLANO PROPERTY
L02 Saturday, 8:00 AM-12:00 PM

private respondents aver, by acquisitive prescription. The Court of Appeals justified this by
saying there is "no conflict of interest" between the owners of the surface rights and the
owners of the sub-surface rights. Under the aforesaid ruling, the land isclassified as mineral
underneath and agricultural on the surface, subject to separate claims of title. However,
the rights over the land are indivisible and that the land itself cannot be half agricultural and
half mineral. The classification must be categorical; the land must be either completely
mineral or completely agricultural. In the instant case, as already observed, the land which
was originally classified as forest land ceased to be so and became mineral — and
completely mineral — once the mining claims were perfected. As long as mining operations
were being undertaken thereon, or underneath, it did not cease to be so and become
agricultural, even if only partly so, because it was enclosed with a fence and was cultivated
by those who were unlawfully occupying the surface.

This is an application of the Regalian doctrine. If a person is the owner of agricultural land
in which minerals are discovered, his ownership of such land does not give him the right to
extract or utilize the said minerals without the permission of the State to which such
minerals belong. Benguet and Atok have exclusive rights to the property in question by
virtue of their respective mining claims which they validly acquired before the Constitution
of 1935 prohibited the alienation of all lands of the public domain except agricultural lands,
subject to vested rights existing at the time of its adoption. The land was not and could not
have been transferred to the private respondents by virtue of acquisitive prescription, nor
could its use be shared simultaneously by them and the mining companies for agricultural
and mineral purposes. The decision is set aside and that of the trial court is reinstated.

43 | P a g e
MABELLE J. ARELLANO PROPERTY
L02 Saturday, 8:00 AM-12:00 PM

NATIONAL POWER CORPORATION V. IBRAHIM


G.R. NO. 168732 JUNE 29, 2007
526 SCRA 149 (2007)

AZCUNA, J.:

FACTS
Ibrahim owns a parcel of land located in Lanao del Norte. In 1978, NAPOCOR took
possession of the sub-terrain area of the land and constructed underground tunnels on the
said property. The tunnels were apparently being used by NAPOCOR in siphoning the
water of Lake Lanao and in the operation of NAPOCOR’s Agus projects.

In 1991, Maruhom (one of the co-heirs of Ibrahim) requested Marawi City Water District for
a permit to construct or install a motorized deep well on the parcel of land but it was rejected
on the grounds that the construction would cause danger to lives and property by reason
of the presence of the underground tunnels.

Maruhom demanded NAPOCOR to pay damages and to vacate the sub-terrain portion of
the land.

ISSUE

Whether or not Ibrahim is the rightful owner of the sub-terrain area of the land.

If yes, are they entitled to the payment of just compensation?

HELD

YES. The sub-terrain portion of the property belongs to Ibrahim.

The Supreme Court cited Article 437 of the Civil Code which provides that: The owner of a
parcel of land is the owner of its surface and of everything under it, and he can construct
thereon any works or make any plantations and excavations which he may deem proper,
without detriment to servitudes and subject to special laws and ordinances.

Hence, the ownership of land extends to the surface as well as to the subsoil under it.
Therefore, Ibrahim owns the property as well as the sub-terrain area of the land where the
underground tunnels were constructed.

On the issue of just compensation, the Supreme Court also said that Ibrahim should be
paid a just compensation.

Ibrahim could have dug upon their property and built motorized deep wells but was
prevented from doing so by the authorities because of the construction of the tunnels
underneath the surface of the land.

Ibrahim still had a legal interest in the sub-terrain portion insofar as they could have
excavated the same for the construction of the deep wells. It has been shown that the
underground tunnels have deprived the plaintiffs of the lawful use of the land and
considerably reduced its value.

It was held that: If the government takes property without expropriation and devotes the
property to public use, after many years, the property owner may demand payment of just
compensation in the event restoration of possession is neither convenient nor feasible.
This is in accordance with the principle that persons shall not be deprived of their property
except by competent authority and for public use and always upon payment of just
compensation.

44 | P a g e
MABELLE J. ARELLANO PROPERTY
L02 Saturday, 8:00 AM-12:00 PM

PALERO-TAN V. URDANETA
AM NO. P--‐07--‐2399, JUN. 18, 2008

FACTS

In the, Jr. (respondent), instant administrative complaint, Edna Palero-Tan (complainant),


Court Stenographer III of the Regional Trial Court (RTC), Branch 14, Baybay, Leyte,
charged Ciriaco I. Urdaneta Utility Worker I of the same court, with Conduct Unbecoming
a Court Personnel, for stealing her ring and bracelet.Complainant claimed that it has been
her practice to keep her and her sister’s pieces of jewelry in the locked drawer of her table
at her RTC office because she fears that they might be lost at the boarding house she is
renting. However, on 8 July 2005, she discovered that her ring and bracelet worth fifteen
thousand pesos (P15,000.00) were missing.

Complainant remembered that on 18 June 2005, a Saturday, her younger sister went to
the RTC to ask for her necklace. Complainant took out from her table drawer a transparent
plastic sachet which contained her ring and bracelet, and her sister’s necklace, and after
handing over to her sister the necklace, she returned the plastic sachet, still containing the
bracelet and ring, to her table drawer. She maintained that the only person who was
present and saw her take out the jewelry from her table drawer was respondent, whose
table is adjacent to hers.

According to complainant, when she found out that her ring and bracelet were missing, she
informed her officemates about it, but nobody claimed to have seen the missing jewelry.
On 28 July 2005, an officemate, Anecito D. Altone (Altone), confided to her that he heard
from his landlady, Anastacia R. Nable (Nable), that respondent and his wife, Milagros, had
a quarrel because the latter discovered a ring and a bracelet in respondent’s coin purse.
Milagros suspected that respondent bought the jewelry for his mistress. Complainant
approached the RTC presiding judge, Judge Absalon U. Fulache (Judge Fulache), and
relayed to him the information she gathered. Judge Fulache advised her to invite Nable
and Milagros to his chambers so he could confirm the information.

In a separate meeting with Judge Fulache, respondent confessed that he found


complainant’s jewels in the court’s premises, but he could no longer return them because
he already threw them away.

ISSUE

Whether or not Urdaneta can be faulted for throwing away the jewelry he found.

RULING

Yes, he can be faulted. When a person who finds a thing that has been lost or mislaid by
the owner takes the thing into his hands, he acquires physical custody only and does not
become vested with legal possession. In assuming such custody, the finder is charged with
the obligation of restoring the thing to its owner. It is thus respondent’s duty to report to his
superior or his officemates that he found something.

The Civil Code, in Article 719, explicitly requires the finder of a lost property to report it to
the proper authorities, thus: Article 719. Whoever finds a movable, which is not treasure,
must return it to its previous possessor. If the latter is unknown, the finder shall immediately
deposit it with the mayor of the city or municipality where the finding has taken place.The
finding shall be publicly announced by the mayor for two consecutive weeks in the way he
deems best.If the movables cannot be kept without deterioration, or without the expenses
which considerably diminish its value, it shall be sold at public auction eight days after the
publication.Six months from the publication having elapsed without the owner having

45 | P a g e
MABELLE J. ARELLANO PROPERTY
L02 Saturday, 8:00 AM-12:00 PM

appeared, the thing found, or its value, shall be awarded to the finder. The finder and the
owner shall be obliged, as the case may be, to reimburse the expenses.

46 | P a g e
MABELLE J. ARELLANO PROPERTY
L02 Saturday, 8:00 AM-12:00 PM

FLOREZA V. EVANGELISTA
G.R. NO. L-25462 FEBRUARY 21, 1980
96 SCRA 130
MELENCIO-HERRERA, J:

FACTS

The Evangelistas were the owner of a residential lot in Rizal with an area of 204.08 sq. m.
assessed at P410.

• May 1945: Evangelistas borrowed P100 from Floreza.


• November 1945: Floreza occupied the residential lot and built a house of
lightmaterial (barong-barong) with the consent of the Evangelistas.
• Additional Loans made by the Evangelistas: Sept. 1946 – P100, August 1947 –
P200, January 1949 – P200, April 1949 – P140. TOTAL = P740 (including firstloan)
• January 1949: Floreza demolished the house of light material and constructedone
of strong material assessed at P1400. Floreza has not been paying anyrentals
since the beginning of their transactions.
• August 1949: Evangelistas sold, with a right to repurchase within 6 years,their land
to Floreza for P1000.
• Seven months before the expiry of the repurchase period, the Evangelistaswere
able to pay in full.
• Floreza refused to vacate the lot unless he was first reimbursed for the valueof the
house he built.
• Evangelistas filed a complaint. CFI ruled based on Art, 448 of the Civil Codesaying
that Evangelistas have the choice between purchasing the house orselling the land
to Floreza.
• CA ruled that Art. 448 was inapplicable and that Floreza was not entiled tothe
reimbursement of his house and could remove the same at his ownexpense.

ISSUES

1. Whether or not Floreza was entitled to reimbursement of the cost of his house. NO.

2. Whether or not he (his heirs who replaced him) should pay rental of the land. YES.

RULING

1. Issue of reimbursement is not moot because if Floreza has no right of retention, then
he must pay damages in the form of rentals. Agree with CA that Art. 448 is inapplicable
because it applies only when the builder is in good faith (he believed he had a right to
build).Art. 453 is alsonot applicable because it requires both of the parties to be in bad
faith. Neither is Art. 1616 applicable because Floreza is not a vendee a retro.

Thehouse was already constructed in 1945 (light materials) even before the pacto de
retro was entered into in 1949.Floreza cannot be classified as a builder in good faith nor a
vendee a retro, who made useful improvements during the pacto de retro, he has no right
to reimbursement of the value of the house, much less to the retention of the premises until
he is paid.

His rights are more akin to a usufructury under Art. 579, who may make on the property
useful improvements but with no right to be indemnified thereof, He may, however, remove
such improvements should it be possible to do so without damage to the property.

2. From the time the redemption price was paid in January 3, 1955, Floreza’s right to use
the residential lot without rent ceased. He should be held liable for damages in the form of
rentals for the continued use of the lot for P10monthly from January 3, 1955 until the house
was removed and the property vacated by Floreza or his heirs.

Judgment affirmed with modification

47 | P a g e
MABELLE J. ARELLANO PROPERTY
L02 Saturday, 8:00 AM-12:00 PM

MERCADO V. CA
G.R. NO. L-44001 JUNE 10, 1988
162 SCRA 75, 85 1988

NARVASA, J.:

FACTS

The private respondents Bulaong Group, had for many years been individual lessees of
stalls in the public market of Baliuag, Bulacan; from 1956 to 1972. The market was
destroyed by fire on February 17, 1956; the members of the Bulaong Group constructed
new stalls therein at their expense; and they thereafter paid rentals thereon to the
Municipality of Baliuag.In 1972, the members of the group sub-leased their individual stalls
to other persons, referred to as the Mercado Group. After the Mercado Group had been in
possession of the market stalls for some months, as sub-lessees of the Bulaong Group, the
municipal officials of Baliuag cancelled the long standing leases of the Bulaong Group and
declared the persons comprising the Mercado Group as the rightful lessees of the stalls in
question, in substitution of the former.The members of the Bulaong Group sued.

They filed several individual complaints with the Court of First Instance seeking recovery
of their stalls from the Mercado Group as well as damages.Their theory was anchored on
their claimed ownership of the stalls constructed by them at their own expense, and their
resulting right, as such owners, to sub-lease the stalls, and necessarily, to recover them
from any person withholding possession thereof from them.On October 24,1975,
respondent Judge rendered a summary judgment in all the cases. 3 It rejected the claim of
the Municipality of Baliuag that it had automatically acquired ownership of the new stalls
constructed after the old stalls had been razed by fire, declaring the members of the
Bulaong Group to be builders in good faith, entitled to retain possession of the stalls
respectively put up by them until and unless indemnified for the value thereof. The decision
also declared that the Bulaong and Mercado Groups had executed the sub-letting
agreements with full awareness that they were thereby violating Ordinance No. 14; they
were thus in pari delicto,and hence had no cause of action one against the other and no
right to recover whatever had been given or demand performance of anything undertaken.

The judgment therefore decreed (1) the annulment of the leases between the Municipality
and the individuals comprising the Mercado Group (the defendants who had taken over the
original leases of the Bulaong Group); and (2) the payment to the individual members of
the Bulaong Group (the plaintiffs) of the stated, adjudicated value of the stalls, with interest
IF —The members of the Mercado Group are now before this Court on an appeal
by certiorari, this time timely taken, assailing the above rulings of the Court of Appeals.
Their appeal must fail for lack of merit. No error can be ascribed to the judgment of the
Court of Appeals which is hereby affirmed in toto.

ISSUE

Whether or not Bulaong Group can be considered builders in good faith.

RULING

It was held that “to be deemed a builder in good faith, it is essential that a person assert
title to the land on which he builds; i.e., that he be a possessor in concept of owner, and
that he be unaware ‘that there exists in his title or mode of acquisition any flaw which
invalidates it.’Lessees cannot be considered builders in good faith (taken from Haystacks,
by Berne Guerrero)The members of the Bulaong group were admittedly lessees of space
in the public market; they therefore could not, and in truth never did make the claim, that
they were owners of any part of the land occupied by the market so that in respect of any
new structure put up by them thereon, they could be deemed builders in good faith (in
accordance with Article 526 of the Civil Code).

To be deemed a builder in good faith, it is essential that a person assert title to the land on
48 | P a g e
MABELLE J. ARELLANO PROPERTY
L02 Saturday, 8:00 AM-12:00 PM

which he builds; i.e., that he be a possessor in concept of owner, and that he be unaware
“that there exists in his title or mode of acquisition any flaw which invalidates it. It is such a
builder in good faith who is given the right to retain the thing, even as against the real
owner, until he has been reimbursed in full not only for the necessary expenses but also
for useful expenses. On the other hand, unlike the builder in good faith, a lessee who
“makes in good faith useful improvements which are suitable to the use for which the lease
is intended, without altering the form or substance of the property leased,” can only claim
payment of “one-half of the value of the improvements” or, “should the lessor refuse to
reimburse said amount, remove the improvements, even though the principal thing may
suffer damage thereby.”

49 | P a g e
MABELLE J. ARELLANO PROPERTY
L02 Saturday, 8:00 AM-12:00 PM

BULACANAG V. FRANCISCO
122 SCRA 498, 502 (1983)

FACTS

 The petitioner bought a lot owned by Mrs. Charvet which was then previously
leased by the latter to one Richard Stohner. The said lease contract provided that
the lessee may erect structures and improvements which shall remain as lessee's
property and he may remove them at any time.
 It further provided that should the lessee fail to remove the same structures or
improvements within two months after the expiration of the lease, the lessor may
remove them or cause them to be removed at the expense of the lessee.
 Stohner made fillings on the land and constructed a house. When he failed to pay
the rent, the petitioner, through counsel, sent Stohner a demand letter ordering him
to vacate the lot. The lessee contended that he is a 'builder in good faith.'

ISSUE

Whether or not the the lessee a builder in good faith?

RULING

 The lessee cannot be considered a builder in good faith. The provision under Art.
448 of the New Civil Code (Philippine) on a builder of good faith applies only to the
owner of the land who believes he is the rightful owner thereof, but not to a lessee
who's interest in the land is derived only from a rental contract. Neither can Stohner
be considered a 'possessor in good faith'.

 A possessor in good faith is a party who possesses property believing that he is its
rightful owner but discovers later on a flaw in his title that could indicate that he
might not be its legal owner. It cannot apply to a lessee because he knows right
from the start that he is merely a lessee and not the owner of the premises.

 As a mere lessee, he introduces improvements to the property at his own risk such
that he cannot recover from the owner the reimbursements nor he has any right to
retain the premises until reimbursements. What applies in this case is Art. 1678
(NCC) which provides that, " if the lessee, makes, in good faith, useful
improvements which are suitable to the use for which the lease is intended, without
altering the form or substance of the property leased, the lessor upon the
termination of the lease shall pay the lessee 1/2 of the value of the improvements
at the time. Should the lessor refuse to reimburse said amount, the lessee may
remove the improvements even though the principal thing may suffer damage
thereby. He shall not. however, cause any more impairment upon the property
leased than is necessary."

WHEREFORE, the decision in Civil Case No. 67503 is hereby set aside, with costs against
respondent Stohner. The latter is ordered to vacate the premises in question and to pay
Rogelio Balucanag the rentals due from March 1969 up to the time he surrenders the
premises, at the rate of P40.00 a month.

50 | P a g e
MABELLE J. ARELLANO PROPERTY
L02 Saturday, 8:00 AM-12:00 PM

PNB VS DE JESUS
G.R. NO. 149295. SEPTEMBER 23, 2003
411 SCRA 557

VITUG, J.:

FACTS

 Respondent filed a complaint against petitioner before the Regional Trial Court of
Occidental Mindoro for recovery of ownership and possession, with damages, over
the questioned property. In his complaint, respondent stated that he had acquired
a parcel of land situated in Mamburao, Occidental Mindoro, with an area of 1,144
square meters covered by TCT No. T-17197, and that on 26 March 1993, he had
caused a verification survey of the property and discovered that the northern portion
of the lot was being encroached upon by a building of petitioner to the extent of 124
square meters. Despite two letters of demand sent by respondent, petitioner failed
and refused to vacate the area.

 Petitioner, in its answer, asserted that when it acquired the lot and the building
sometime in 1981 from then Mayor Bienvenido Ignacio, the encroachment already
was in existence and to remedy the situation, Mayor Ignacio offered to sell the area
in question (which then also belonged to Ignacio) to petitioner at P100.00 per
square meter which offer the latter claimed to have accepted. The sale, however,
did not materialize when, without the knowledge and consent of petitioner, Mayor
Ignacio later mortgaged the lot to the Development Bank of the Philippines. The
trial court decided the case in favor of respondent declaring him to be the rightful
owner

ISSUE

Whether or not being a builder in good faith matters under article 448 of the New Civil
Code.

RULING

Article 448, of the Civil Code refers to a piece of land whose ownership is claimed by two
or more parties, one of whom has built some works (or sown or planted something) and
not to a case where the owner of the land is the builder, sower, or planter who then later
loses ownership of the land by sale or otherwise for, elsewise stated, “where the true
owner himself is the builder of works on his own land, the issue of good faith or bad faith
is entirely irrelevant.”

WHEREFORE, the decision of the Court of Appeals in CA-G.R. CV No. 56001 is


AFFIRMED.

51 | P a g e
MABELLE J. ARELLANO PROPERTY
L02 Saturday, 8:00 AM-12:00 PM

JAVIER V. CONCEPCION, JR
G.R. NO. L-36566 NOVEMBER 7, 1979
94 SCRA 212

SANTOS, J.:

FACTS

 On October 17, 1959, respondents as plaintiffs, Lim Chua, Tan Tian On alias Tan
Tian Una and Tan Sick Tan alias Tan Shiok Tuan filed against herein petitioners,
then defendants- spouses, Urbano Javier and Leonila Albiela, with the Court of First
Instance of the Province of Quezon, Civil Case No. 6253, for the reconveyance to
the former of a parcel of land with improvements thereon, known as Lot 12
consisting of fifty (50) hectares, more or less, and an accounting and recovery of
the produce of the land from the time the latter, i.e., petitioners herein, took
possession of the same in 1945 up to the time possession is returned to the former.

 Defendants-petitioners claimed P5,000.00 for attorney's fees and P1,000.00 for


litigation expenses and that in the event that plaintiffs-respondents are declared the
lawful owners of the lot in question, they be reimbursed the amount of P150,000.00
for the reasonable value of improvements they introduced thereon consisting of a
house, camarin made of strong materials and various fruit trees.

 With respect to the claim of the defendants that they acquired the property by
prescription, the same is without merit taking into consideration that the defendant
Urbano Javier knew that the property in question is within lot No. 6 and covered by
a certificate of title in favor of the plaintiffs since 1924 when he filed an opposition
to the registration of the land in question and, therefore, could not be said to have
acted in good faith for the purpose of applying the provision of the Civil Code in
ordinary prescription.

ISSUE

1. Whether or not in holding that there was no fraud in the registration of Lot No. 12,
Plan Psu-16536-AMD-3;
2. Whether or not in holding that the cause of action of the private respondents has
not been barred by the Statute of Limitation or by laches; and
3. Whether or not in not ordering private respondents to pay for the improvements
introduced by them on the land in question from 1945.

RULING

1. The decree of registration has long become final, absent a showing that the same
was questioned within one (1) year after the entry thereof was made. Under Section
38 of the Land Registration Act, as amended, the person allegedly deprived of the
land by a decree of registration obtained by fraud should file in the competent Court
of First Instance a petition for review within one year after the entry of the decree
provided no innocent purchaser for value has acquired an interest. Thus, granting
that there was actual or positive fraud in securing the title, defendants-petitioners
are now barred from questioning the same.

2. It can be readily seen that in the above-cited case the land in question came into
the possession of the defendant-appellant Gamponia after a series of transfers from
Domingo Mejia, the original owner and plaintiff-appellee's (Mejia de Lucas')
predecessor-in-interest to three other persons and their successors-in-interest,
whose rights and obligation would have been affected by a contrary decision. Here,
there are no intervening rights of third persons which may be affected or prejudiced
by a decision directing the return of Lot No. 12 to plaintiffs-respondents, Hence, the
equitable defense of laches will not also apply as against the registered owners in
this case.

52 | P a g e
MABELLE J. ARELLANO PROPERTY
L02 Saturday, 8:00 AM-12:00 PM

3. As possessors in good faith, petitioners are entitled to the fruits received before
their possession was legally interrupted upon receipt of judicial summons in
connection with the filing of the complaint for reconveyance on October 17,
1959. However, the records do not show when the summons were received by the
defendants-spouses, Javier, In the absence of such proof and in the interest of
justice, we hold that possession in good faith was legally interrupted on November
11, 1959. In view of Article 544 of the Civil Code, petitioners shall be accountable
for the fruits of subject property only after 1959, not from 1945.

WHEREFORE, the decision of the Court of Appeals is hereby AFFIRMED, with the
modification that petitioners render an accounting of the entire produce of the holding from
November 11, 1959, which, with respect to palay crop had been fixed at sixty (60) cavans
a year at seven pesos (p7.00) per cavan as of September 23, 1960, up to the time the
subject land is actually reconveyed to private respondents. The value of necessary and
useful expenses due petitioners in the amount of One Hundred Fifty Thousand Pesos
P150,000.00) having been proved and not controverted, no further proof is required. Let
the records of this case be remanded to the Court of origin for the determination of the
value of the entire produce, in addition to the palay crop, to which private respondents are
entitled from November 11, 1959 to the time possession of subject property is delivered to
them by petitioners.

53 | P a g e
MABELLE J. ARELLANO PROPERTY
L02 Saturday, 8:00 AM-12:00 PM

NUGUID V. CA
G.R. NO. 151815. FEBRUARY 23, 2005
452 SCRA 243, 252

FACTS

The deceased spouses Victorino and Crisanta dela Rosa (spouses dela Rosa) were
registered owners of a parcel of land in Orani, Bataan, and covered by OCT No. 3778. On
or about May 4, 1931, Victorino dela Rosa (widowed by then) sold one-half of the said
property to Juliana Salazar for P95.00. This sale between him and Salazar, though
evidenced by a document, was not registered. Nevertheless, Juliana Salazar constructed
a house on the lot she purchased immediately after the sale. On March 10, 1964, petitioner
spouses Diosdado Nuguid and Marqiueta Venegas (spouses Nuguid) caused the
registration of a document entitled "Kasulatan ng Partihan at Bilihan" (Kasulatan) dated
June 6, 1961. In this document, Marciana dela Rosa, together with the heirs of Victorino
and Crisanta dela Rosa, sold to spouses Nuguid the entire area of the property for the sum
of P300.00. Subsequently, OCT No. 3778 was cancelled by the Register of Deeds of
Bataan, and TCT No. T-12782 was issued in the spouses Nuguid’s names.

Private respondents claimed that the presented by spouses Nuguid was forged. They also
allegedly discovered the forged deed as well as the certificate of title in the name of the
petitioners much later, that is, on February 28, 1978, when respondents Amorita Guevarra
and Teresita Guevarra thought of having the title of their grandmother Juliana Salazar,
registered. On the other hand, spouse Nuguid assert that in the latter part of 1960, Nicolas
dela Rosa, uncle of respondent Marciana dela Rosa and grandfather of the other heirs-
signatories, offered to sell the subject land to them. Apparently, Nicolas dela Rosa claimed
that he had already purchased the shares of the heirs over the subject property as
evidenced by a private document entitled "Kasunduan" (Kasunduan) dated August 31,
1955, and as a matter of fact, he had in his possession the original certificate of title
covering the property in the name of the deceased Victorino and Crisanta dela Rosa.

The CFI of Bataan dismissed the complaint filed by private respondents, but the Court of
Appeals reversed said decision and ordered the spouses Nuguid to execute a deed of
reconveyance in favor of herein respondents.

ISSUE

Who is the rightful owner of the subject property?

RULING

The Supreme Court reinstated the decision of the CFI of Bataan. The basis for the Court
of Appeals' conclusion that petitioners were buyers in bad faih is ambiguous because said
court relied on the singular circumstance that the petitioners are from Orani, Bataan, and
should have personally known that the private respondents were the persons in actual
possession. However, at the time of the purchase, the spouses Nuguid dealt with Pedro
Guevarra and Pascuala Tolentino, the latter being the actual occupants. The respondents
Guevarras, children of the said Pedro and Pascuala Guevarra, came into the picture only
after their parents died. As for the respondent heirs of Victorino dela Rosa, their being in
actual possession of any portion of the property was, likewise, simply presumed or taken
for granted by the Court of Appeals.

The private respondents cannot also honestly claim that they became aware of the
spouses Nuguid’s title only in 1978, because ever since the latter bought the property in
1961, the spouse Nuguid have occupied the same openly, publicly, and continuously in the
concept of owners, even building their house thereon. For seventeen years they were in
peaceful possession, with the respondents Guevarras occupying less than one-half of the
same property.

54 | P a g e
MABELLE J. ARELLANO PROPERTY
L02 Saturday, 8:00 AM-12:00 PM

BALLATAN V. COURT OF APPEALS


G.R. NO. 125683. MARCH 2, 1999
304 SCRA 37 (1999)

PUNO, J.:

FACTS

Eden Ballatan, together with other petitioners, is living in and registered owners of Lot No.
24. Respondent Winston Go is living in and registered owners of Lot No. 25 and 26. And
Li Ching Yao is living in and the registered owner of Lot. 27. The Lots are adjacent to each
other.

When Ballatan constructed her house in her lot, she noticed that the concrete fence and
side pathway of the adjoining house of respondent Winston Go encroached on the entire
length of the eastern side of her property. She was informed by her contractor of this
discrepancy, who then told respondent Go of the same. Respondent, however, claims that
his house was built within the parameters of his father’s lot; and that this lot was surveyed
by engineer Jose Quedding, the authorized surveyor of Araneta Institute of Agriculture
(AIA). Petitioner called the attention of AIA on the matter and so the latter authorized
another survey of the land by Engineer Quedding. The latter then did the survey twice
which led to the conclusion that Lots Nos 25, 26 (owned by respondent Go) and 27 (owned
by Li Ching Yao) moved westward to the eastern boundary of Lot 24 (owned by petitioner
Ballatan.) –(it was later on discovered by the courts that Go encroached 42 square meters
from the property of Ballatan and Yao encroached 37 square meters on Go’s property, all
of which were in GOOD FAITH) Ballatan made written demands to the respondent to
dismantle and move their improvements and since the latter wasn’t answering the
petitioner filed accion publiciana in court. Go’s filed their “Answer with Third-Party
Complaint” impleading as third party defendants respondents Li Ching Yao, the AIA and
Engineer Quedding.

RTC ruled in favor of the petitioner ordering respondent Go to demolish their improvements
and pay damages to Petitioner but dismissing the third-party complaint. CA affirmed the
dismissal of the third party-complaint as to AIA but reinstated the the complaint against
Yao and the Engineer. CA also affirmed the demolition and damages awarded to petitioner
and added that Yao should also pay respondent for his encroachment of respondent Go’s
property. Jose Quedding was also ordered to pay attorney’s fees for his negligence which
caused all this fuzz.

ISSUE

What is the proper remedy in this situation (everyone was in good faith)?

RULING

Art 448 is the proper remedy (Lower Courts are wrong in awarding the damages). It was
established in the case that the parties had no knowledge of the encroachment until
Ballatan noticed it there all of them were builders in Good faith. In that scenario they have
two options. 1st option is that the land owner will buy the improvements and the 2nd option
is to oblige the builders to buy the land given that the value of the land is not considerably
more than the buildings or tree; otherwise the owner may remove the improvements
thereon.

The builder, planter or sower, however, is not obliged to purchase the land if its value is
considerably more than the building, planting or sowing. In such case, the builder, planter
or sower must pay rent to the owner of the land. If the parties cannot come to terms over
the conditions of the lease, the court must fix the terms thereof. The right to choose
between appropriating the improvement or selling the land on which the improvement of
the builder, planter or sower stands, is given to the owner. If the option chooses is to sell
the lot, the price must be fixed at the prevailing market value at the time of payment.

55 | P a g e
MABELLE J. ARELLANO PROPERTY
L02 Saturday, 8:00 AM-12:00 PM

Petitioner was given by SC 30 days to decide on what to do or which right to exercise.


Likewise, Go was also given time to do the regarding Yao’s encroachment. Engineer
Quedding was still asked to pay attorney’s fees.

56 | P a g e
MABELLE J. ARELLANO PROPERTY
L02 Saturday, 8:00 AM-12:00 PM

HEIRS OF EMILIANO NAVARRO V. IAC


G.R. NO. 68166, FEBRUARY 12, 1997
268 SCRA 74, 85

HERMOSISIMA, JR., J.:

FACTS

Sinforoso Pascual sits in the midst of a land registration case. The story begins on 1946
upon his desire to register land on the northern section of his existing property. His current
registered property is bounded on the east by Talisay River, on the West by Bulacan River
and on the North by the Manila bay. Both rivers flow towards the Manila Bay. Because of
constantly flowing water, extra land of about 17hectares (that’s about the size of Disney
Park!) formed in the northern most section of the property. It is this property he sought to
register.

The RTC denied the registration claiming this to be foreshore land and part of public
domain (remember, accretion formedby the sea is public dominion). His Motion for
Reconsideration likewise burned. In 1960, he attempted registry again, claiming that the
Talisay and Bulacan rivers deposited more silt resulting on accretion. He claimed this land
as riprarian owner. The Director of Lands, Director of Forestry and the Fiscal opposed.

Then a new party surfaced. Mr Emiliano Navarro jumped into the fray opposing the same
application, stating the he leased part of the property sought to be registered. He sought
to protect his fishpond that rested on the same property. Sinforoso was not amused and
filed ejectment against Mr. Navarro, claiming that Navarro used stealth force and strategy
to occupy a portion of his land. Pascual lost the case against Navarro so he appealed.
During the appeal, his original land registration case was consolidated and tried jointly.
(alas Pascual died) The heirs of Pascual took over the case.
On 1975, the court decided that the property was foreshore land and therefore part of public
domain. The RTC dismissed the complaint of Pascual for ejectment against Navarro and
also denied his land registration request. Pascual’s heirs appealed and the RTC was
reversed by the IAC. The Apellate court granted petition for registration! The reason? The
accretion was caused by the two rivers, not manila bay. Hence it wasn’t foreshore land.
(BUT the confusion lies in the fact that the accretion formed adjacent to Manila Bay… which
is sea!) Aggrieved, the Director of Forestry moved for reconsideration (Government insists
it is foreshore and hence, public domain). The Apellate court denied all motions of the
Director and the Government.

The matter went to the SC.

ISSUE

Whether or not the accretion taking place on property adjacent to the sea can be registered
under the Torrens system.

RULING

It cannot be registered. This is land of Public domain. Pascual claimed ownership under
Article 457 of the Civil Code saying that the disputed 14-hectare land is an accretion caused
by the joint action of the Talisay and Bulacan Rivers Art 457: Accretion as a mode of
acquiring property and requires the concurrence of the following requisites: (1) that the
accumulation of soil or sediment be gradual and imperceptible; (2) that it be the result of
the action of the waters of the river; and (3) that the land where the accretion takes place
is adjacent to the bank of the river.

Unfortunately, Pasucal and Heirs claim of ownership based on Art 457 is misplaced. If
there’s any land to be claimed, it should be land ADJACENT to the rivers Talisay and
Bulacan. The law is clear on this. Accretion of land along the river bank may be registered.
This is not the case of accretion of land on the property adjacent to Manila Bay.

Furthermore, Manila Bay is a sea. Accretion on a sea bank is foreshore land and the
57 | P a g e
MABELLE J. ARELLANO PROPERTY
L02 Saturday, 8:00 AM-12:00 PM

applicable law is not Art 457 but Art 4 of the Spanish Law of Waters of 1866. This law,
while old, holds that accretion along sea shore cannot be registered as it remains public
domain unless abandoned by government for public use and declared as private property
capable of alienation.

Article 4 of the Spanish Law of Waters of August 3, 1866 provides as follows:

Lands added to the shores by accretions and alluvial deposits caused by the action of the
sea, form part of the public domain. When they are no longer washed by the waters of the
sea and are not necessary for purposes of public utility, or for the establishment of special
industries, or for the coast-guard service, the Government shall declare them to be the
property of the owners of the estates adjacent thereto and as increment thereof.

The IAC decision granting registration was reversed and set aside. Registration cannot be
allowed.

58 | P a g e
MABELLE J. ARELLANO PROPERTY
L02 Saturday, 8:00 AM-12:00 PM

VDA. DE NAZARENO V. CA
G.R. NO. 98045. JUNE 26, 1996
257 SCRA 598 (1996)

ROMERO, J.:

FACTS

A parcel of land situated in Telegrapo, Puntod, Cagayan de Oro City is said to have been
formed as a result of sawdust dumped into the dried-up Balacanas Creek and along the
banks of the Cagayan river. Jose Salasalan and Leo Rabaya leased the subject lots on
which their houses stood from one Antonio Nazareno, petitioners’ predecessor-in-interest.
Salasalan and Rabaya allegedly stopped paying rentals.

As a result, Antonio Nazareno and petitioners filed a case for ejectment with the MTC. A
decision was rendered against Salasalan and Rabaya, which decision was affirmed by the
RTC. The case was remanded to the Municipal trial court for execution of judgment after
the same became final and executory. Private respondents filed a case for annulment of
judgment before the RTC Misamis Oriental which was dismissed. The decision of the lower
court was finally enforced with the private respondents being ejected from portions of the
subject lots they occupied.

Before he died, Antonio Nazareno caused the approval by the Bureau of Lands of the
survey plan designated with a view to perfecting his title over the accretion area being
claimed by him. Before the approved survey plan could be released to the applicant,
however, it was protested by private respondents before the Bureau of Lands. The report
of the Land Investigator, made in compliance with the order of the District Land Officer,
recommended the Survey Plan MSI-10-06-000571-D (Lot 36302, Cad. 237) in the name
of Antonio Nazareno be cancelled and that private respondents be directed to file
appropriate public application. Based on the report, the Regional Director of the Bureau of
Lands rendered a decision ordering an amendment to the survey plan of Nazareno by
segregating therefrom the areas occupied by the private respondents. Antonio Nazareno
filed a motion for reconsideration with the Undersecretary of Natural Resources and OIC
of the Bureau of Lands; which was denied.

The petitioners Desamparada vda. De Nazareno and Leticia Tapia Nazero filed a case
before the RTC for the annulment of the decision and order of the Bureau of Lands
regarding the parcel of land.

The RTC dismissed the complaint for failure to exhaust administrative remedies, resulting
to the finality of the administrative decision of the Bureau of Lands. On appeal, the Court
of Appeals affirmed the decision of the RTC dismissing the complaint. Hence, the petition.

ISSUE

Whether or not accretion belongs to the riparian owners.

RULING

No. Supreme Court dismissed the petition for lack of merit.

Article 457 of the Civil Code provides that “to the owners of land adjoining the banks of
rivers belong the accretion which they gradually receive from the effects of the current of
the waters.” Accretion as a mode of acquiring property under Article 457 of the Civil Code,
requires the concurrence of these requisites: (1) that the deposition of soil or sediment be
gradual and imperceptible; (2) that it be the result of the action of the waters of the river (or
sea); and (3) that the land where accretion takes place is adjacent to the banks or rivers
(or the sea coast). These are called the rules on alluvion which if present in a case, give to
the owners of lands adjoining the banks of rivers or streams any accretion gradually
received from the effects of the current of waters.
59 | P a g e
MABELLE J. ARELLANO PROPERTY
L02 Saturday, 8:00 AM-12:00 PM

The application of the rules on alluvion cannot be made in the present case as the first and
second requirements of the rules were not met. Thus, the Nazarenos cannot claim the
rights of a riparian owner. By their own admission, the accretion was formed by the
dumping of boulders, soil and other filling materials on portions of the Balacanas Creek
and the Cagayan River bounding their land. It cannot be claimed, therefore, that the
accumulation of such boulders, soil and other filling materials was gradual and
imperceptible, resulting from the action of the waters or the current of the Balacanas Creek
and the Cagayan River.

Petitioners are estopped from denying the public character of the subject land, as well as
the jurisdiction of the Bureau of Lands when the late Antonio Nazareno filed his
Miscellaneous Sales Application MSA (G-6) 571. The mere filing of said Application
constituted an admission that the land being applied for was public land, having been the
subject of Survey Plan MSI-10-06-000571-D which was conducted as a consequence of
Antonio Nazareno’s Miscellaneous Sales Application wherein said land was described as
an orchard. Said description by Antonio Nazareno was controverted by the findings of the
ocular inspection that said land actually covers a dry portion of Balacanas Creek and a
swampy portion of Cagayan River.

The Bureau of Lands classified the subject land as an accretion area which was formed by
deposits of sawdust in the Balacanas Creek and the Cagayan river, in accordance with the
ocular inspection conducted by the Bureau of Lands. It has often enough held that findings
of administrative agencies which have acquired expertise because their jurisdiction is
confined to specific matters are generally accorded not only respect but even finality.
Again, when said factual findings are affirmed by the Court of Appeals, the same are
conclusive on the parties and not reviewable by the Supreme Court.

In the present case, the subject land was the direct result of the dumping of sawdust by the
Sun Valley Lumber Co. consequent to its sawmill operations. As the accretion site was the
result of the late Antonio Nazareno’s labor consisting in the dumping of boulders, soil and
other filling materials into the Balacanas Creek and Cagayan River bounding his land, the
same would still be part of the public domain.

60 | P a g e
MABELLE J. ARELLANO PROPERTY
L02 Saturday, 8:00 AM-12:00 PM

ROXAS V. TUASON
G.R. NO. L-3788 DECEMBER 21, 1907
9 PHIL. 408

TORRES, J.:

FACTS

On 19 February 1906, attorneys Rosado, Sanz & Opisso, on behalf of the petitioner Pedro
Roxas, applied for the registration of his the estate in accordance with the provisions of the
Land Registration Act. Said hacienda was acquired by the petitioner by inheritance under
the will of his late father. The property consists of four different parcels of land, irregular
shape, designated on the accompanying plan under the letters "A", "B", "C", and "D". The
building constructed of strong materials, called the "Casa-Quinta" or "Casa de Ingenieros,"
belonging also to the petitioner, is erected within parcel "C". It does not appear that said
hacienda is mortgaged nor that any person has any right to or any interest therein; and it
is almost wholly occupied at the present time, under lease, by about 429 tenants.

The owners of the adjoining properties having been summoned and notified by means of
subpoenas and notices published in the daily papers, one of them is the respondent Julia
Tuason, as regards the parcel marked "C," for the reason that two old monuments which
had separated their respective properties had been pulled down and new ones erected
without her consent, and in her opinion the latter included a considerable portion of the
land owned by her. The municipality of San Pedro Macati also filed opposition to the
requested registration, alleging that the land occupied by the municipal building and the
public school had been in the possession of the town from time immemorial, and that all
the land occupied by roads, highways, lanes, and public landing places belonged to the
public domain and should be excluded from registration in favor of the petitioner. Under
date of the 18th of September, 1906, the attorney for the Aguirre’s also filed opposition to
said application for registration alleging that the two parcels of land owned by them had
been improperly included within the bounds of said hacienda in the parcel marked "C".

Evidence consisting of both oral testimony and documents, which appears in the record,
having been adduced by both parties in the suit, the judge ordered the registration of the
Hacienda of San Pedro Macati in favor of the petitioner. The respondents excepted to said
judgment and moved for a new trial on the ground that the same was contrary to law and
to the weight of the evidence. The said motion was overruled, the respondents again
excepting. The respective bills of exceptions having been presented, the same were
forwarded in the ordinary manner to the Supreme Court.

ISSUE

Whether or not there is an evidence showing that the respondents owns the strip of land
on the bank of the creek.

RULING

The record does not show that the boundary of the land of Julia Tuason was inclosed by
monuments belonging to her or that the creek is included within the Tuason’s land, since
in the bill of sale executed by the procurador general of the Augustinian friars to Julia
Tuason, no mention is made of monuments erected thereon nor of any creek existing in
the large tract of land purchased by her. Nor does the record show that there was more
land on the side of the hacienda, and that said creek traversed said barrio from one end to
the other. Thus, from the fact that the land of Julia Tuason was bounded on two sides by
the Hacienda of San Pedro Macati it does not follow that the strip of a few meters in width
on the bank of the creek belonged to her, there being no evidence in support thereof, and
if her statement were true, she would have applied for a survey and demarcation of her
property in accordance with the area of the same stated in her title deed; and if she did not
do so it must be because she renounces its verification in this manner or for some other
reason.

61 | P a g e
MABELLE J. ARELLANO PROPERTY
L02 Saturday, 8:00 AM-12:00 PM

The fact that said creek was wider in 1871 than at present, is the best explanation as to
why some of the monuments of the Hacienda of San Pedro Macati are now at some
distance from the bank of the same, and no legal reason exists why the slow increase
which has taken place on the hacienda's side should be considered as belonging to the
respondent, inasmuch as the latter does not own the bed of the creek and because it may
be assumed that the slow decrease in the width thereof benefited both properties equally
since the respondent has not been able to show or prove that her land has been thereby
reduced.

Article 366 of the Civil Code in dealing with the right of accession to real property reads:
“The accretions which banks of rivers may gradually receive from the effects of the currents
belong to the owners of the estates bordering thereon.”

The provision in this article is perfectly applicable to the strip of land, which, on account of
the accretion, has come to be undeniable increase in the land of the hacienda inasmuch
as it has increased all along the bank of the creek, the gradual effect of the currents; and
even though the law does not require an express act of possession of the accretion which
has enlarged the estate, it is certain that the owner of the hacienda has possessed it for
more than thirty years through his tenants, who have been cultivating their respective
parcels of land together with the corresponding portion of the said strip down to the bank
of said creek.

The result of the evidence, as stated in the judgment appealed from, does not maintain the
claim of the respondent; on the contrary, it has been shown in a convincing manner that
the present natural limit of both properties is the aforesaid creek; therefore, the opposition
filed by Julia Tuason is untenable.

The Supreme Court affirmed the judgment appealed from should be affirmed as regards
the respondents who have appealed.

62 | P a g e
MABELLE J. ARELLANO PROPERTY
L02 Saturday, 8:00 AM-12:00 PM

CUREG V. IAC
G.R. NO. 73465 SEPTEMBER 7, 1989
177 SCRA 313

MEDIALDEA, J.:

FACTS

In 1982 Apostol, et al filed a complaint for quieting of title and damages with preliminary
injunction against the Carniyans with the RTC of Isabela. Apostol, et al alleged that they
are the legal heirs of Domingo Geraro who has been in open, continuous, exclusive and
notorious possession of a parcel of land referred to as “motherland” since time immemorial.
That said land was declared for taxation purposes in the name of Francisco Gerardo.
During the execution of the extra-judicial partition with voluntary reconveyance, the
motherland already manifested signs of accretion of about 3 hectares on the north caused
by the northward movement of the Cagayan River; that Domingo Apostol declared the
motherland and its accretion for tax purposes under a tax declaration. Apostol et al. were
about to cultivate their “motherland” together with its accretion, they were prevented and
threatened by the Carniyans from continuing to do so.

Carniyans’ alleged that the “motherland” is non-existent. that the "motherland; that Antonio
Carniyan, petitioners' predecessor-in-interest, was the owner of a piece of land bounded
on the north by Cagayan River and not by the land of Francisco Gerardo as claimed by
private respondents; that the "subject land" is an accretion to their registered land and that
petitioners have been in possession and cultivation of the "accretion" for many years now.

In a decision rendered by the trial court held that respondent Domingo Apostol, thru his
predecessors-in-interest had already acquired an imperfect title to the subject land. The
petitioners appealed to the then Intermediate Appellate Court which affirmed the decision
of the trial court.

ISSUE

Whether or not the petitioners can be considered as riparian owners who are entitled to the
"subject land" which is an accretion to the registered land.

RULING

Yes. Original certificate of title is better than tax declaration.

It should be noted that the herein Apostols’ claim of ownership of their alleged two and a
half hectare "motherland" is anchored mainly on four (4) tax declarations while the
Carniyans relied on the indefeasibility and incontrovertibility of their OCT. The declaration
of ownership for purposes of assessment on the payment of the tax is not sufficient
evidence to prove ownership nor proof of the area covered therein. Since OCT clearly
stated that subject land is bounded on the north by the Cagayan River, Apostol’s claim
over their “motherland”, allegedly existing between petitioners’ land and the Cagayan
River, is deemed barred and nullified with the issuance of the OCT.

Thus the alleged “motherland” claimed by private respondents is non-existent. The “subject
land” is an alluvial deposit left by the northward movement of the Cagayan River and
pursuant to NCC 457: “to the owners of the land adjoining the banks of river belong the
accretion which they gradually receive from the effects of the current of the waters.”

It should be noted that the area covered by the OCT is only 4,584 square meters. The
accretion attached to said land is approximately 5.5 hectares. The increase in the area of
petitioners' land, being an accretion left by the change of course or the northward
movement of the Cagayan River does not automatically become registered land just
because the lot which receives such accretion is covered by a Torrens title. The petition is
GRANTED. The decision of AIC is REVERSED.

63 | P a g e
MABELLE J. ARELLANO PROPERTY
L02 Saturday, 8:00 AM-12:00 PM

AGNE V. DIRECTOR OF LANDS


G.R. NO. L-40399 FEBRUARY 6, 1990
181 SCRA 793, 805

REGALADO, J.:

FACTS

Petitioners filed a complaint against the respondents Director of Lands and spouses
Agpoon with the former Court of First Instance of Pangasinan for annulment of title,
reconveyance of and/or action to clear title to a parcel of land. Petitioners alleged in their
said complaint that the land in question, which was formerly a portion of the bed of Agno-
Chico river which was abandoned as a result of the big flood in 1920, belongs to them
pursuant to the provision of Article 370 of the old Civil Code; that when respondent spouses
filed a complaint against them, they found out that the said land was granted by the
Government to Herminigildo Agpoon under Free Patent No. 23263, pursuant to which
Original Certificate of Title No. 2370 issued in the latter's name; and that the said patent
and subsequent titles issued pursuant thereto are null and void since the said land, an
abandoned river bed, is of private ownership and, therefore, cannot be the subject of a
public land grant.

ISSUE

Whether or not the subsequent derivative certificates of title in question were null and
void ab initio.

RULING

Yes. Private ownership of land is not affected by the issuance of a free patent over the
same land because the Public Land Act applies only to lands of the public domain. Only
public land may be disposed of by the Director of Lands. Since as early as 1920, the land
in dispute was already under the private ownership of herein petitioners and no longer a
part of the lands of the public domain, the same could not have been the subject matter of
a free patent. The patentee and his successors in interest acquired no right or title to the
said land. Necessarily, Free Patent No. 23263 issued to Herminigildo Agpoon is null and
void and the subsequent titles issued pursuant thereto cannot become final and
indefeasible.

A free patent which purports to convey land to which the Government did not have any title
at the time of its issuance does not vest any title in the patentee as against the true
owner. The Court has previously held that the Land Registration Act and the Cadastral Act
do not give anybody who resorts to the provisions thereof a better title than what he really
and lawfully has.

64 | P a g e
MABELLE J. ARELLANO PROPERTY
L02 Saturday, 8:00 AM-12:00 PM

CHUNG JR. VS. MONDRAGON


G.R. 179754,NOV. 21, 2012

DEL CASTILLO, J.:

FACTS

Petitioners Joaquin G. Chung, Jr., Paz Royeras-Soler, and Mansueto Maceda are
descendants of Rafael Mondragon (Rafael) by his first wife, Eleuteria Calunia (Eleuteria),
while respondent Jack Daniel Mondragon (Jack Daniel) is Rafaels descendant by his
second wife, Andrea Baldos (Andrea).

OCT No. 22447 is registered in the name of "Heirs of Andrea Baldos represented by Teofila
G. Maceda" and covers 16,177 square meters of land in Macrohon, Southern Leyte (the
land). Chung, et al. claim that from 1921 up to 2000, Rafael appeared as owner of the land
in its tax declaration, and that a free patent was issued in 1987 in the name of Andreas
heirs upon application of Teofila G.Maceda (Teofila), who is petitioners sister.

On the other hand, respondents Bourbon, et al. claim that Andrea is the exclusive owner
of the land, having inherited the same from her father and that after Andrea died, his son
Fortunato Mondragon inherited the land; and when the latter died, his son Jack Daniel
(herein respondent) came into possession and enjoyment thereof. Sometime in the year
2000, Jack Daniel sold a 1,500-square meter portion of the land to his co-respondent
Clarinda Regis-Schmitz (Regis-Schmitz).

On the claim that Jack Daniel had no right to sell a portion of the land and that the sale to
Regis-Schmitz created a cloud upon their title, Chung, Jr., et al. filed an action to quiet title.
The RTC dismissed the complaint of Chung, Jr., et al. The CA sustained the trial court.

ISSUE

Whether or not the action to quiet title should prosper?

RULING

The petition lacks merit.

CIVIL LAW: quieting of title

The issues in a case for quieting of title are fairly simple; the plaintiff need to prove only
two things, namely: "(1) the plaintiff or complainant has a legal or an equitable title to or
interest in the real property subject of the action; and (2) that the deed, claim, encumbrance
or proceeding claimed to be casting a cloud on his title must be shown to be in fact invalid
or inoperative despite its prima facie appearance of validity or legal efficacy.

It is evident from the title that the land belongs to no other than the heirs of Andrea Baldos,
Rafaels second wife. The land could not have belonged to Rafael, because he is not even
named in OCT No. 22447.With greater reason may it be said that the land could not belong
to petitioners, who are Rafaels children by his first wife Eleuteria. Unless Eleuteria and
Andrea were related by blood such fact is not borne out by the record they could not be
heirs to each other. Add to this is the fact that petitioners are not in possession of the land.
Petitioners do not possess legal or equitable title to the land.

Petition is DENIED.

65 | P a g e
MABELLE J. ARELLANO PROPERTY
L02 Saturday, 8:00 AM-12:00 PM

BAHA’IS V. PASCUAL
G.R. 169272,JULY 11, 2012

BRION, J.:

FACTS

December 11, 2000, Bahais filed a complaint with RTC for quieting of title, injunction and
other claims against Silverio Songcuan and/or his heirs, the secretary of DENR and the
regional executive director of DENR of Tuguegarao, Cagayan.

Bahais alleged that it is the lawful and absolute owner of two parcels of land, who acquired
ownership from Marcelina Ordono. The Bahais had been in possession of the land for 30
years, until the Bureau of Lands rejected the sales applications of the Bahais
predecessors-in-interest for the lots and ordered all those in privity to vacate the lots and
to remove their improvements. DENR secretary affirmed with this decision. Recourse to
the office of the President had been unavailing, so DENR issued writs of execution
pursuant to the President's decision.

Pascual moved to dismiss the complaint for failure to state cause of action. Saying that the
petitioner had no legal right to file the complaint since the final and executory Bureau of
Lands’ decision ruled that the petitioner was not entitled to possess the lots.

RTC's Ruling: denied the motion to dismiss, Bureau of Lands was not yet final since
President's ruling on the appeal was unavailable. Respondent elevated this case to the
CA, questioning the propriety of the RTC's denial of the motion to dismiss.

CA's Ruling: CA set aside the RTC, the RTC should have dismissed the complaint for
prematurity.

ISSUE

Whether CA committed a reversible error in finding that RTC committed a grave abuse of
discretion in not dismissing the complaint.

RULING

Petition lacks merit, CA committed no reversible error.

A cause of action is the act or omission by which a party violates a right of another.

A complaint states a cause of action when it contains three essential elements: (1) a right
in favor of the plaintiff by whatever means and whatever law it arises; (2) the correlative
obligation of the defendant to respect such right; and (3) the act or omission of the
defendant violates the right of the plaintiff. If any of these elements is absent, the complaint
becomes vulnerable to a motion to dismiss on the ground of failure to state a cause of
action.

"Failure to state a cause of action refers to the insufficiency of allegation in the pleading. In
resolving a motion to dismiss based on the failure to state a cause of action only the facts
alleged in the complaint must be considered. The test is whether the court can render a
valid judgment on the complaint based on the facts alleged and the prayer asked for."

From these allegations, we find it clear that the petitioner no longer had any legal or
equitable title to or interest in the lots. The petitioner’s status as possessor and owner of
the lots had been settled in the final and executory December 4, 1985 decision of the
Bureau of Lands that the DENR Secretary and the OP affirmed on appeal. Thus, the
petitioner is not entitled to the possession and ownership of the lots.

66 | P a g e
MABELLE J. ARELLANO PROPERTY
L02 Saturday, 8:00 AM-12:00 PM

VDA.DE AVILES V. CA
264 SCRA 473
G.R. NO. 95748. NOVEMBER 21, 1996

FACTS

Petitioners aver that they are the actual possessors of a parcel of land, bounded on the N
by Camilo Aviles; on the E by Malawa River, on the S by Anastacio Aviles and on the W
by Juana and Apolonio Joaquin. This property is the share of their father, Eduardo Aviles
and brother of the defendant, in the estate of their deceased parents.

Eduardo Aviles was in actual possession property since 1957. In fact, the latter mortgaged
the same with the bank. When the property was inspected by a bank representative,
Eduardo Aviles, in the presence of the boundary owners, namely, defendant Camilo Aviles,
Anastacio Aviles and Juana and Apolonio Joaquin pointed to the inspector the existing
earthen dikes as the boundary limits of the property and nobody objected. When the real
estate mortgage was foreclosed, the property was sold at public auction but this was
redeemed by plaintiffs’ mother and the land was subsequently transferred and declared in
her name. Defendant Camilo Aviles asserted a color of title over the northern portion of the
property by constructing a bamboo fence (thereon) and moving the earthen dikes.

Defendant Camilo Aviles admitted the agreement of partition executed by him and his
brothers, Anastacio and Eduardo. The respective area(s) allotted to them was agreed and
measured before the execution of the agreement. At present, he is only occupying a
smaller than his actual share.

The trial court disposed of the case thus ordering the parties to employ the services of a
Land Surveyor of the Bureau of Lands to relocate and determine the extent and the
boundary limit of the land of the defendant on its southern side in order that the actual area
given to the defendant be determined.

Dissatisfied with the trial court’s decision, petitioners appealed to the respondent appellate
Court. The Court of Appeals affirmed in part the decision of the trial court, reasoning that
a special civil action for quieting of title is not the proper remedy for settling a boundary
dispute, and that petitioners should have instituted an ejectment suit instead.

ISSUE

WON complaint for quieting of title is the proper remedy for settling boundary dispute.

RULING

No. The Supreme Court ruled that Quieting of Title Not Proper Remedy For Settling
Boundary Dispute. Quieting of title is a common law remedy for the removal of any cloud
upon or doubt or uncertainty with respect to title to real property.
The Civil Code authorizes the said remedy in the following language:

“Art. 476. Whenever there is a cloud on title to real property or any interest therein, by
reason of any instrument, record, claim, encumbrance or proceeding which is apparently
valid or effective but is, in truth and in fact, invalid, ineffective, voidable, or unenforceable,
and may be prejudicial to said title, an action may be brought to remove such cloud or to
quiet the title.

An action may also be brought to prevent a cloud from being cast upon a title to real
property of any interest therein."

In fine, to avail the remedy of quieting of title, a plaintiff must show that there is an
instrument, record, claim, encumbrance or proceeding which constitutes or casts a cloud,
doubt, question or shadow upon the owner’s title to or interest in real property.

As correctly held by the respondent Court, both plaintiffs and defendant admitted the
67 | P a g e
MABELLE J. ARELLANO PROPERTY
L02 Saturday, 8:00 AM-12:00 PM

existence of the agreement of partition and in accordance therewith, a fixed area was
allotted to them and that the only controversy is whether these lands were properly
measured. There is no adverse claim by the defendant “which is apparently valid, but is, in
truth and in fact, invalid, ineffective, voidable, or unenforceable” and which constitutes a
cloud thereon.

Corollarily, and equally as clear, the construction of the bamboo fence enclosing the
disputed property and the moving of earthen dikes are not the “clouds” or “doubts” which
can be removed in an action for quieting of title.

68 | P a g e
MABELLE J. ARELLANO PROPERTY
L02 Saturday, 8:00 AM-12:00 PM

ALEJANDRINO V. COURT OF APPEALS


G.R. NO. 114151, SEPT. 17, 1998
295 SCRA 536, 548

FACTS

The late spouses Alejandrino left their six children a lot in Cebu City. Upon the death of the
spouses, the property should have been divided among their children, however, the
estate of the Alejandrino spouses was not settled in accordance with the procedures.
Petitioner Mauricia (one of the children) allegedly purchased portion of the lots from her
brothers, Gregorio’s, Ciriaco’s and Abundio’s share. It turned out, however, that a third
party named Nique, the private respondent in this case, also purchased portions of the
property from Laurencia, Abundio and Marcelino. However, Laurencia (the alleged seller
to Nique) later questioned the sale in an action for quieting of title and damages. The trial
court (Quieting of title case) ruled in favor of Nique and declared him the owner of the lots.
Laurencia appealed the decision to the Court of Appeals but later withdrew the same.
Nique filed a motion for the segregation of the portion of the property that had been
declared by the trial court (Quieting of title case) as his own by virtue of purchase. The trial
court segregated the property on the basis of the Extra-Judicial Settlement between
Mauricia and Laurencia.

ISSUE

WON a co-owner may validly sell specific portions of unpartitioned property to a third party.

RULING

Yes. A co-owner is entitled to sell his undivided share. A sale of the entire property by one
co-owner without the consent of the other co-owners is not null and void. However, only
the rights of the co-owner-seller are transferred, thereby making the buyer a co-owner of
the property.( Bailon-Casilao vs. Court of Appeals)

Article 1078 of the Civil Code provides that where there are two or more heirs, the whole
estate of the decedent is, before partition, owned in common by such heirs, subject to the
payment of the debts of the deceased. Under a co-ownership, the ownership of an
undivided thing or right belongs to different persons. Each co-owner of property which is
held pro indiviso exercises his rights over the whole property and may use and enjoy the
same with no other limitation than that he shall not injure the interests of his co-owners.
The underlying rationale is that until a division is made, the respective share of each cannot
be determined and every co-owner exercises, together with his co-participants, joint
ownership over the pro indiviso property, in addition to his use and enjoyment of the same.

Although the right of an heir over the property of the decedent is inchoate as long as the
estate has not been fully settled and partitioned,[11] the law allows a co-owner to exercise
rights of ownership over such inchoate right. Thus, the Civil Code provides:

ART. 493. Each co-owner shall have the full ownership of his part and of the fruits and
benefits pertaining thereto, and he may therefore alienate, assign or mortgage it, and even
substitute another person in its enjoyment, except when personal rights are involved. But
the effect of the alienation or the mortgage, with respect to the co-owners, shall be limited
to the portion which may be allotted to him in the division upon the termination of the co-
ownership.

With respect to properties shared in common by virtue of inheritance, alienation of a pro


indiviso portion thereof is specifically governed by Article 1088 that provides:

ART. 1088. Should any of the heirs sell his hereditary rights to a stranger before the
partition, any or all of the co-heirs may be subrogated to the rights of the purchaser by
reimbursing him for the price of the sale, provided they do so within the period of one month
from the time they were notified in writing of the sale by the vendor.

69 | P a g e
MABELLE J. ARELLANO PROPERTY
L02 Saturday, 8:00 AM-12:00 PM

In the instant case, Laurencia was within her hereditary rights in selling her pro
indiviso share in Lot No. 2798. However, because the property had not yet been partitioned
in accordance with the Rules of Court, no particular portion of the property could be
identified as yet and delineated as the object of the sale. Thus, interpreting Article 493 of
the Civil Code providing that an alienation of a co-owned property shall be limited to the
portion which may be allotted to (the seller) in the division upon the termination of the co-
ownership, the Court said:

x x x pursuant to this law, a co-owner has the right to alienate his pro-indiviso share in the
co-owned property even without the consent of the other co-owners. Nevertheless, as a
mere part owner, he cannot alienate the shares of the other co-owners.

70 | P a g e
MABELLE J. ARELLANO PROPERTY
L02 Saturday, 8:00 AM-12:00 PM

DE GUIA V. CA
G.R. NO. 120864. OCTOBER 8, 2003
413 SCRA 114, 124 (2003)

CARPIO, J.:

FACTS

On 12 May 1986, ABEJO instituted an action for recovery of possession with damages
against DE GUIA. In his complaint, ABEJO alleged that he is the owner of the undivided
portion of a property used as a fishpond. He alleged ownership over approximately 39,611
square meters out of the FISHPONDs total area of 79,220 square meters. ABEJO further
averred that DE GUIA continues to possess and use the FISHPOND without any contract
and without paying rent to ABEJOs damage and prejudice. ABEJO also complained that
DE GUIA refuses to surrender ownership and possession of the FISHPOND despite
repeated demands to do so after DE GUIAs sublease contract over the FISHPOND had
expired. ABEJO asked the trial court to order DE GUIA to vacate an approximate area of
39,611 square meters as well as pay damages.

The trial court rendered its decision in favor of plaintiff ABEJO and against the defendant
DEGUIA. Aggrieved, DE GUIA went to the Court of Appeals insisting the trial court erred
in ordering him to vacate and surrender possession of the undivided portion of the
FISHPOND. The Court of Appeals found DE GUIA’s appeal without merit and affirmed the
trial court’s decision. Hence, this petition.

ISSUE

Whether or not a co-owner can be excluded from a specific portion of an undivided


property?

RULING

No. Under Article 484 of the Civil Code, there is co-ownership whenever the ownership of
an undivided thing or right belongs to different persons. A co-owner of an undivided parcel
of land is an owner of the whole, and over the whole he exercises the right of dominion,
but he is at the same time the owner of a portion which is truly abstract. On the other hand,
there is no co-ownership when the different portions owned by different people are already
concretely determined and separately identifiable, even if not yet technically described.

Article 487 of the Civil Code provides, any one of the co-owners may bring an action
in ejectment. This article covers all kinds of actions for the recovery of possession. Article
487 includes forcible entry and unlawful detainer (accion interdictal), recovery of
possession (accion publiciana), and recovery of ownership (accion de reivindicacion). The
summary actions of forcible entry and unlawful detainer seek the recovery of physical
possession only. These actions are brought before municipal trial courts within one year
from dispossession. However, accion publiciana, which is a plenary action for recovery of
the right to possess, falls under the jurisdiction of the proper regional trial court when the
dispossession has lasted for more than one year. Accion de reivindicacion, which seeks
the recovery of ownership, also falls under the jurisdiction of the proper regional trial court.

Any co-owner may file an action under Article 487 not only against a third person, but also
against another co-owner who takes exclusive possession and asserts exclusive
ownership of the property. In the latter case, however, the only purpose of the action is to
obtain recognition of the co-ownership. The plaintiff cannot seek exclusion of the defendant
from the property because as co-owner he has a right of possession. The plaintiff cannot
recover any material or determinate part of the property.

Following the inherent and peculiar features of co-ownership, while ABEJO and DE GUIA
have equal shares in the FISHPOND quantitatively speaking, they have the same right in
a qualitative sense as co-owners. Simply stated, ABEJO and DE GUIA are owners of the
71 | P a g e
MABELLE J. ARELLANO PROPERTY
L02 Saturday, 8:00 AM-12:00 PM

whole and over the whole, they exercise the right of dominion. However, they are at the
same time individual owners of a portion, which is truly abstract because until there is
partition, such portion remains indeterminate or unidentified. As co-owners, ABEJO and
DE GUIA may jointly exercise the right of dominion over the entire FISHPOND until they
partition the FISHPOND by identifying or segregating their respective portions.

We rule that a co-owner may file an action for recovery of possession against a co-owner
who takes exclusive possession of the entire co-owned property. However, the only effect
of such action is a recognition of the co-ownership. The courts cannot proceed with the
actual partitioning of the co-owned property. Thus, judicial or extra-judicial partition is
necessary to effect physical division of the FISHPOND between ABEJO and DE GUIA. An
action for partition is also the proper forum for accounting the profits received by DE GUIA
from the FISHPOND. However, as a necessary consequence of such recognition, ABEJO
shall exercise an equal right to possess, use and enjoy the entire FISHPOND.

72 | P a g e
MABELLE J. ARELLANO PROPERTY
L02 Saturday, 8:00 AM-12:00 PM

BAILON-CASILAO V. CA
160 SCRA 738, 745
APRIL 15, 1988

RULING

Rosalia, Gaudencio, Sabina, Bernabe, Nenita and Delia Bailon are co-owners of a parcel
of land with an area of 48,849 square meters, with a 1/6 share each. On August 23, 1948,
Rosalia Bailon and Gaudencio Bailon sold a portion of the said land consisting of 16,283
square meters to Donato Delgado. On May 13, 1949, Rosalia Bailon alone sold the
remainder of the land consisting of 32,566 square meters to Ponciana V. Aresgado de
Lanuza. On the same date, Lanuza acquired from Delgado the 16,283 square meters of
land which the latter had earlier acquired from Rosalia and Gaudencio. On December 3,
1975, John Lanuza, acting under a special power of attorney given by his wife, Ponciana
V. Aresgado de Lanuza, sold the two parcels of land to Celestino Afable, Sr.

The lower court rendered a decision declaring Celestino Afable, a co-owner of the land
described in paragraph III of the complaint having validly bought the two-sixth (2/6)
respective undivided shares of Rosalia Bailon and Gaudencio Bailon. The court also
declaring Sabina, Bernabe, Delia and the heirs of Nebita as pro-indiviso co-owners, having
1/6 share each, of the property described in paragraph III of the complaint.

ISSUE

Whether or not a sale by one or more co-owners of the entire property held in common
without the consent of all the co-owners is valid.

RULING

Yes. The rights of a co-owner of a certain property are clearly specified in Article 493 of the
Civil Code which states that each co-owner shall have the full ownership of his part and of
the acts and benefits pertaining thereto, and he may therefore alienate assign or mortgage
it and even substitute another person in its enjoyment, except when personal rights are
involved. But the effect of the alienation or mortgage, with respect to the co-owners, shall
be limited to the portion which may be allotted to him in the division upon the termination
of the co-ownership.

As early as 1923, this Court has ruled that even if a co-owner sells the whole property as
his, the sale will affect only his own share but not those of the other co-owners who did not
consent to the sale. This is because under the aforementioned codal provision, the sale or
other disposition affects only his undivided share and the transferee gets only what would
correspond to his grantor in the partition of the thing owned in common. Consequently, by
virtue of the sales made by Rosalia and Gaudencio Bailon which are valid with respect to
their proportionate shares, and the subsequent transfers which culminated in the sale to
private respondent Celestino Afable, the said Afable thereby became a co-owner of the
disputed parcel of land.

A co-owner is entitled to sell his undivided share, a sale of the entire property by one co-
owner without the consent of the other co-owners is not null and void. However, only the
rights of the co-owner-seller are transferred, thereby making the buyer a co-owner of the
property.

73 | P a g e
MABELLE J. ARELLANO PROPERTY
L02 Saturday, 8:00 AM-12:00 PM

GAPACAN V. OMIPET
387 SCRA 383
G.R. NO. 148943. AUGUST 15, 2002

BELLOSILLO, J.:

FACTS

Paicat Gapacan is the primitive possessor of an unregistered land in Mt. Province, divided
into 3 parcels of riceland and another planted to camote and declared by him for taxation
purposes. He had two children Maria and Antonio. Antonio left for a long while to try his
luck in the mines Benguet. Maria remained, took care of their father and eventually took
over the cultivation of the land.

Antonio Gapacan returned to and executed an Affidavit of Transfer of Real Property


showing that the property had been transferred to him by his sister Maria Gapacan-Omipet
(Omipet) making him in effect the legal owner of the property in question. Since then,
Antonio Gapacan’s family (Gapacans) had been occupying and cultivating the property.

Sometime in 1992, Omipet hired laborers to clear and cultivate portions of the disputed
property. Gapacans prohibited them Gapacans and ordered the defendants to vacate the
land and restore possession to plaintiffs.

Omipet then filed an action to quiet title in RTC and that she be declared the lawful owner.
RTC adjudged that Gapacans have right of possession over the land. On appeal CA,
declared that the land is common property of both Omipet and Gapacans and ordered its
partition. Both parties appealed.

ISSUE

Whether or not the petitioner and defendant are co-owners of the disputed property.

RULING

Yes. Although the evidence preponderates in favor of Antonio Gapacan and subsequently
his heirs upon his death and it has been clearly established that Antonio and his family had
been in possession of the subject realty since 1971, Antonio could not honestly claim the
rights of a possessor in good faith since his tax declarations, and more so, his Affidavit of
Transfer of Real Property, were either spurious or founded on false and unlawful claims.
The parcels of land in question, as part of the hereditaments of Paicat, a common ancestor
of Maria and Antonio, were given to neither of them in particular.

It is difficult to believe that Maria and Antonio were blissfully ignorant of their respective
legal rights over the disputed realty. As the two (2) surviving heirs of the Paicat Gapacan,
neither Maria nor Antonio can claim absolute ownership over the entire property to the
prejudice of the other, for each, in legal contemplation, is entitled to only one-half (1/2) pro-
indiviso share of his or her father's estate. Prior to partition, Maria and Antonio, and upon
the latter's death, the petitioners, hold the disputed property in their capacity as co-owners.

The juridical concept of co-ownership is unity of the object or property and plurality of
subjects. Each co-owner, jointly with the other co-owners, is the owner of the whole
property, but at the same time of the undivided aliquot part. Each co-owner has the right to
sell, assign or dispose of his share, unless personal rights are involved. He may also lose
such rights to others, as by prescription thereof by a co-owner.

74 | P a g e
MABELLE J. ARELLANO PROPERTY
L02 Saturday, 8:00 AM-12:00 PM

ADILLE V. COURT OF APPEALS


157 SCRA 455
G.R. NO. L-44546 JANUARY 29, 1988

SARMIENTO, J.

FACTS

Felisa Alzul owned a lot with an area of some 11,325 sq. m. She married twice in her
lifetime; the first, with one Bernabe Adille, with whom she had as an only child, herein
defendant Rustico Adille; in her second marriage with one Procopio Asejo, her children
were herein plaintiffs, — now, sometime in 1939, said Felisa sold the property in pacto de
retro to certain 3rd persons, period of repurchase being 3 years, but she died in 1942
without being able to redeem and after her death, but during the period of redemption,
herein defendant repurchased, by himself alone, and after that, he executed a deed of
extra-judicial partition representing himself to be the only heir and child of his mother Felisa
with the consequence that he was able to secure title in his name alone also, so that OCT.
No. 21137 in the name of his mother was transferred to his name, that was in 1955.

ISSUE

Whether or not a co-owner may acquire exclusive ownership over the property held in
common.

RULING

No. While the records show that the petitioner redeemed the property in its entirety,
shouldering the expenses therefore, that did not make him the owner of all of it. In other
words, it did not put to end the existing state of co-ownership.

The property remains to be in a condition of co-ownership. While a vendee a retro, under


Article 1613 of the Code, "may not be compelled to consent to a partial redemption," the
redemption by one co-heir or co-owner of the property in its totality does not vest in him
ownership over it. Failure on the part of all the co-owners to redeem it entitles the vendee
a retro to retain the property and consolidate title thereto in his name. But the provision
does not give to the redeeming co-owner the right to the entire property. It does not provide
for a mode of terminating a co-ownership.

Neither does the fact that the petitioner had succeeded in securing title over the parcel in
his name terminate the existing co-ownership. While his half-brothers and sisters are liable
to him for reimbursement as and for their shares in redemption expenses, he cannot claim
exclusive right to the property owned in common. Registration of property is not a means
of acquiring ownership. It operates as a mere notice of existing title, that is, if there is one.

75 | P a g e
MABELLE J. ARELLANO PROPERTY
L02 Saturday, 8:00 AM-12:00 PM

SANCHEZ V. COURT OF APPEALS


404 SCRA 541, 548,
G.R. NO. 152766 JUNE 20, 2003

BELLOSILLO, J.

FACTS

Lilia Sanchez, petitioner, constructed a house on a lot owned by her parents-in-law. The
lot was registered under TCT together with her co-owners. The lot was registered under
TCT in the name of private respondent Virginia Teria by virtue of a Deed of Absolute Sale
by all six (6) co-owners in her favor. Petitioner claimed that she did not affix her signature
on the document and subsequently refused to vacate the lot, thus prompting private
respondent to file an action for recovery of possession of the aforesaid lot with the
Metropolitan Trial Court (MeTC) of Caloocan City. The MeTC of Caloocan City ruled in
favor of private respondent declaring that the sale was valid only to the extent of 5/6 of the
lot and the other 1/6 remaining as the property of petitioner, on account of her signature in
the Deed of Absolute Sale having been established as a forgery.

Petitioner then elevated her appeal to the Regional Trial Court of Caloocan City, which
ordered the parties to file their respective memoranda of appeal. Counsel for petitioner did
not comply with this order, nor even inform her of the developments in her case. Petitioner
not having filed any pleading with the RTC of Caloocan City, the trial court affirmed decision
of the MeTC.

The MeTC issued an order for the issuance of a writ of execution in favor of private
respondent. On November 4, 1999, a Notice to Vacate was served by the sheriff upon
petitioner who however refused to heed the Notice. Private respondent started demolishing
petitioner’s house without any special permit of demolition from the court. Due to the
demolition of her house, petitioner filed her Petition for Relief from Judgment with the RTC
on the ground that she was not bound by the inaction of her counsel who failed to submit
petitioners appeal memorandum. However the RTC denied the Petition and the
subsequent Motion for Reconsideration.
Petitioner filed her petition for Certiorari with the Court of Appeals alleging grave abuse of
discretion on the part of the court a quo. The appellate court dismissed the petition for lack
of merit. Petitioner then filed a Motion for Reconsideration but the Court of Appeals denied
the motion.

ISSUE

Whether or not the Court of Appeals committed grave abuse of discretion in dismissing the
challenged case before it.

RULING

As a matter of policy, the original jurisdiction of the Supreme Court to issue the so-called
extraordinary writs should generally be exercised relative to actions or proceedings before
the Court of Appeals or before constitutional or other tribunals or agencies the acts of which
for some reason or other are not controllable by the Court of Appeals.

Where the issuance of the extraordinary writ is also within the competence of the Court of
Appeals or the Regional Trial Court, it is either of these courts that the specific action for
the procurement of the writ must be presented.

The Rules of Court should be liberally construed in order to promote their object of securing
a just, speedy and inexpensive disposition of every action or proceeding. The rules of
procedure should be viewed as mere tools designed to aid the courts in the speedy, just
and inexpensive determination of the cases before them. Liberal construction of the rules
and the pleadings is the controlling principle to effect substantial justice.

76 | P a g e
MABELLE J. ARELLANO PROPERTY
L02 Saturday, 8:00 AM-12:00 PM

Verily, the negligence of petitioners counsel cannot be deemed as negligence of petitioner


herself. A notice to a lawyer who appears to have been unconscionably irresponsible
cannot be considered as notice to his client. Under the peculiar circumstances of this case,
it appears from the records that counsel was negligent in not adequately protecting his
client’s interest, which necessarily calls for a liberal construction of the Rules.

This case overlooks a basic yet significant principle of civil law: co-ownership. Throughout
the proceedings from the MeTC to the Court of Appeals, the notion of co-ownership was
not sufficiently dealt with. Certiorari should therefore be granted to cure this grave abuse
of discretion. In co-ownership, the relationship of such co-owner to the other co-owners is
fiduciary in character and attribute. Whether established by law or by agreement of the co-
owners, the property or thing held pro-indiviso is impressed with a fiducial nature so that
each co-owner becomes a trustee for the benefit of his co-owners and he may not do any
act prejudicial to the interest of his co-owners.

Before the partition of a land or thing held in common, no individual or co-owner can claim
title to any definite portion thereof. All that the co-owner has is an ideal or abstract quota
or proportionate share in the entire land or thing. Although assigned an aliquot but abstract
part of the property, the metes and bounds of petitioner’s lot has not been designated. As
she was not a party to the Deed of Absolute Sale voluntarily entered into by the other co-
owners, her right to 1/6 of the property must be respected. Partition needs to be effected
to protect her right to her definite share and determine the boundaries of her property. Such
partition must be done without prejudice to the rights of private respondent as buyer of the
5/6 portion of the lot under dispute.

The Petition is GRANTED. The Decision of the Court of Appeals is ANNULLED and SET
ASIDE.

77 | P a g e
MABELLE J. ARELLANO PROPERTY
L02 Saturday, 8:00 AM-12:00 PM

PANGAN V. COURT OF APPEALS


166 SCRA 375, 382
OCT. 17, 1988

FACTS

The petitioners filed an application for the registration of the land situated in San Pascual,
Obando, Bulacan in their names by virtue of their continuous and exclusive possession
thereof since 1895, by themselves and their father and grandfather before them. The trial
court issued an order of general default, there being no opposition to the application, and
proceeded to hear the evidence of the applicants ex-parte. The application was then
approved. The herein private respondent filed a petition to set aside the said decision,
which the trial Court granted, admitting at the same time her opposition to the application
and setting the case for reception of her evidence. This evidence sought to show that the
land was inherited by Leon Hilario's three children, but the son, waived his right thereto
and thereby made his two sisters, Silvestra and Catalina, its exclusive co-owners. As
Catalina's daughter, she was entitled to one-half of the property, the other half going to
Silvestra's heirs, the petitioners herein and the latter's grandchildren.

The trial court judge issued an order dismissing the opposition and reinstating his original
order stating that whatever rights the private respondent Teodora might have had over the
property had been forfeited by extinctive prescription because she had left the land in 1942
and had not since then asserted any claim thereto until 1966. On appeal to the court of
appeals, the decision of the lower court was reversed on the ground that the appellees had
not clearly proved that they had acquired the property by prescription. Hence, the appellant
was entitled to one-half of the property as heir.

ISSUE

1. Whether or not the court of appeals erred in holding that the private respondent
was entitled to one-half of the land.
2. Whether or not Teodora Garcia, by her failure to assert her right, allowed the
statutory period to lapse, thus enabling the petitioners to perfect their claim of
ownership by acquisitive prescription and so exclude her from her share in the
subject property.

HELD

1. No. The court of appeals is not necessarily bound by the factual findings of the trial
court. It may still reverse the findings of the trial court judge of fact if they are not
based on the evidence submitted or have been reached without considering the
other matters of record that might have dictated a different conclusion.

When, therefore, the respondent court accepted the private respondent's allegation that
the land was inherited by the parties from their common ancestor, such a finding, based
on the record and not rejected but even assumed by the trial court, did not constitute grave
abuse of discretion. And when, on the strength of this finding, it then held that an implied
trust was created between the petitioners who were in possession of the land, and Teodora
Garcia, their aunt and co-heir is not an arbitrary assumption. In fact, the Court feels this is
the more plausible relationship between the parties, compared to the version offered by
the petitioners who claim they acquired the property from their grandfather through their
father, who apparently acquired it from his mother. It does not appear that they have pre-
empted the other heirs to the property through any other mode of acquisition. They have
not submitted any evidence of how they acquired the land from their great grandfather,
confining themselves to the assertion that they have continued his original possession. If
this be their theory, then they unavoidably must recognize Teodora Garcia's own claim to
the subject property as she too was an heir, being the daughter of Catalina, who was also
a daughter of Hilario.

2. It is a settled rule that possession by one co-owner will not be regarded as adverse
to the other co-owners but in fact as beneficial to all of them. Hence, as long as his
78 | P a g e
MABELLE J. ARELLANO PROPERTY
L02 Saturday, 8:00 AM-12:00 PM

co-ownership is recognized, an action to compel partition will not prescribe and may
be filed at any time against the actual possessor by any of the other co-owners.
However, if the co-owner actually holding the property asserts exclusive dominion
over it against the other co-owners, the corollary of the rule is that he can acquire
sole title to it after the lapse of the prescribed prescriptive period.

The established evidence clearly shows that the subject land was inherited by the
petitioners and the private respondent as co-heirs, whose possession they continued to
acquire prescriptive title over the property. That possession was originally in the name of
all the heirs, including Teodora Garcia. The petitioners have not proved that their
possession excluded their co-owner and aunt. The petitioners appear to have arrogated
the entire property to themselves upon their father's death when they sought to register the
land in their names to the exclusion of Teodora Garcia. Manifestly, the petitioners have
acted in bad faith in denying their aunt and co-heir her legal share to the property they had
all inherited. The decision of the Court of Appeals has been AFFIRMED that there was no
adequate notice by the petitioners to the private respondent of the rejection of her claim to
her share in the subject property.

79 | P a g e
MABELLE J. ARELLANO PROPERTY
L02 Saturday, 8:00 AM-12:00 PM

HEIRS OF FLORES RESTAR V. HEIRS OF DOLORES R. CICHON


475 SCRA 731
NOV. 22, 2005

FACTS

Emilio Restar died intestate, leaving eight children-compulsory heirs. Restar’s eldest child,
Flores, on the basis of a Joint Affidavit he executed with Helen Restar, caused
the cancellation of Tax Declaration in Emilio Restar’s name. The same covers a 5,918
square meter parcel of land in Aklan which was among the properties left by Restar. Flores
thereafter sought the issuance of another Tax Declaration in his name. Flores later on died.

Ten years later, the heirs of Flores’ sisters, Dolores R. Cichon, et. al. (Heirs of Cichon) filed
a Complaint against Flores’ heirs for “partition of the lot, declaration of nullity of documents,
ownership with damages and preliminary injunction” before the Regional Trial Court (RTC)
of Aklan alleging that the widow Esmenia appealed to them to allow her to hold on to the
lot to finance the education of her children, to which they agreed on the condition that after
the children had finished their education, it would be divided into eight equal parts; and
upon their demand for partition of the lot, the defendants Flores‘ heirs refused, they
claiming that they were the lawful owners thereof as they had inherited it from Flores.
Flores‘ heirs claimed that they had been in possession of the lot in the concept of owner
for more than thirty (30) years and have been paying realty taxes since time immemorial.
And they denied having shared with the plaintiffs the produce of the lot or that upon Flores’
death in 1989, Esmenia requested the plaintiffs to allow her to hold on to it to finance her
children’s education, they contending that by 1977, the children had already finished their
respective courses.

The RTC of Kalibo, Aklan held that Flores and his heirs had performed acts sufficient to
constitute repudiation of the co-ownership, concluded that they had acquired the lot by
prescription. The Court of Appeals reversed the decision finding that there was no
adequate notice by Flores to his co-heirs of the repudiation of the co-ownership and neither
was there a categorical assertion by the defendants of their exclusive right to the entire lot
that barred the plaintiffs’ claim of ownership.

ISSUE

Whether or not Heirs of Flores acquired ownership over the lot by extraordinary
prescription.

RULING

Yes. Acquisitive prescription of dominion and other real rights may be ordinary or
extraordinary. Ordinary acquisitive prescription requires possession of things in good faith
and with just title for a period of ten years. Without good faith and just title, acquisitive
prescription can only be extraordinary in character which requires
uninterrupted adverse possession for thirty years.

When Restar died in 1935, his eight children became pro indiviso co-owners of the lot by
intestate succession. Heirs of Chichon never possessed the lot, however, much less
asserted their claim thereto until January 21, 1999 when they filed the complaint for
partition subject of the present petition. In contrast, Flores took possession of the lot after
Restar’s death and exercised acts of dominion thereon — tilling and cultivating the land,
introducing improvements, and enjoying the produce thereof. Flores’ possession thus
ripened into ownership through acquisitive prescription after the lapse of thirty years in
accordance with the earlier quoted Article 1137 of the New Civil Code.

Heirs of Cichon did not deny that aside from the verbal partition of one parcel of land in
Carugdog, Lezo, Aklan way back in 1945, they also had an amicable partition of the lands
of Emilio Restar in Cerrudo and Palale, Banga Aklan on September 28, 1973. Indeed, the
following acts of Flores show possession adverse to his co-heirs: (1) the cancellation of
80 | P a g e
MABELLE J. ARELLANO PROPERTY
L02 Saturday, 8:00 AM-12:00 PM

the tax declaration certificate in the name of Restar and securing another in his name; (2)
the execution of a Joint Affidavit stating that he is the owner and possessor thereof to the
exclusion of respondents; (3) payment of real estate tax and irrigation fees without
respondents having ever contributed any share therein; and (4) continued enjoyment of the
property and its produce to the exclusion of respondents. And Flores’ adverse possession
was continued by his heirs.

The trial court’s finding and conclusion that Flores and his heirs had for more than 38 years
possessed the land in open, adverse and continuous possession in the concept of owner
— which length of possession had never been questioned, rebutted or disputed by any of
the heirs of Cichon, being thus duly supported by substantial evidence, he and his heirs
have become owner of the lot by extraordinary prescription. It is unfortunate that
respondents slept on their rights.

81 | P a g e
MABELLE J. ARELLANO PROPERTY
L02 Saturday, 8:00 AM-12:00 PM

DELIMA V. COURT OF APPEALS


201 SCRA 641, SEPT. 24, 1991

MEDIALDEA, J.:

FACTS

Lino Delima acquired Lot. No. 7758 of the Talisay-Minglanilla Friar Lands Estate in Cebu
by sale on installments from the government. After his demise in 1921 he had his three
brothers and a sister listed as his heirs. The heirs were Eulalio Delima, Juanita Delima,
Galileo Delima, and Vicente Delima. Anew Transfer Certificate of Title was issued in the
name of the Legal Heirs of Lino Delima represented by Galileo Delima. On September 22,
1953, Galileo executed an affidavit of Extra-judicial Declaration of Heirs adjudicating to
himself the subject property excluding the other heirs. He declared the lot to be of his own
and paid for its taxes. On February 29, 1968, the surviving heirs of Eulalio and Juanita
Delima,filed with the Court of First Instance of Cebu an action for conveyance and partition
of property and for the annulment of the certificate of title issued plus damages against
their Uncle Galileo. Vicente Delima was also later included as party defendant for his
refusal to help in the action.

The trial court decided in favor of the petitioners rendering the TCT No. 3009 null and void
and declaring Vicente, the Heirs of Juanita, the Heirs of Eulalio and the Heirs of Galileo to
be owners of the property, each sharing a pro-indiviso share of one-fourth of the whole.
The respondents, Heirs of Galileo Delima, appealed to the Court of Appeals which reversed
the decision in their favor. It upheld the claim of Galileo that the other brothers and sisters
have already waived their rights to the property being that it was Galileo alone that paid for
the balance of the purchase price and the realty taxes for the property.

ISSUE

Whether or not petitioners’ action for partition is already barred by the statutory period
provided by law which shall enable Galileo Delima to perfect his claim of ownership by
acquisitive prescription to the exclusion of petitioners from their shared in the disputed
property?

RULING

Yes, Article 494 (5) of the Civil Code provides that: “No prescription shall run in favor of a
co-owner or co-heir against his co-owners or co-heirs so long as he expressly or impliedly
recognized the co-ownership”.

By this it is therefore understood that possession by a co-owner will not be presumed to be


adverse to the others, but will be held to benefit all. Being that Galileo was holding the
property in representation of the co-owners; he was therefore acting as an administrator
who took care of the property yet still having the ultimate obligation to deliver the property
to his co-owners.

However this rule shall no longer apply when one of the co-owners begin to claim the
absolute and exclusive ownership and denies the others any share therein. The
imprescriptability of the action for partition shall no longer apply since Galileo is adversely
claiming lone ownership over the property. In order that a possession be considered
adverse amounting to a repudiation of the co-ownership, the following elements must
concur: (1) that the trustee has performed the unequivocal acts amounting to a repudiation
of ownership ,the following must concur: 1) that the trustee has performed the unequivocal
acts amounting to ouster of cestui cui trust;2) that such positive acts of repudiation has
made known to the cestui cui trust and 3) that the evidence thereon is should be clear and
conclusive.

Since Galileo, having executed a deed of partition and obtained subsequent to that the
82 | P a g e
MABELLE J. ARELLANO PROPERTY
L02 Saturday, 8:00 AM-12:00 PM

cancellation of the old title and the creation of a new one wherein he appears as the new
owner of the property, he thereby in effect denied and repudiated the ownership of the
other co-owners over their shares. From this act, the statute of limitations started to run.
Since an action for conveyance.

Doctrine:

No prescription shall run in favor of a co-owner against his co-owners or co-heirs as long
as he expressly or impliedly recognizes the co-ownership. The exception however is that
the from the moment one of the co-owners claims that he is the absolute and exclusive
owner of the properties and denies the others any share therein, the question is no longer
one of partition but of ownership

83 | P a g e
MABELLE J. ARELLANO PROPERTY
L02 Saturday, 8:00 AM-12:00 PM

MARITEGUI V. COURT OF APPEALS


205 SCRA 337

BIDIN, J.:

FACTS

Lupo Mariategui died without a will on June 26, 1953 and contracted 3 marriages during
his lifetime. He acquired the Muntinlupa Estate while he was still a bachelor. He had 4
children with his first wife Eusebia Montellano, who died in 1904 namely Baldomera, Maria
del Rosario, Urbano and Ireneo. Baldomera had 7 children namely Antero, Rufina,
Catalino, Maria, Gerardo, Virginia and Federico, all surnamed Espina. Ireneo on the other
hand had a son named Ruperto. On the other hand, Lupo’s second wife is Flaviana
Montellano where they had a daughter named Cresenciana. Lupo got married for the third
time in 1930 with Felipa Velasco and had 3 children namely Jacinto, Julian and
Paulina. Jacinto testified that his parents got married before a Justice of the Peace of
Taguig Rizal. The spouses deported themselves as husband and wife, and were known in
the community to be such.

Lupo’s descendants by his first and second marriages executed a deed of extrajudicial
partition whereby they adjudicated themselves Lot NO. 163 of the Muntinlupa Estate and
was subjected to a voluntary registration proceedings and a decree ordering the
registration of the lot was issued. The siblings in the third marriage prayed for inclusion in
the partition of the estate of their deceased father and annulment of the deed of extrajudicial
partition dated Dec. 1967.

ISSUE

Whether the marriage of Lupo with Felipa is valid in the absence of a marriage license.

RULING

Although no marriage certificate was introduced to prove Lupo and Felipa’s marriage, no
evidence was likewise offered to controvert these facts. Moreover, the mere fact that no
record of the marriage exists does not invalidate the marriage, provided all requisites for
its validity are present.

Under these circumstances, a marriage may be presumed to have taken place between
Lupo and Felipa. The laws presume that a man and a woman, deporting themselves as
husband and wife, have entered into a lawful contract of marriage; that a child born in lawful
wedlock, there being no divorce, absolute or from bed and board is legitimate; and that
things have happened according to the ordinary course of nature and the ordinary habits
of life.

Hence, Felipa’s children are legitimate and therefore have successional rights.

84 | P a g e
MABELLE J. ARELLANO PROPERTY
L02 Saturday, 8:00 AM-12:00 PM

LACBAYAN V. SAMOY
G.R. NO. 165427, MAR. 21, 2011

VILLARAMA, JR., J.:

FACTS

During Bettys Lacbayan and Bayani Samoy’s illicit relationship, they, got with three more
incorporators, were able to establish a manpower services company, by which they
acquired Five (5) parcels of land, registered in their names, ostensibly as husband and
wife. Having parted ways eventually, both of them agreed to divide the said properties and
terminate their business partnership by executing Partition Agreement. Initially Samoy
agreed to Lacbayan’s proposal that the properties in Malvar St and Don Enrique Heights
be assigned to the latter, while the ownership over the three properties will go to Samoy.
How when Lacbayan wanted additional demands, Samoy refused. Thus, Lacbayan filed a
complaint for judicial partition of the said properties before the Quezon City RTC. In his
answer Howeverr, Samoy denied Lacbayan’s claim of cohabitation and said that the
properties were acquired out of his personal funds without any contribution from her.

ISSUE

Does an action for partition preclude a settlement on the issue of ownership?

RULING

No. While it is true that the complaint involved her is one for partition, the same is premised
on the existence or non-existence of co-ownership between the parties. Petitioner insists
that is a co-owned pro indiviso of the five real estate properties based on the TCTs covering
the subject properties. Respondents maintains otherwise. Indubitable, therefore, until and
unless the issue of co-ownership is definitely finally resolved, it would premature to effect
a partition of the disputed properties.

More importantly, the complaint will not even if the claimant of petitioner in this case does
not even have any rightful interest over the subject properties. A careful perusal of the
contents of the so-called Partition Agreement indicates that the document involves matter
which necessitate prior settlement of questions of basic of which is a determination as to
whether the parties have the rights to freely divide among themselves the subject
properties.

Moreover, to follow petitioner’s argument would be to allow respondent not only to admit
against his own interest but that of his legal spouses as well as who may also be lawful
entitled co- ownership over the said properties. Respondent is not allowed by law to waive
whatever share his lawful spouse may have on the disputed properties. Petitioner herself
admitted that she did not assent to the Partition Agreement after seeing the need to amend
the same to include other matters. Petitioner does not have any right to insist on the
contents of an agreement she intentionally refused to sign.

85 | P a g e
MABELLE J. ARELLANO PROPERTY
L02 Saturday, 8:00 AM-12:00 PM

CRUZ V. CATAPANG
G.R. 164110, FEB. 12, 2008

QUISUMBING, J.:

FACTS

Petitioners Leonor Cruz, Luz Cruz and Norma Maligaya are the co-owners of a parcel of
land covering an area of 1,435 square meters located at Barangay Mahabang Ludlod, Taal,
Batangas. Sometime in 1992, Teofila Catapang, with the consent of Norma Maligaya as
one of the aforementioned co-owners, built a house on a lot adjacent to the subject parcel
of land. The house built by Catapang intruded on a portion of the co-owned property.In
September 1995, Cruz learned about the intrusion and made several demandsfor
Catapang to demolish and vacate the part of the structure encroaching upon their property.

However, Catapang refused and disregarded the demands of Cruz. Cruz then filed a
complaint for forcible entry against Catapang before the MCTC of Taal, Batangas. The
MCTC decided in favor of Cruz, ruling that consent of only one of the co-owners is not
sufficient to justify defendant’s construction of the house and possession of the portion of
the lot in question.

On appeal, the RTC affirmed the decision of the MCTC. Catapang filed a petition for review
with the Court of Appeals, which reversed the RTC’s decision and ruled in favor of her. The
Court of Appeals held that there is no cause of action for forcible entry in this case because
respondent’s entry into the property, considering the consent given by co-owner Norma
Maligaya, cannot be characterized as one made through strategy or stealth which gives
rise to a cause of action for forcible entry. Thus, the case went to the Supreme Court.

ISSUE

Whether the consent given by one of the co-owners is sufficient to warrant the dismissal of
a complaint for forcible entry.

RULING

No, Co-owners cannot devote common property to his or her exclusive use to the prejudice
of the co-ownership. In this case, the act of Norma Maligaya is tantamount to devoting the
property to her exclusive use. Under Article 491 of the Civil Code, none of the co-owners
shall, without the consent of the others, make alterations in the thing owned in common.

The Court ruled that it would necessarily follow that none of the co-owners can, without the
consent of the other co-owners, validly give consent to the making of an alteration by
another person, such as Catapang in this case, in the thing owned in common. In addition,
Article 486 of the same Code states each co-owner may use the thing owned in common
provided he does so in accordance with the purpose for which it is intended and in such a
way as not to injure the interest of the co-ownership or prevent the other co-owners from
using it according to their rights.

The Court ruled that, to give consent to a third person to construct a house on the co-
owned property would be to injure the interest of the co-ownership and would prevent other
co-owners from using the property in accordance with their rights. In this case, the consent
of only one co-owner will not warrant the dismissal of the complaint for forcible entry filed
against the respondent Catapang. The consent given by Norma Maligaya in the absence
of the consent of her other co-owners did not grant Catapang any right to enter and even
build upon the co-owned property.

According to the Supreme Court, the respondent Catapang’s act of getting only the consent
of one co-owner, her sister Norma Maligaya, and allowing the latter to stay in the
constructed house, can in fact be considered as a strategy which she utilized in order to
enter into the co-owned property. As such, respondent’s acts constitute forcible entry.

86 | P a g e
MABELLE J. ARELLANO PROPERTY
L02 Saturday, 8:00 AM-12:00 PM

SANTOS V. HEIRS OF LUSTRE


G.R. NO. 151016, AUG. 06, 2008
NACHURA, J.:

FACTS

Lustre owned a lot which she mortgaged & later on sold to Natividad Santos who
subsequently sold it to her son Froilan for which a TCT was issued in his name. Lustre’s
heirs Macaspac & Maniquiz filed w/ RTC of Gapan, Nueva Ecija a Complaint for
Declaration of the Inexistence of Contract, Annulment of Title, Reconveyance and
Damages against Froilan Santos. Lustre’s other heirs filed a Complaint for Annulment
of Transfer Certificate of Title and Deed of Absolute Sale against spouses Santos, Froilan
Santos, R Transport Corp, Cecilia Macaspac with the same RTC. Macaspac was
impleaded as defendant in the 2nd case because she refused to join the other heirs as
plaintiffs. Alleging that the plaintiffs’ right of action for annulment of the Deed of Sale and
TCT had long prescribed and was barred by laches, petitioners filed a Motion to Dismiss,
also on the ground of litis pendentia. The RTC denied the Motion to Dismiss. They then
filed a petition for certiorari with the Court of Appeals (CA) which dismissed the petition for
lack of merit.

ISSUES

1. Was there forum shopping?


2. Does prescription or laches apply?

RULING

Issue No.1:

No. Forum shopping exists when the elements of litis pendentiaare present or when a final
judgment in one case willamount to res judicata in the other. Its elements are identityof the
subject matter, identity of the causes of action andidentity of the parties in the two cases.
There is substantialidentity of parties when there is a community of interest between a party
in the first case and a party in the second case. There is no forum shopping because there
is no identity of parties because the plaintiff in the 1st case (Macaspac) does not, in fact,
share a common interest with the plaintiffs in the 2nd case.

Plaintiffs in both cases are the heirs of Lustre; they are therefore co-owners of the
property. However, the fact of being a co-owner does not necessarily mean that a
plaintiff is acting for the benefit of the co-ownership when he files an action respecting the
co-owned property. Co-owners are not parties inter se in relation to the property owned in
common. The test is whether the “additional” party, the co-owner in this case, acts in the
same capacity or is in privity with the parties in the former action. Macaspac filed the 1st
case seeking the reconveyance of the property to her, and not to Lustre or her heirs. This
is a clear act of repudiation of the co-ownership which would negate a conclusion that she
acted in privity with the other heirs or that she filed the complaint in behalf of the co-
ownership. In contrast, respondents were evidently acting for the benefit of the co-
ownership when they filed the 2nd case wherein they prayed that TCT Lustre be reinstated,
or a new certificate of title be issued in her name.

Issue No. 2:

The action for reconveyance on the ground that the certificate of title was obtained by
means of a fictitious deed of sale is virtually an action for the declaration of its nullity, which
does not prescribe. Moreover, a person acquiring property through fraud becomes, by
operation of law, a trustee of an implied trust for the benefit of the real owner of the
property. An action for reconveyance based on an implied trust prescribes in ten
years. And in such case, the prescriptive period applies only if there is an actual need to
reconvey the property as when the plaintiff is not in possession of the property.

Otherwise, if plaintiff is in possession of the property, prescription does not commence to


87 | P a g e
MABELLE J. ARELLANO PROPERTY
L02 Saturday, 8:00 AM-12:00 PM

run against him. Thus, when an action for reconveyance is nonetheless filed, it would be
in the nature of a suit for quieting of title, an action that is imprescriptible. It follows then
that the respondents’ present action should not be barred by laches. Laches is a doctrine
in equity, which may be used only in the absence of, and never against, statutory
law. Obviously, it cannot be set up to resist the enforcement of an imprescriptible legal
right.

88 | P a g e
MABELLE J. ARELLANO PROPERTY
L02 Saturday, 8:00 AM-12:00 PM

89 | P a g e
MABELLE J. ARELLANO PROPERTY
L02 Saturday, 8:00 AM-12:00 PM

PNB V. DE JESUS
G.R. NO. 149295, SEPTEMBER 23, 2003

VITUG, J.:

FACTS

 Respondent filed a complaint against petitioner before the Regional Trial Court of
Occidental Mindoro for recovery of ownership and possession, with damages, over
the questioned property. In his complaint, respondent stated that he had acquired
a parcel of land situated in Mamburao, Occidental Mindoro, with an area of 1,144
square meters covered by TCT No. T-17197, and that on 26 March 1993, he had
caused a verification survey of the property and discovered that the northern portion
of the lot was being encroached upon by a building of petitioner to the extent of 124
square meters. Despite two letters of demand sent by respondent, petitioner failed
and refused to vacate the area.

 Petitioner, in its answer, asserted that when it acquired the lot and the building
sometime in 1981 from then Mayor Bienvenido Ignacio, the encroachment already
was in existence and to remedy the situation, Mayor Ignacio offered to sell the area
in question (which then also belonged to Ignacio) to petitioner at P100.00 per
square meter which offer the latter claimed to have accepted. The sale, however,
did not materialize when, without the knowledge and consent of petitioner, Mayor
Ignacio later mortgaged the lot to the Development Bank of the Philippines.

 The trial court decided the case in favor of respondent declaring him to be the
rightful owner

ISSUE

Whether or not being a builder in good faith matters under article 448 of the New Civil
Code.

RULING

One is considered in good faith if he is not aware that there exists in his title or
mode of acquisition any flaw which invalidates it.

Article 448, of the Civil Code refers to a piece of land whose ownership is claimed by two
or more parties, one of whom has built some works (or sown or planted something) and
not to a case where the owner of the land is the builder, sower, or planter who then later
loses ownership of the land by sale or otherwise for, elsewise stated, “where the true
owner himself is the builder of works on his own land, the issue of good faith or bad faith
is entirely irrelevant.”

90 | P a g e
MABELLE J. ARELLANO PROPERTY
L02 Saturday, 8:00 AM-12:00 PM

PARILLA V. PILAR
G.R. NO. 167680, NOV. 30, 2006

CARPIO MORALES, J.:

FACTS

A tenant cannot be said to be a builder in good faith as he has no pretension to be owner.


At all events, under the Civil Code, it is the lessor who is given the option, upon termination
of the lease contract, either to appropriate the useful improvements by paying one-half of
their value at that time, or to allow the lessee to remove the improvements.

Spouses Samuel and Chinita Parilla and their son, as dealers of Pilipinas Shell
Petroleum Corporation (Pilipinas Shell), have been in possession of a parcel of land in
Bantay, Ilocos Sur which was leased to them by respondent Dr. Prospero Pilar. When the
lease contract between Pilipinas Shell and Pilar expired, and despite demands to vacate,
the Parillas remained in possession of the property on which they built improvements, the
Parillas and the other occupants remained in the property. Hence, Pilar filed a complaint for
ejectment before the Municipal Trial Court (MTC) of Bantay, Ilocos Sur. The MTC ordered
the Parillas to vacate and to pay Pilar a reasonable compensation for the use of the
property. It also ordered Pilar to reimburse the Parillas the amount Two Million Pesos
representing the value of the improvements introduced on the property. Pilar appealed to
the Regional Trial Court of Vigan and the RTC affirmed the MTC‘s Decision. However, on
Pilar‘s petition for review, the Court of Appeals set aside the lower court’s decision.

ISSUES

Whether or not the Parillas are entitled to reimbursement for the improvements
being builders in good faith.

RULING

One whose interest is merely that of a holder, such as a mere tenant, agent or usufructuary,
is not qualified to become a possessor builder in good faith.

Jurisprudence is replete with cases which categorically declare that Article 448 covers only
cases in which the builders, sowers or planters believe themselves to be owners of the
land or, at least, have a claim of title thereto, but not when the interest is merely that of a
holder, such as a mere tenant, agent or usufructuary. A tenant cannot be said to be a
builder in good faith as he has no pretension to be owner.

The right of the lessor upon the termination of a lease contract with respect to useful
improvements introduced on the leased property by a lessee is covered by Article 1678.
Clearly, it is Article 1678 of the New Civil Code which applies to the present case. The
Parillas claim for reimbursement of the alleged entire value of the improvements does not
thus lie under Article 1678. Not even for one-half of such alleged value, there being no
substantial evidence, e.g., receipts or other documentary evidence detailing costs of
construction. Besides, by the Parillas‘admission, of the structures they originally built —
the billiard hall, restaurant, sari-sari store and a parking lot, only the ―bodega-like‖ sari-
sari store and the parking lot now exist.

At all events, under Article 1678, it is the lessor who is given the option, upon termination
of the lease contract, either to appropriate the useful improvements by paying one-half of
their value at that time, or to allow the lessee to remove the improvements. This option
solely belongs to the lessor as the law is explicit that ―should the lessor refuse to
reimburse said amount, the lessee may remove the improvements, even though the
principal thing may suffer damage thereby.‖ It appears that the lessor has opted not to
reimburse.

91 | P a g e
MABELLE J. ARELLANO PROPERTY
L02 Saturday, 8:00 AM-12:00 PM

ABALOS V. HEIRS OF TORIO


G.R. NO. 175444, DEC.14,2011

PERALTA, J.:

FACTS

On July 24, 1996, herein respondents filed a Complaint for Recovery of Possession and
Damages with the Municipal Trial Court (MTC) of Binmaley, Pangasinan against Jaime
Abalos (Jaime) and the spouses Felix and Consuelo Salazar. Respondents contended
that: they are the children and heirs of one Vicente Torio (Vicente) who died intestate on
September 11, 1973; at the time of the death of Vicente, he left behind a parcel of land
measuring 2,950 square meters, more or less, which is located at San Isidro Norte,
Binmaley, Pangasinan; during the lifetime of Vicente and through his tolerance, Jaime and
the Spouses Salazar were allowed to stay and build their respective houses on the subject
parcel of land; even after the death of Vicente, herein respondents allowed Jaime and the
Spouses Salazar to remain on the disputed lot; however, in 1985, respondents asked
Jaime and the Spouses Salazar to vacate the subject lot, but they refused to heed the
demand of respondents forcing respondents to file the complaint.

Jaime and the Spouses Salazar filed their Answer with Counterclaim, denying the material
allegations in the Complaint and asserting in their Special and Affirmative Defenses that:
respondents' cause of action is barred by acquisitive prescription; the court a quo has no
jurisdiction over the nature of the action and the persons of the defendants; the absolute
and exclusive owners and possessors of the disputed lot are the deceased predecessors
of defendants; defendants and their predecessors-in-interest had been in actual,
continuous and peaceful possession of the subject lot as owners since time immemorial;
defendants are faithfully and religiously paying real property taxes on the disputed lot as
evidenced by Real Property Tax Receipts; they have continuously introduced
improvements on the said land, such as houses, trees and other kinds of ornamental plants
which are in existence up to the time of the filing of their Answer.

On the same date as the filing of defendants' Answer with Counterclaim, herein petitioners
filed their Answer in Intervention with Counterclaim. Like the defendants, herein petitioners
claimed that their predecessors-in-interest were the absolute and exclusive owners of the
land in question; that petitioners and their predecessors had been in possession of the
subject lot since time immemorial up to the present; they have paid real property taxes and
introduced improvements thereon.

The MTC adjudged the case in favor of the plaintiffs and against the defendants and
defendants-intervenors are ordered to turn over the land in question to the plaintiffs. Jaime
and the Spouses Salazar appealed the Decision of the MTC with the RTC of Lingayen,
Pangasinan. Herein petitioners, who were intervenors, did not file an appeal. The RTC
ruled in favor of Jaime and the Spouses Salazar, holding that they have acquired the
subject property through prescription. Accordingly, the RTC dismissed herein respondents'
complaint.

ISSUE

Whether the petitioners and their predecessors-in-interest possessed the disputed lot in
the concept of an owner, or whether their possession is by mere tolerance of respondents
and their predecessors-in-interest.

RULING

Acts of possessory character executed due to license or by mere tolerance of the owner
are inadequate for purposes of acquisitive prescription. Possession, to constitute the
foundation of a prescriptive right, must be en concepto de dueño, or, to use the common
law equivalent of the term, that possession should be adverse, if not, such possessory
acts, no matter how long, do not start the running of the period of prescription.

92 | P a g e
MABELLE J. ARELLANO PROPERTY
L02 Saturday, 8:00 AM-12:00 PM

BUNYI V. FACTOR
G.R. NO. 172547,JUN. 30, 2009
591 SCRA 350

QUISUMBING, J.:

FACTS

Respondent Fe S. Factor is one of the co-owners of an 18-hectare piece of land located in


Almanza, Las Pias City. The ownership of the land originated from respondents paternal
grandparents Constantino Factor and Maura Mayuga-Factor who had been in actual,
continuous, peaceful, public, adverse and exclusive possession and occupation of the land
even before 1906.

On December 9, 1975, the children of Constantino Factor and Maura Mayuga-Factor filed
a Petition for Original Registration and Confirmation of Imperfect Title to the said parcel of
land and on December 8, 1994, the trial court granted the petition and declared the children
of Constantino Factor and Maura Mayuga-Factor as co-owners of the property. The
children thereafter sold seven (7) hectares of the Factor family property during the same
year. The siblings, except Enrique Factor, respondent’s father, shared and divided the
proceeds of the sale among themselves, with the agreement that Enrique would have as
his share the portion of the property located in Antioch Street, Pilar Executive Village,
Almanza I, Las Pias City, known as the Factor compound.
Enrique caused the construction of several houses in the compound including the subject
property, a rest house, where members of the Factor family stayed during get-togethers
and visits. Petitioners Precy Bunyi and her mother, Mila Bunyi, were tenants in one of the
houses inside the compound.

When Enrique Factor died on August 7, 1993, the administration of the Factor compound
including the subject rest house and other residential houses for lease was transferred and
entrusted to Enriques eldest child, Gloria Factor-Labao.

Gloria Factor-Labao, together with her husband Ruben Labao and their son Reggie F.
Labao, lived in Tipaz, Taguig, Metro Manila but visited and sometimes stayed in the rest
house because Gloria collected the rentals of the residential houses and oversaw the
Factor compound.

When Gloria died on January 15, 2001, the administration and management of the Factor
compound including the subject rest house, passed on to respondent Fe S. Factor as co-
owner of the property. As an act of goodwill and compassion, considering that Ruben
Labao was sickly and had no means of income, respondent allowed him to stay at the rest
house for brief, transient and intermittent visits as a guest of the Factor family.

On May 31, 2002, Ruben Labao married petitioner Precy Bunyi. On November 10, 2002,
Ruben Labao died. At about this time, respondent discovered that petitioners forcibly
opened the doors of the rest house and stole all the personal properties owned by the
Factor family and then audaciously occupied the premises. Respondent alleged that
petitioners unlawfully deprived her and the Factor family of the subject propertys lawful use
and possession.

Petitioners, for their part, questioned Fes claim of ownership of the subject property and
the alleged prior ownership of her father Enrique Factor. They asserted that the subject
property was owned by Ruben Labao, and that petitioner Precy with her husband moved
into the subject property.

On July 13, 2004, the Metropolitan Trial Court (MeTC) of Las Pias City, Branch 79 ruled in
favor of Fe S. Factor.

Petitioners appealed the decision to the RTC of Las Pias City, Branch 198, which, however,
affirmed in toto the decision of the MeTC and later denied their motion for reconsideration.
Undaunted, petitioners filed a petition for review before the Court of Appeals but it was

93 | P a g e
MABELLE J. ARELLANO PROPERTY
L02 Saturday, 8:00 AM-12:00 PM

denied also.

ISSUE

Who has the better right of physical and material possession of the subject property?

RULING

The Respondent. In ejectment cases, the only issue for resolution is who is entitled to the
physical or material possession of the property involved, independent of any claim of
ownership set forth by any of the party-litigants. The one who can prove prior
possession de facto may recover such possession even from the owner
himself. Possession de facto is the physical possession of real property. Possession de
facto and not possession de jure is the only issue in a forcible entry case. This rule holds
true regardless of the character of a partys possession, provided, that he has in his favor
priority of time which entitles him to stay on the property until he is lawfully ejected by a
person having a better right by either accion publiciana or accion reivindicatoria.

For one to be considered in possession, one need not have actual or physical occupation
of every square inch of the property at all times. Possession can be acquired not only by
material occupation, but also by the fact that a thing is subject to the action of ones will or
by the proper acts and legal formalities established for acquiring such right. Possession
can be acquired by juridical acts. These are acts to which the law gives the force of acts of
possession. Examples of these are donations, succession, execution and registration of
public instruments, and the inscription of possessory information titles.

While petitioners claim that respondent never physically occupied the subject property,
they failed to prove that they had prior possession of the subject property. On the other
hand, it was established that respondents grandparents, Constantino Factor and Maura
Mayuga-Factor, had been the occupants and in possession of various agricultural parcel
of lands situated in Almanza, Las Pias City, in the concept of owners, for more than thirty
years prior to 1975.

The right of respondents predecessors over the subject property is more than sufficient to
uphold respondents right to possession over the same. Respondents right to the property
was vested in her along with her siblings from the moment of their fathers death. As heir,
respondent had the right to the possession of the property, which is one of the attributes of
ownership. Such rights are enforced and protected from encroachments made or
attempted before the judicial declaration since respondent acquired hereditary rights even
before judicial declaration in testate or intestate proceedings.

After the death of Enrique Factor, it was his eldest child, Gloria Factor-Labao who took
over the administration of the subject property. And as a consequence of co-
ownership, soon after the death of Gloria, respondent, as one of the surviving co-owners,
may be subrogated to the rights of the deceased co-owner, which includes the right to the
administration and management of the subject property.

94 | P a g e
MABELLE J. ARELLANO PROPERTY
L02 Saturday, 8:00 AM-12:00 PM

PEOPLE V. PEÑAFLORIDA
G.R. NO. 175604, APR. 10, 2008

TINGA, J.:

FACTS

SPO3 Vicente Competente received a tip from an asset that a bundle of marijuana was
being transported by appellant to Huyon-huyon from another barangay in Tigaon,
Camarines Sur. (Agravante), chief of police of Tigaon, then organized a team composed
of Competente as team leader, SPO2 Callo, SPO1 Portugal, PO3 Pillos and PO2 Edgar
Latam. The team boarded the police mobile car and proceeded to Barangay Huyon-huyon.
They overtook appellant who was on a bicycle. The police officers flagged appellant down
and found marijuana wrapped in a cellophane and newspaper together with other grocery
items. The amount of P1550.00 was also found in appellant's possession. The police
officers confiscated these items and took photographs thereof. Appellant was then brought
to the headquarters where he was booked.

Major Lorlie Arroyo, a forensic chemist at the PNP Crime Laboratory Regional Office No.
V, was presented as an expert witness to identify the subject marijuana leaves. She related
that after taking a representative sample from the 928-gram confiscated dried leaves, the
same was tested positive of marijuana.

Appellant denied the accusations against him. That on his way home, they met Boyet Obias
who requested appellant to bring a package wrapped in a newspaper to Jimmy Gonzales,
he placed it in the basket in front of his bicycle and Gonzales proceeded to the Tiagon town
proper. On his way home, he was flagged down by the police and was invited to go with
them to the headquarters.

TC ruled that there was violation of Section 4, Article II of Republic Act (R.A.) No. 6425,
otherwise known as The Dangerous Drugs Acts of 1972; hence, the instant case is now
before this Court on automatic review.

In assailing his conviction, appellant submits that there is doubt that he had freely and
consciously possessed marijuana. One of the issues raised is that, upon receipt of the
information from the asset, the police officers should have first investigated and tried to
obtain a warrant of arrest against appellant, instead of arbitrarily arresting him.

ISSUE

Whether there is doubt that accused knowingly possessed the prohibited articles in his
person, or that animus possidendi is shown to be present together with his possession or
control of such article.

RULING

Knowledge refers to a mental state of awareness of a fact. Since courts cannot penetrate
the mind of an accused and thereafter state its perceptions with certainty, resort to other
evidence is necessary. Animus possidendi, as a state of mind, may be determined on a
case-to-case basis by taking into consideration the prior or contemporaneous acts of the
accused, as well as the surrounding circumstances. Its existence may and usually must be
inferred from the attendant events in each particular case.

Appellant failed to satisfactorily establish his lack of knowledge of possession in the instant
case. First, the marijuana was found in the bicycle he himself was driving. Second, the
police officers first readily saw in plain view the edges of the marijuana leaves jutting out
of the package. Third, it is incredulous that appellant did not ask Obias what the package
contained when the latter requested him to do the delivery errand since the package was
wrapped in a newspaper and weighed almost one kilogram.

95 | P a g e
MABELLE J. ARELLANO PROPERTY
L02 Saturday, 8:00 AM-12:00 PM

SOLEDAD V. PEOPLE
G.R. NO. 184274, FEB 23, 2011

NACHURA, J.:

FACTS

Sometime in June 2004, Henry C. Yu (complainant) received a call on his mobile phone
from a certain "Tess" or "Juliet Villar" (later identified as Rochelle Bagaporo), a credit card
agent, who offered a Citifinancing loan assistance at a low interest rate. Yu invited
Bagaporo to go to his office in QC. Bagaporo then indorsed Yu to her boss, a certain
"Arthur" (later identified as Mark Soledad). Soledad instructed Yu to submit documents to
a certain "Carlo" (later identified as Ronald Gobenchiong). Yu submitted various
documents, such as his Globe handyphone original platinum gold card, identification cards
and statements of accounts. Yu followed up his loan status but failed to get in touch with
either Soledad or Gobenchiong. In the first week of August 2004, Yu received his Globe
handyphone statement of account. He was charged for two mobile phone numbers which
were not his. Upon verification, Yu learned that he had additional five mobile numbers in
his name, and the application for those phone lines bore his picture and his forged
signature. Yu also learned that his Citibank Credit Card database information was altered
and he had a credit card application with Metrobank Card Corporation. Yu and Metrobank’s
junior assistant manager Jefferson Devilleres lodged a complaint with the NBI which
conducted an entrapment operation.

Entrapment operation:
NBI’s Special Investigator Salvador Arteche, together with some other NBI operatives,
arrived in Las Piñas around 5PM. Arteche posed as the delivery boy of the Metrobank
credit card. Upon reaching the address written on the delivery receipt, he asked for Henry
Yu. Soledad responded that he was Henry Yu and presented two identification cards which
bore the name and signature of Yu, while the picture showed Soledad’s face. Soledad
signed the delivery receipt. Arteche introduced himself as an NBI operative and
apprehended Soledad. Arteche recovered from Soledad the two identification cards he
presented earlier.

Soledad was charged with Violation of Section 9(e), R.A. No. 8484 for "possessing a
counterfeit access device or access device fraudulently applied for." RTC found Soledad
guilty as charged. CA affirmed his conviction.

ISSUE

Whether Soledad is guilty of Violation of Section 9(e), R.A. No. 8484 (possessing a
counterfeit access device or access device fraudulently applied for)

RULING

YES. Since RA 8484 does not define the word "possession.", the Court used it as defined
in Article 523 of the Civil Code, "possession is the holding of a thing or the enjoyment of a
right." –

The acquisition of possession involves two elements: the corpus or the


material holding of the thing, and the animus possidendi or the intent to
possess it. Animus possidendi is a state of mind, the presence or
determination of which is largely dependent on attendant events in each
case. It may be inferred from the prior or contemporaneous acts of the
accused, as well as the surrounding circumstances.

Soledad materially held the envelope containing the credit card with the intent to possess.
Contrary to his contention that the credit card never came into his possession because it
was only delivered to him, the facts show that he played an active part in acquiring
possession by presenting the identification cards purportedly showing his identity as Henry
Yu. Certainly, he had the intention to possess it. Moreover, his signature on the

96 | P a g e
MABELLE J. ARELLANO PROPERTY
L02 Saturday, 8:00 AM-12:00 PM

acknowledgment receipt indicates that there was delivery and that possession was
transferred to him as the recipient. Undoubtedly, he knew that the envelope contained the
Metrobank credit card, as clearly indicated in the acknowledgment receipt, coupled with
the fact that he applied for it using the identity of Yu.

97 | P a g e
MABELLE J. ARELLANO PROPERTY
L02 Saturday, 8:00 AM-12:00 PM

CHUA--‐BRUCE V. CA
G.R. NO. 109595, APR.27,2000

QUISUMBING, J.:

FACTS

Ramon Rocamora, manager of Metrobank, requested FructuosoPenaflor, Assistant


Cashier, to conduct a physical bundle count of cash inside the vault, which should total to
P4 million. They found out that there was a shortage of P150,000. After 4 investigations
conducted by the bank and NBI, the reports concluded that Cristeta Chua-Burce, Cash
Custodian, was primary responsible for the shortage. Unable to explain the shortage, the
services of the accused was terminated.

Chua-Burce, together with her husband Antonio Burce, were charged with the crime of
estafa. A civil case was also instituted. The accused prayed for suspension of criminal case
due to a prejudicial question. It was first granted but denied by the CA. The CRIMINAL and
CIVIL cases continued.

The CRIMINAL CASE ruled that she was guilty of estafa. CIVIL CASE also found her liable
for the shortage of P150,000. She appealed both rulings to the CA but the court affirmed
the two TC rulings.

Hence this case.

ISSUE

Whether the elements of estafa were proven beyond reasonable doubt.

RULING

The elements of estafa through conversion or misappropriation under Art. 315 (1) (b) of the
Revised Penal Code are:
(1) that personal property is received in trust, on commission, for administration or
under any other circumstance involving the duty to make delivery of or to return the
same, even though the obligation is guaranteed by a bond;

(2) that there is conversion or diversion of such property by the person who has so
received it or a denial on his part that he received it;

(3) that such conversion, diversion or denial is to the injury of another and
(4) that there be demand for the return of the property.

We find the first element absent. When the money, goods, or any other personal property is
received by the offender from the offended party (1) in trust or (2) on commission or (3)
for administration, the offender acquires both material or physical possession and juridical
possession of the thing received.

Juridical possession means a possession which gives the transferee a right over
the thing which the transferee may set up even against the owner. In this case,
petitioner was a cash custodian who was primarily responsible for the cash-in-vault.
Her possession of the cash belonging to the bank is akin to that of a bank teller, both
being mere bank employees.

98 | P a g e
MABELLE J. ARELLANO PROPERTY
L02 Saturday, 8:00 AM-12:00 PM

EDCA PUBL. V. SANTOS


G.R. NO. 80298, APR. 26,1990
184 SCRA 614

CRUZ, J.:

FACTS

On a relevant date, one person who identified himself as Professor Jose Cruz placed an
order through telephone with Edca Publishing. He ordered 400+ books and issued a
personal check as payment. Then he sold some of the books to Santos who, after verifying
the seller’s ownership from the invoice shown, paid Cruz. Meanwhile, Edca being
suspicious over the second order placed by Cruz verified with De La Salle College where
he had claimed to be dean and was informed that no such person was under its employ.

It was also found out that there was no account with the bank against which he had drawn
his check. It was later found out that his real name was Tomas de la Pena. Edca reported
this to the police and through an entrapment, de la Pena was captured. On the same date,
Edca sought the assistance of the police in recovery of the books bought from it. They
forced their way inside Santos’ store and seized the books without any warrant.

ISSUE

Whether or not the owner was unlawfully deprived of the property?

RULING

First, the contention of petitioner that Santos has not established ownership over the
disputed books because they have not even shown the receipt evidencing the purchase,
is without merit. The possession of movable property acquired in good faith is equivalent
to title.

Second, Santos acquired the books in good faith as found by the lower courts. She first
ascertained the ownership and relied on the invoice shown to her by de la Pena. Santos
was in the business of buying and selling books and often deal with hard-up sellers who
urgently have to part with their books at reduced prices.

Third, and on the real issue, on whether Edca had been unlawfully deprived of the books,
Edca argued that the impostor acquired no title to the books because of the lack of funds
in the check issued and want of consideration. This is without merit. Nonpayment of
purchase price only gives rise to the right to demand payment or rescission of the contract.
Actual delivery was made to the impostor and thus, ownership was acquired by him. Non-
payment was a matter privy to him and EDCA and doesn't involve Santos who later
acquired the books.

Actual delivery of the books having been made, Cruz acquired ownership over the books
which he could then validly transfer to the private respondents. The fact that he had not yet
paid for them to EDCA was a matter between him and EDCA and did not impair the title
acquired by the private respondents to the books.

99 | P a g e
MABELLE J. ARELLANO PROPERTY
L02 Saturday, 8:00 AM-12:00 PM

BPI FAMILY V. FRANCO


G.R. NO. 123498, NOV. 23, 2007

NACHURA, J.:

FACTS

On August 15, 1989, Tevesteco Arrastre-Stevedoring Co., Inc. (Tevesteco) opened a


savings and current account with BPI-FB. Soon thereafter, or on August 25, 1989, First
Metro Investment Corporation (FMIC) also opened a time deposit account with the same
branch of BPI-FB with a deposit of ₱100,000,000.00, to mature one year thence.

Subsequently, on August 31, 1989, Franco opened three accounts, namely, a current,
savings and time deposit, with BPI-FB. The current and savings accounts were respectively
funded with an initial deposit of ₱500,000.00 each, while the time deposit account had
₱1,000,000.00 with a maturity date of August 31, 1990. The total amount of ₱2,000,000.00
used to open these accounts is traceable to a check issued by Tevesteco allegedly in
consideration of Franco’s introduction of Eladio Teves,7 who was looking for a conduit bank
to facilitate Tevesteco’s business transactions, to Jaime Sebastian, who was then BPI-FB
SFDM’s Branch Manager. In turn, the funding for the ₱2,000,000.00 check was part of the
₱80,000,000.00 debited by BPI-FB from FMIC’s time deposit account and credited to
Tevesteco’s current account pursuant to an Authority to Debit purportedly signed by FMIC’s
officers.

In light of BPI-FB’s refusal to heed Franco’s demands to unfreeze his accounts and release
his deposits therein, the latter filed on June 4, 1990 with the Manila RTC the subject suit.
In his complaint, Franco prayed for the following reliefs: (1) the interest on the remaining
balance of his current account which was eventually released to him on October 31, 1991;
(2) the balance on his savings account, plus interest thereon; (3) the advance interes paid
to him which had been deducted when he pre-terminated his time deposit account; and (4)
the payment of actual, moral and exemplary damages, as well as attorney’s fees.

BPI-FB traversed this complaint, insisting that it was correct in freezing the accounts of
Franco and refusing to release his deposits, claiming that it had a better right to the
amounts which consisted of part of the money allegedly fraudulently withdrawn from it by
Tevesteco and ending up in Franco’s accounts. BPI-FB asseverated that the claimed
consideration of ₱2,000,000.00 for the introduction facilitated by Franco between George
Daantos and Eladio Teves, on the one hand, and Jaime Sebastian, on the other, spoke
volumes of Franco’s participation in the fraudulent transaction.

ISSUE

Whether BPI-FB was in bad faith and as such liable for the advance interest it deducted
from Franco’s time deposit account, and for moral as well as exemplary damages?

RULING

Bad faith does not simply connote bad judgment or negligence; it imports a dishonest
purpose or some moral obliquity and conscious doing of wrong; it partakes of the nature of
fraud. We have held that it is a breach of a known duty through some motive of interest or
ill will. In the instant case, we cannot attribute to BPI-FB fraud or even a motive of self-
enrichment. As the trial court found, there was no denial whatsoever by BPI-FB of the
existence of the accounts. The computer-generated document which indicated that the
current account was "not on file" resulted from the prior debit by BPI-FB of the deposits.
The remedy of freezing the account, or the garnishment, or even the outright refusal to
honor any transaction thereon was resorted to solely for the purpose of holding on to the
funds as a security for its intended court action, and with no other goal but to ensure the
integrity of the accounts.

100 | P a g e
MABELLE J. ARELLANO PROPERTY
L02 Saturday, 8:00 AM-12:00 PM

FAJARDO V. FREEDOM TO BUILD


G.R. NO. 134692, AUG. 1, 2000

VITUG, J.:

FACTS

Freedom to Build Inc., an owner-developer and seller of low-cost housing sold to petitioner-
spouses a house and lot in the De La Costa Homes, in Barangka, Marikina, Metro Manila.
The Contract to sell executed between the parties, contained a Restrictive Covenant
providing certain prohibitions, to wit:

“Easements. For the good of the entire community, the homeowner must observe a two-
meter easement in front. No structure of any kind (store, garage, bodega, etc.) may be built
on the front easement.

“Upward expansion. A second storey is not prohibited. But the second storey expansion
must be placed above the back portion of the house and should not extend forward beyond
the apex of the original building.

“Front expansion: 2nd Storey: No unit may be extended in the front beyond the line as
designed and implemented by the developer in the 60 sq. m. unit. In other words, the 2nd
floor expansion, in front, is 6 meters back from the front property line and 4 meters back
from the front wall of the house, just as provided in the 60 sq. m. units.”

The above restrictions were also contained in Transfer Certificate of Title No. N-115384
covering the lot issued in the name of petitioner-spouses.

The controversy arose when the petitioners despite repeated demand from the respondent,
extended the roof of their house to the property line and expanded the second floor of their
house to a point directly above the original front wall. Respondent filed before the RTC an
action to demolish the unauthorized structures.

The RTC rendered a judgment against the petitioner ordering them to immediately
demolish and remove the extension of their expanded housing unit that exceeds the
limitations imposed by the Restrictive Covenant, otherwise the Branch Sheriff of this Court
will execute the this decision at the expense of the defendants.

On appeal, the CA affirmed the decision of the RTC. Hence, this petition for review.

ISSUE

Whether or not the provisions of the Restrictive Covenant are valid.

RULING

Valid. Petitioners must be held to be bound thereby. Since the extension constructed
exceeds the floor area limits of the Restrictive Covenant, petitioner-spouses can be
required to demolish the structure to the extent that it exceeds the prescribed floor area
limits.

Restrictive covenants are not, strictly speaking, synonymous with easements. While it may
be correct to state that restrictive covenants on the use of land or the location or character
of buildings or other structures thereon may broadly be said to create easements or rights,
it can also be contended that such covenants, being limitations on the manner in which
one may use his own property, do not result in true easements, but a case of servitudes
(burden), sometimes characterized to be negative easements or reciprocal negative
easements. Negative easement is the most common easement created by covenant or
agreement whose effect is to preclude the owner of the land from doing an act, which, if no

101 | P a g e
MABELLE J. ARELLANO PROPERTY
L02 Saturday, 8:00 AM-12:00 PM

easement existed, he would be entitled to do.

Courts which generally view restrictive covenants with disfavor for being a restriction on
the use of one's property, have, nevertheless, sustained them where the covenants are
reasonable, not contrary to public policy, or to law, and not in restraint of trade.

The provisions in a restrictive covenant prescribing the type of the building to be erected
are crafted not solely for the purpose of creating easements, generally of light and view,
nor as a restriction as to the type of construction, but may also be aimed as a check on the
subsequent uses of the building conformably with what the developer originally might have
intended the stipulations to be.

Viewed accordingly, the statement of petitioners that their immediate neighbors have not
opposed the construction is unavailing to their cause, the subject restrictive covenant not
being intended for the benefit of adjacent owners but to prescribe the uses of the building,
i.e., to ensure, among other things, that the structures built on De la Costa Homes
Subdivision would prevent overcrowding and promote privacy among subdivision dwellers.

102 | P a g e
MABELLE J. ARELLANO PROPERTY
L02 Saturday, 8:00 AM-12:00 PM

ABELLANA V. CA
G.R. NO. 97039 APR.24, 1992

GRIÑO-AQUINO, J.:

FACTS

Petitioners are owners of a parcel of land on the northwest side of Nonoc Subdivision,
Cebu. They sued to establish an easement of a right of way over a subdivision road, which
they claim they have acquired through prescription since their ancestors have been using
these since time immemorial. They pray that the concrete wall surrounding the village be
taken down to allow easy access to the public highway.

RTC held in favor of the petitioners ordering the respondents to demolish the subject fences
or enclosures at the dead ends of the subject lot at their expense and to leave them open
for the use of the plaintiffs and the general public.

However, the Court of Appeals reversed the lower court’s decision averring that road lots
in subdivisions are private property and may only be used as a public highway once
acquired by the government through donation, purchase or expropriation. The petitioner
filed for a motion for reconsideration which was also denied. Hence, the petition of the
instant case.

ISSUE

Whether or not the easement of a right of way may be acquired by prescription?

RULING

No. Article 620 of the Civil Code provides that only continuous and apparent easements
may be acquired by prescription. The easement of a right of way cannot be considered
continuous because its use is at intervals and is dependent on the acts of man.

The use of a footpath or road may be apparent but it is not a continuous easement because
its use is at intervals and depends upon the acts of man. It can be exercised only if a man
passes or puts his feet over somebody else's land.

103 | P a g e
MABELLE J. ARELLANO PROPERTY
L02 Saturday, 8:00 AM-12:00 PM

BICOL AGRO--‐IND V. OBIAS


G.R. NO. 172077, OCT. 09, 2009

PERALTA, J.:

FACTS

Sometime in 1972, the Bicol Sugar Development Corporation (BISUDECO) was


established at Himaao, Pili, Camarines Sur. In the same year, BISUDECO constructed a
road measuring approximately 7 meters wide and 2.9 kilometers long which was used in
hauling and transporting sugarcane to and from its mill site (Pensumil) and has thus
become indispensable to its sugar milling operations.

On April 19, 1993, petitioner filed a Complaint against respondents Edmundo Obias,
Perfecto Obias, Victor Bagasina, Elena Benosa, Melchor Brandes, Rogelio Montero, Pedro
Montero, Claudio Resari, Pilar Galon, Antonio Buison, Prudencio Benosa, Jr., Victor
Bagasina Jr., Maria Villamer, and Roberto Padua, alleging that on March 27, 1993 and
April 3, 1993, respondents unjustifiably barricaded the disputed road by placing bamboos,
woods, placards and stones across it, preventing petitioners and the other sugar planters
vehicles from passing through the disputed road, thereby causing serious damage and
prejudice to petitioner.

Petitioner alleged that they constructed the disputed road pursuant to an agreement with
the owners of the ricefields the road traversed. The respondents denied said agreement
and alleged that BISUDECO, surreptitiously and without their knowledge and consent,
constructed the disputed road on their properties and has since then intermittently and
discontinuously used the disputed road for hauling sugarcane despite their repeated
protests .

The RTC held that petitioner was entitled to a compulsory easement of right of way as
provided for under Article 649 of the New Civil Code upon payment of proper indemnity to
respondents. A Writ of Preliminary Injunction was issued ordering the respondents to
desist from constructing barricades across the road.

The CA affirmed the finding of the RTC that there was no conclusive proof to sufficiently
establish the existence of an agreement between BISUDECO and respondents regarding
the construction of the disputed road. Moreover, the CA also declared that an easement of
right of way is discontinuous and as such cannot be acquired by prescription. The CA
likewise affirmed the finding of the RTC that petitioner was entitled to a compulsory
easement of right of way upon payment of proper indemnity to respondents. The CA,
however, declared that ownership over the disputed road should remain with respondents,
despite the grant of a compulsory easement. Lastly, the CA deleted the awards to
Prudencio Benosa (Benosa) and Clara Padua (Padua), since the former never claimed
ownership of any portion of the lands affected by the disputed road and the latter was not
a party to the proceedings below.

Petitioner then filed a Motion for Reconsideration alleging among others that the CA
Decision failed to rule on the issue of estoppel and laches. Moreover, Benosa and Padua
filed a Motion for Reconsideration assailing the portion of the CA Decision deleting the
award of indemnity to them. On March 28, 2006, the CA issued a Resolution denying the
same.

ISSUE

Whether or not easement of right of way may be acquired by prescription.

RULING

In this case, the presence of railroad tracks for the passage of petitioners trains denotes

104 | P a g e
MABELLE J. ARELLANO PROPERTY
L02 Saturday, 8:00 AM-12:00 PM

the existence of an apparent but discontinuous easement of right of way. And under Article
622 of the Civil Code, discontinuous easements, whether apparent or not, may be acquired
only by title.

Unfortunately, petitioner never acquired any title over the use of the railroad right of way
whether by law, donation, testamentary succession or contract. Its use of the right of way,
however long, never resulted in its acquisition of the easement because, under Article 622,
the discontinuous easement of a railroad right of way can only be acquired by title and not
by prescription. Hence, even if the road in dispute has been improved and maintained over
a number of years, it will not change its discontinuous nature but simply make the same
apparent. To stress, Article 622 of the New Civil Code states that discontinuous easements,
whether apparent or not, may be acquired only by virtue of a title.

105 | P a g e
MABELLE J. ARELLANO PROPERTY
L02 Saturday, 8:00 AM-12:00 PM

QUINTANILLA V. ABANGAN
G.R. NO. 160613, FEB.12, 2008

NACHURA, J.:

FACTS

Sometime in the 1960s, Perfecta bought Lot No. 3771-B-1-A, with an area of 2,244 square
meters, located at Inayawan, Cebu City (the dominant estate) from one Dionisio Abasolo,
who formerly owned all the properties therein. Thereafter, Perfecta donated the dominant
estate to Apolinardito, who is now the registered owner thereof. [4] Petitioners own QC
Rattan Inc., a domestic corporation engaged in the manufacture and export of rattan-made
furniture. In the conduct of their business, they use vans to haul and transport raw materials
and finished products. As they wanted to expand their business and construct a warehouse
on their property (the dominant estate), they asked for a right of way from Pedro sometime
in April 1994.

However, it appears that Pedro, who was the owner of Lot No. 3771-A-1, containing an
area of 1,164 square meters[5] (the servient estate) and a lot near the dominant estate, sold
the same to DARYL'S on March 24, 1994,[6] and thereafter, DARYL'S constructed a
warehouse over the servient estate, enclosing the same with a concrete fence.

Petitioners, thus, sought the imposition of an easement of right of way, six (6) meters in
width, or a total area of 244 square meters, over the servient estate.

ISSUE

Whether petitioners can validly claim a compulsory right of way against respondents?

RULING

Apolinardito as owner of the dominant estate together with Perfecta failed to discharge the
burden of proving the existence and concurrence of all the requisites in order to validly
claim a compulsory right of way against respondents.

It should be remembered that to be entitled to a legal easement of right of way, the following
requisites must be satisfied: (1) the dominant estate is surrounded by other immovables
and has no adequate outlet to a public highway; (2) proper indemnity has been paid; (3)
the isolation was not due to acts of the proprietor of the dominant estate; and (4) the right
of way claimed is at the point least prejudicial to the servient estate. The fourth requisite is
absent.

As provided for under the provisions of Article 650 of the New Civil Code, the easement of
right of way shall be established at the point least prejudicial to the servient estate, and,
insofar as consistent with this rule, where the distance from the dominant estate to a public
highway may be the shortest. Where there are several tenements surrounding the
dominant estate, and the easement may be established on any of them, the one where the
way is shortest and will cause the least damage should be chosen. But if these two
circumstances do not concur in a single tenement, as in the instant case, the way which
will cause the least damage should be used, even if it will not be the shortest. The criterion
of least prejudice to the servient estate must prevail over the criterion of shortest
distance. The court is not bound to establish what is the shortest; a longer way may be
established to avoid injury to the servient tenement, such as when there are constructions
or walls which can be avoided by a round-about way, as in the case at bar.

As between a right of way that would demolish a fence of strong materials to provide
ingress and egress to a public highway and another right of way which although longer will
only require a van or vehicle to make a turn, the second alternative should be
preferred. Mere convenience for the dominant estate is not what is required by law as the
basis for setting up a compulsory easement. Even in the face of necessity, if it can be
satisfied without imposing the easement, the same should not be imposed.

106 | P a g e
MABELLE J. ARELLANO PROPERTY
L02 Saturday, 8:00 AM-12:00 PM

QUIMEN V. CA
G.R. NO. 112331, MAY 29, 1996

FACTS

Anastacia Quimen, together with her 3 brothers and sister, inherited a piece of property in
Bulacan. They agreed to subdivide the property equally among themselves. The shares of
Anastacia and 3 other siblings were next to the municipal road. Anastacia’s was at the
extreme left of the road while the lots on the right were sold by her brothers to Catalina
Santos. A portion of the lots behind Anastacia’s were sold by her (as her brother’s
administratix) brother to Yolanda.

Yolanda was hesitant to buy the back property at first because it had no access to the
public road. Anastacia prevailed upon her by assuring her that she would give her a right
of way on her adjoining property (which was in front) for P200 per square meter. Yolanda
constructed a house on the lot she bought using as her passageway to the public highway
a portion of Anastacia’s property. But when Yolanda finally offered to pay for the use of the
pathway, Anastacia refused to accept the payment. In fact she was thereafter barred by
Anastacia from passing through her property.

After a few years, Yolanda purchased another lot from the Quimens (a brother). In 1987,
Yolanda filed an action with the proper court praying for a right of way through Anastacia’s
property, but the way was unobstructed except for an avocado tree standing in the middle.

The trial court dismissed the complaint. It held that it was more practical to extend the
existing pathway to the public road by removing that portion of the store (owned by one of
the Quimen brothers) blocking the path as that was the shortest route to the public road
and the least prejudicial to the parties concerned than passing through Anastacia’s
property.

CA reversed and held that Yolanda was entitled to a right of way on Anastacia’s property.
Anastacia went to the SC and she also strongly maintains that the proposed right of way
is not the shortest access to the public road because of the detour and that, moreover, she
is likely to suffer the most damage as she derives a net income of P600.00 per year from
the sale of the fruits of her avocado tree, and considering that an avocado has an average
life span of seventy (70) years, she expects a substantial earning from it.

ISSUE

Whether or not the right of way proposed by Yolanda is the least onerous/least prejudicial
to the parties.

RULING

YES. Article 650 of the NCC (Civil Code) explicitly states that “the easement of right of way
shall be established at the point least prejudicial to the servient estate and, insofar as
consistent with this rule, where the distance from the dominant estate to a public highway
may be the shortest.”

The criterion of least prejudice to the servient estate must prevail over the criterion of
shortest distance although this is a matter of judicial appreciation. When the easement may
be established on any of several tenements surrounding the dominant estate, the one
where the way is shortest and will cause the least damage should be chosen. However, as
elsewhere stated, if these two (2) circumstances do not concur in a single tenement, the
way which will cause the least damage should be used, even if it will not be the shortest.

CA’s finding: The proposed right of way of Yolanda, which is 1m wide and 5m long at the
extreme right of Anastacia’s property will cause the least prejudice and/or damage as
compared to the suggested passage through the property of Yolanda’ s father which would
mean destroying the sari-sari store made of strong materials.

107 | P a g e
MABELLE J. ARELLANO PROPERTY
L02 Saturday, 8:00 AM-12:00 PM

The SC accepts and adopts them. As between a right of way that would demolish a store
of strong materials to provide egress to a public highway, and another right of way which
although longer will only require an avocado tree to be cut down, the second alternative
should be preferred.

108 | P a g e
MABELLE J. ARELLANO PROPERTY
L02 Saturday, 8:00 AM-12:00 PM

UNISOURCE V. CHUNG
G.R. NO. 173252, JUL. 17, 2009

FACTS

Unisource Commercial and Development Corporation is the registered owner of a parcel


land of which contains a memorandum of encumbrance of a voluntary easement carried
over from the OCT of Encarnacion S. Sandico declaring that Francisco Hidalgo has the
right to open doors in the course of his lot and to pass through the land of Encarnacion
Sandico, until the bank of the estero that goes to the Pasig River, and towards the right of
a Callejon. The annotation does not expressly provide that it will be binding to the heirs
and assigns of the parties. Furthermore, the property of Hidalgo was already subdivided.

The memorandum of encumbrance was consistently annotated at the back of every title
covering Sandico’s property until it was acquired by the petitioner. Hidalgo’s property, on
the other hand was eventually transferred to respondents Joseph, Kia and Cleto all
surnamed Chung.

Petitioner filed a Petition to Cancel the Encumbrance of Voluntary Easement of Right of


Way on the ground that the dominant estate has an adequate access to a public road which
is Matienza Street which was granted by the trial court but eventually reversed by the Court
of Appeals.

ISSUES

 Whether or not the Encumbrance of Voluntary Easement of Right of Way can be


cancelled by the petitioners who owns the servient estate on the ground that that
the dominant estate has an adequate access to a public road.

 Whether or not the easement is binding only between Hidalgo and Sandico since
the annotation did not expressly provides the intention to bind their heirs and
assigns.

RULING

An easement is a real right on another’s property, corporeal and immovable, whereby the
owner of the latter must refrain from doing or allowing somebody else to do or something
to be done on his property, for the benefit of another person or tenement. These are
established either by law (legal easement) or by the will of the owner (voluntary easement).

(1) The opening of an adequate outlet to a highway can extinguish only legal or compulsory
easements, not voluntary easements like in the case at bar. Petitioner itself admitted that
the existing easement is voluntary and cannot now claim that what exists is a legal
easement and that the same should be cancelled since the dominant estate is not an
enclosed estate as it has an adequate access to a public road which is Callejon Matienza
Street. The fact that an easement by grant may have also qualified as an easement of
necessity does not detract from its permanency as a property right, which survives the
termination of the necessity. This easement of right of way, like any other contract, could
be extinguished only by mutual agreement or by renunciation of the owner of the dominant
estate. (Art. 631,NCC)

(2) Neither can petitioner claim that the easement is personal only to Hidalgo since the
annotation merely mentioned Sandico and Hidalgo without equally binding their heirs or
assigns. That the heirs or assigns of the parties were not mentioned in the annotation does
not mean that it is not binding on them. Again, a voluntary easement of right of way is like
any other contract. As such, it is generally effective between the parties, their heirs and
assigns, except in case where the rights and obligations arising from the contract are not
transmissible by their nature, or by stipulation or by provision of law.

109 | P a g e
MABELLE J. ARELLANO PROPERTY
L02 Saturday, 8:00 AM-12:00 PM

TELMO V. BUSTAMANTE
G.R. NO. 182567, JUL. 13, 2009

NACHURA, J.:

FACTS

The complaint alleged that respondent is a co-owner of a real property of 616 square
meters in Brgy. Halang, Naic, Cavite, known as Lot 952-A and covered by Transfer
Certificate of Title No. T-957643 of the Register of Deeds of Cavite. Petitioner and Elizalde
Telmo (Telmos) are the owners of the two (2) parcels of land denominated as Lot 952-B
and 952-C, respectively, located at the back of respondents lot. When his lot was
transgressed by the construction of the Noveleta-Naic-Tagaytay Road, respondent offered
for sale the remaining lot to the Telmos. The latter refused because they said they would
have no use for it, the remaining portion being covered by the roads 10-meter easement.

The complaint further alleged that, on May 8, 2005, respondent caused the resurvey
of Lot 952-A in the presence of the Telmos. The resurvey showed that the Telmos
encroached upon respondents lot. Petitioner then uttered, Hanggat ako ang municipal
engineer ng Naic, Cavite, hindi kayo makakapagtayo ng anuman sa lupa nyo; hindi ko kayo
bibigyan ng building permit.

On May 10, 2005, respondent put up concrete poles on his lot. However, around 7:00 p.m.
of the same day, the Telmos and their men allegedly destroyed the concrete poles. The
following day, respondents relatives went to Brgy. Chairman Consumo to report the
destruction of the concrete poles. Consumo told them that he would not record the same,
because he was present when the incident occurred. Consumo never recorded the incident
in the barangay blotter.

Respondent complained that he and his co-owners did not receive any just compensation
from the government when it took a portion of their property for the construction of the
Noveleta-Naic-Tagaytay Road. Worse, they could not enjoy the use of the remaining part
of their lot due to the abusive, Illegal, and unjust acts of the Telmos and Consumo.

ISSUE

Whether the concrete posts were in the nature of a nuisance per se, which may be the
subject of summary abatement?

RULING

We disagree.

A nuisance per se is that which affects the immediate safety of persons and property and
may be summarily abated under the undefined law of necessity. Evidently, the concrete
posts summarily removed by petitioner did not at all pose a hazard to the safety of persons
and properties, which would have necessitated immediate and summary abatement. What
they did, at most, was to pose an inconvenience to the public by blocking the free passage
of people to and from the national road.

110 | P a g e
MABELLE J. ARELLANO PROPERTY
L02 Saturday, 8:00 AM-12:00 PM

GANCAYCO V. QUEZON CITY


G.R. NO. 177807, OCT 11, 2011

SERENO, J.:

FACTS

The consolidated petitions of Retired Justice Emilio Gancayco, City Government of Quezon
City and the Metro Manila Development Authority stemmed from a local ordinance
pertaining to Construction of Arcades, and the clearing of Public Obstructions. Gaycanco
owns a property, of which he was able to obtain a building permit for a two-storey
commercial building, which was situated along EDSA, in an area which was designated as
part of a Business/Commercial Zone by the Quezon City Council. The Quezon City Council
also issued Ordinance No. 2904, which orders the construction of Arcades for Commercial
Buildings. The ordinance was amended to not require the properties located at the Quezon
City - San Juan boundary, and commercial buildings from Balete - Seattle Street to
construct the arcades, moreover, Gancayco had been successful in his petition to have his
property, already covered by the amended ordinance, exempted from the ordinance.
MMDA on April 28, 2003, sent a notice to Gancayco, under Ordinance no. 2904, part of his
property had to be demolished, if he did not clear that part within 15 days, which Gancayco
did not comply with, and so the MMDA had to demolish the party wall, or “wing walls.”

Gancayco then filed a temporary restraining order and/or writ of preliminary injunction
before the RTC of Quezon City, seeking to prohibit the demolition of his property, without
due process and just compensation, claiming that Ordinance no. 2904 was discriminatory
and selective. He sought the declaration of nullity of the ordinance and payment for
damages. MMDA contended that Gancayco cannot seek nullification of an ordinance that
he already violated, and that the ordinance had the presumption of constitutionality, and it
was approved by the Quezon City Council, taking to note that the Mayor signed the
ordinance. The RTC, however, declared that the Ordinance was unconstitutional, invalid
and void ab initio. MMDA appealed to the Court of Appeals, and the CA partly granted the
appeal, with the contention that the ordinance was to be modified; it was constitutional
because the intention of the ordinance was to uplift the standard of living, and business in
the commercial area, as well as to protect the welfare of the general public passing by the
area, however the injunction against the enforcement and implementation of the ordinance
is lifted. With that decision, the MMDA and Gancayco filed Motions for Reconsideration,
which the CA denied, as both parties have no new issues raised. Therefore they petitioned
to the Court.

ISSUES

Whether or not the wing wall of Gancayco’s property can be constituted as a public
nuisance. Whether or not MMDA was in their authority to demolish Gancayco’s property.

RULING

The court affirmed the decision of the Court of Appeals. The court decided that the wing
wall of Gancayco’s building was not a nuisance per se, as under Art. 694 of the Civil Code
of the Philippines, nuisance is defined as any act, omission, establishment, business,
condition or property, or anything else that (1) injures of endangers the health or safety of
the others; (2) annoys or offends the senses; (3) shocks, defies or disregards decency or
morality; (4) obstruct or interferes with the free passage of any public highway or street, or
any body of water; or (5) hinders or impairs the use of property. A nuisance may be a
nuisance per se or a nuisance per accidens. A nuisance per se are those which affect the
immediate safety of persons and property and may summarily be abated under the
undefined law of necessity. As Gaycanco was able to procure a building permit to construct
the building, it was implied that the city engineer did not consider the building as such of a
public nuisance, or a threat to the safety of persons and property. The MMDA was only to
enforce Authoritative power on development of Metro Manila, and was not supposed to act
with Police Power as they were not given the authority to do such by the constitution, nor
was it expressed by the DPWH when the ordinance was enacted. Therefore, MMDA acted
on its own when it illegally demolished Gancayco’s property, and was solely liable for the
111 | P a g e
MABELLE J. ARELLANO PROPERTY
L02 Saturday, 8:00 AM-12:00 PM

damage.
The wing walls do not per se immediately and adversely affect the safety
of persons and property. The fact that an ordinance may declare a
structure illegal does not necessarily make that structure a nuisance.

112 | P a g e
MABELLE J. ARELLANO PROPERTY
L02 Saturday, 8:00 AM-12:00 PM

PEREZ V. MADRONA
G.R. NO. 184478, MAR. 21,2012

VILLARAMA, JR., J.:

FACTS

Respondent-spouses Fortunito Madrona and Yolanda B. Pante are registered owners of a


residential property located in Lot 22, Block 5, France Street corner Italy Street,
Greenheights Subdivision, Phase II, Marikina City and covered by Transfer Certificate of
Title No. 169365 of the Registry of Deeds of Marikina. In 1989, respondents built their
house thereon and enclosed it with a concrete fence and steel gate.

On 1999, respondents received a letter dated May 25, 1999 from petitioner Jaime S. Perez,
Chief of the Marikina Demolition Office. More than a year later or on February 28, 2001,
petitioner sent another letter with the same contents as the May 25, 1999 letter but this time
giving respondents ten days from receipt thereof to remove the structure allegedly
protruding to the sidewalk. This prompted respondents to file a complaint9 for injunction
before the Marikina City RTC on March 12, 2001.

ISSUE

Whether the fence may be summarily demolished?

RULING

If petitioner indeed found respondents’ fence to have encroached on the sidewalk, his
remedy is not to demolish the same summarily after respondents failed to heed his request
to remove it. Instead, he should go to court and prove respondents’ supposed violations in
the construction of the concrete fence. Indeed, unless a thing is a nuisance per se, it may
not be abated summarily without judicial intervention. Our ruling in Lucena Grand Central
Terminal, Inc. v. JAC Liner, Inc., on the need for judicial intervention when the nuisance is
not a nuisance per se, is well worth mentioning. In said case, we ruled:

Respondents cannot seek cover under the general welfare clause authorizing the
abatement of nuisances without judicial proceedings. That tenet applies to a nuisance per
se, or one which affects the immediate safety of persons and property and may be
summarily abated under the undefined law of necessity (Monteverde v. Generoso, 52 Phil.
123 [1982]). The storage of copra in the quonset building is a legitimate business. By its
nature, it cannot be said to be injurious to rights of property, of health or of comfort of the
community. If it be a nuisance per accidens it may be so proven in a hearing conducted for
that purpose. It is not per se a nuisance warranting its summary abatement without judicial
intervention

Respondents’ fence is not a nuisance per se. By its nature, it is not injurious to the health
or comfort of the community. It was built primarily to secure the property of respondents
and prevent intruders from entering it. And as correctly pointed out by respondents, the
sidewalk still exists. If petitioner believes that respondents’ fence indeed encroaches on
the sidewalk, it may be so proven in a hearing conducted for that purpose. Not being a
nuisance per se, but at most a nuisance per accidens its summary abatement without
judicial intervention is unwarranted.

113 | P a g e
MABELLE J. ARELLANO PROPERTY
L02 Saturday, 8:00 AM-12:00 PM

REPUBLIC V. GUZMAN
G.R. NO. 132964, FEBRUARY 18, 2000

BELLOSILLO, J.:

FACTS

David Rey Guzman, a natural-born American citizen, is the son of the spouses Simeon
Guzman (naturalized American) and Helen Meyers Guzman (American citizen). In 1968,
Simeon died leaving to his heirs, Helen and David, an estate consisting of several parcels
of land in Bulacan.

In 1970, Helen and David executed a Deed of Extrajudicial Settlement of the Estate,
dividing and adjudicating to themselves all of the property, and registered it to the RD a
year after.

In 1981, Helen executed a Deed of Quitclaim, assigning, transferring and conveying her ½
share of the properties to David. But since it was not registered, she executed another
Deed of Quitclaim to confirm the first.

In 1994, Atty. Batongbacal wrote the OSG andfurnished it with documents showing that
David’s ownership of ½ of the estate was defective. He argued that Art. XII of the
Constitution only allows Filipinos to acquire private lands in the country. The only instances
when a foreigner may acquire private property are by hereditary succession and if he was
formerly a natural-born citizen who lost his Filipino citizenship. Moreover, it contends that
the Deeds of Quitclaim executed by Helen were really donations inter vivos.

Republic filed with RTC a Petition for Escheat praying that ½ of David’s interest be forfeited
in its favor. RTC dismissed. CA affirmed.

ISSUE

Whether or not there was a valid donation inter vivos.

RULING

NO.

There are three (3) essential elements of a donation: (a) the reduction of the patrimony of
the donor; (b) the increase in the patrimony of the donee; and, (c) the intent to do an act of
liberality or animus donandi. When applied to a donation of an immovable property, the law
further requires that the donation be made in a public document and that there should be
an acceptance thereof made in the same deed of donation or in a separate public
document. In cases where the acceptance is made in a separate instrument, it is mandated
that the donor should be notified thereof in an authentic form, to be noted in both
instruments.[

Not all the elements of a donation are present. The transfer of the properties by virtue of a
Deed of Quitclaim resulted in the (1) reduction of her patrimony as donor and the (2)
consequent increase in the patrimony of David as donee. However, Helen’s (3) intention
to perform an act of liberality in favor of David was not sufficiently established. The 2
Quitclaims reveal that Helen intended to convey to her son certain parcels of land and to
re-affirm it, she executed a waiver and renunciation of her rights over these properties. It
is clear that Helen merely contemplated a waiver of her rights, title, interest over the lands
in favor of David, not a donation. She was also aware that donation was not possible.

Moreover, the essential element of acceptance in the proper form and registration to make
the donation valid is lacking. The SPA executed by David in favor of Atty. Abela was not
his acceptance, but an acknowledgment that David owns the property referred to and that
he authorizes Atty. Abela to sell the same in his name. Further, there was nothing in the
SPA to show that he indeed accept the donation.

114 | P a g e
MABELLE J. ARELLANO PROPERTY
L02 Saturday, 8:00 AM-12:00 PM

The donation is null and void when (a) the deed of donation fails to show the acceptance,
or (b) where the formal notice of the acceptance made in a separate instrument is either
not given to the donor or else noted in the deed of donation, and in the separate
acceptance.

115 | P a g e
MABELLE J. ARELLANO PROPERTY
L02 Saturday, 8:00 AM-12:00 PM

VILLANUEVA VS. SPOUSES BRANOCO


G.R. NO. 172804, JANUARY 24, 2011

CARPIO, J.:

FACTS

Gonzalo, here represented by his heirs, sued spouses Froilan and Leonila Branoco in the
RTC of Naval, Biliran for the recovery of a parcel of land in Leyte. He claimed ownership
over the property through purchase from Vere who in turn purchased the property from
Rodrigo in 1970. The respondents in this case claimed ownership in their answer through
purchase in 1983 from Rodriguez to whom Rodrigo donated the property in 1965.

The trial court ruled in favor of the petitioner, saying that by the time Rodriguez sold the
property to the respondents in this case she had no title to transfer because the donation
to her by Rodrigo was deemed cancelled when Rodrigo decided to sell the property to Vere
instead.

ISSUE

Whether the contract between the parties predecessors-in-interest, Rodrigo and


Rodriguez, was a donation or a devise.

RULING

Naked Title Passed from Rodrigo to Rodriguez Under a Perfected Donation. It is


immediately apparent that Rodrigo passed naked title to Rodriguez under a perfected
donation inter vivos. First. Rodrigo stipulated that if the herein Donee predeceases me, the
[Property] will not be reverted to the Donor, but will be inherited by the heirs of
x x x Rodriguez, signaling the irrevocability of the passage of title to Rodriguezs estate,
waiving Rodrigos right to reclaim title. This transfer of title was perfected the moment
Rodrigo learned of Rodriguezs acceptance of the disposition which, being reflected in the
Deed, took place on the day of its execution on 3 May 1965. Rodrigos acceptance of the
transfer underscores its essence as a gift in presenti, not in futuro, as only
donations inter vivos need acceptance by the recipient. Indeed, had Rodrigo wished to
retain full title over the Property, she could have easily stipulated, as the testator did in
another case, that the donor, may transfer, sell, or encumber to any person or entity the
properties here donated x x x or used words to that effect. Instead, Rodrigo expressly
waived title over the Property in case Rodriguez predeceases her.

Second. What Rodrigo reserved for herself was only the beneficial title to the
Property, evident from Rodriguezs undertaking to give one [half] x x x of the produce of the
land to Apoy Alve during her lifetime. Thus, the Deeds stipulation that the ownership shall
be vested on [Rodriguez] upon my demise, taking into account the non-reversion clause,
could only refer to Rodrigos beneficial title. We arrived at the same conclusion in Balaqui v.
Dongso where, as here, the donor, while b[inding] herself to answer to the [donor] and her
heirs x x xthat none shall question or disturb [the donees] right, also stipulated that the
donation does not pass title to [the donee] during my lifetime; but when I die, [the donee]
shall be the true owner of the donated parcels of land. In finding the disposition as a
gift inter vivos, the Court reasoned:

Taking the deed x x x as a whole, x x x x it is noted that in the same deed [the
donor] guaranteed to [the donee] and her heirs and successors, the right to said
property thus conferred. From the moment [the donor] guaranteed the right granted
by her to [the donee] to the two parcels of land by virtue of the deed of gift, she
surrendered such right; otherwise there would be no need to guarantee said right.
Therefore, when [the donor] used the words upon which the appellants base their
contention that the gift in question is a donation mortis causa [that the gift does not
pass title during my lifetime; but when I die, she shall be the true owner of the two
aforementioned parcels] the donor meant nothing else than that she reserved
of herself the possession and usufruct of said two parcels of land until her
death, at which time the donee would be able to dispose of them freely.
116 | P a g e
MABELLE J. ARELLANO PROPERTY
L02 Saturday, 8:00 AM-12:00 PM

CENTRAL PHILIPPINES UNIVERSITY VS. CA


G.R. NO. 112127 JULY 17, 1995

BELLOSILLO, J.:

FACTS

In 1939, the late Don Ramon Lopez was a member of the board of trustees of Central
Philippine University when he executed a donation to the school, stating that the land must
be for exclusive use of a medical college. 50 years later, The heirs of Ramon Lopez filed
an action to annul the donation, stating the failure of the school to construct the medical
college over the land. RTC ruled in favor of respondents, which the CA affirmed.

ISSUE

Whether there is a resolutory condition.

RULING

The donation was an onerous one, where failure of the school to construct a medical
college would give the heirs the power to revoke the donation, reverting the property back
to the heirs of the donor. It is therefore a resolutory condition. Although, the period was not
stated, and the courts should have fixed a period, in this case, 50 years has lapsed since
the donation was executed, thus fixing a period would serve no purpose and the property
must already be reverted back.

If there was no fulfillment or compliance with the condition, the donation may now be
revoked and all rights which the donee may have acquired under it shall be deemed lost
and extinguished.

117 | P a g e
MABELLE J. ARELLANO PROPERTY
L02 Saturday, 8:00 AM-12:00 PM

INSULAR LIFE V. EBRADO


G.R. NO. 44059, OCT. 28,1977

MARTIN, J.:

FACTS

Cristor Ebrado was issued by The Life Assurance Co., Ltd., a policy for P5,882.00 with a
rider for Accidental Death. He designated Carponia T. Ebrado as the revocable beneficiary
in his policy. He referred to her as his wife.

Cristor was killed when he was hit by a failing branch of a tree. Insular Life was made liable
to pay the coverage in the total amount of P11,745.73, representing the face value of the
policy in the amount of P5,882.00 plus the additional benefits for accidental death.
Carponia T. Ebrado filed with the insurer a claim for the proceeds as the designated
beneficiary therein, although she admited that she and the insured were merely living as
husband and wife without the benefit of marriage.

Pascuala Vda. de Ebrado also filed her claim as the widow of the deceased insured. She
asserts that she is the one entitled to the insurance proceeds.

Insular commenced an action for Interpleader before the trial court as to who should be
given the proceeds. The court declared Carponia as disqualified.

ISSUE

WON a common-law wife named as beneficiary in the life insurance policy of a legally
married man can claim the proceeds in case of death of the latter?

RULING

In essence, a life insurance policy is no different from a civil donation insofar as the
beneficiary is concerned. Both are founded upon the same consideration: liberality. A
beneficiary is like a donee, because from the premiums of the policy which the insured
pays out of liberality, the beneficiary will receive the proceeds or profits of said insurance.
As a consequence, the proscription in Article 739 of the new Civil Code should equally
operate in life insurance contracts.

Article 739 provides that void donations are those made between persons who were guilty
of adultery or concubinage at the time of donation.

There is every reason to hold that the bar in donations between legitimate spouses and
those between illegitimate ones should be enforced in life insurance policies since the
same are based on similar consideration. So long as marriage remains the threshold
of family laws, reason and morality dictate that the impediments imposed upon married
couple should likewise be imposed upon extra-marital relationship.

118 | P a g e
MABELLE J. ARELLANO PROPERTY
L02 Saturday, 8:00 AM-12:00 PM

ZAMBOANGA V. PLAGATA,
G.R. NO. 148433, SEPT. 30, 2008

CHICO-NAZARIO, J.:

FACTS

On 9 January 1973, President Ferdinand E. Marcos issued Presidential Decree No.


93 which legalized barter trading in the Sulu Archipelago and adjacent areas, and
empowered the Commander of the Southwest Command of the Armed Forces of the
Philippines (AFP) to coordinate all activities and to undertake all measures for the
implementation of said decree.

On 17 June 1981, ZBTKBI, thru its President, Atty. Hassan G. Alam, and the Republic of
the Philippines, represented by Maj. Gen. Delfin C. Castro, Commander, Southern
Command of the AFP, and Chairman, Executive Committee for Barter Trade, entered into
a Deed of Donation whereby ZBTKBI donated to the Republic a parcel of land covered by
Certificate of Title (CTC) No. T-61,628 of the Registry of Deeds of Zamboanga City,
identified as Lot No. 6 of consolidation subdivision plan Pcs-09-000184, situated in the
Barrio of Canelar, City of Zamboanga, containing an area of thirteen thousand six hundred
forty-three (13,643) square meters, more or less. The Republic accepted the donation
which contained the following conditions:

1. That upon the effectivity or acceptance hereof the DONEE shall, thru the authorized
agency/ministry, construct a P5 Million Barter Trade market building at the afore-described
parcel of land;

4. That in the event barter trading shall be phased out, prohibited, or suspended for more
than one (1) year in Zamboanga City, Philippines, the afore-described parcel of land shall
revert back to the DONOR without need of any further formality or documentation, and the
DONOR shall have the first option to purchase the building and improvements thereon.

With the acceptance of the donation, TCT No. T-61,628 in the name of ZBTKBI was
cancelled and, in lieu thereof, TCT. No. T-66,696 covering the same property was issued
in the name of the Republic of the Philippines (Republic)

ISSUE

Whether the stipulation that allows automatic reversion is valid?

RULING

As above ruled, the deed of donation contains a stipulation that allows automatic
reversion. Such stipulation, not being contrary to law, morals, good customs, public order
or public policy, is valid and binding on the parties to the donation. As held in Dolar v.
Barangay Lublub (Now P.D. Monfort North) Municipality of Dumangas, citing Roman
Catholic Archbishop of Manila v. Court of Appeals:

The rationale for the foregoing is that in contracts providing for automatic revocation,
judicial intervention is necessary not for purposes of obtaining a judicial declaration
rescinding a contract already deemed rescinded by virtue of an agreement providing for
rescission even without judicial intervention, but in order to determine whether or not the
rescission was proper.

When a deed of donation, . . . expressly provides for automatic revocation and reversion
of the property donated, the rules on contract and the general rules on prescription should
apply, and not Article 764 of the Civil Code. Since Article 1306 of said Code authorizes
the parties to a contract to establish such stipulations, . . . not contrary to law, . . .
public order or public policy, we are of the opinion that, at the very least, that
stipulation of the parties providing for automatic revocation of the deed of donation,
without prior judicial action for that purpose, is valid subject to the determination of
119 | P a g e
MABELLE J. ARELLANO PROPERTY
L02 Saturday, 8:00 AM-12:00 PM

the propriety of the rescission sought. Where such propriety is sustained, the
decision of the court will be merely declaratory of the revocation, but it is not in itself
the revocatory act.

120 | P a g e
MABELLE J. ARELLANO PROPERTY
L02 Saturday, 8:00 AM-12:00 PM

QUIJADA VS. CA
G.R. NO. 126444, DEC. 4, 1998

MARTINEZ, J.:

FACTS

Petitioners are the children of the late Trinidad Quijada. Trinidad and her siblings executed
a deed of donation of a two-hectare lot in favor of the Municipality of Talacogon (Agusan
del Sur), exclusively for the purpose of constructing the proposed provincial high school.
However, possession remained with Trinidad. She subsequently sold the two hectares on
two separate occasions to Regalado Mondejar, who sold it to different persons. Eventually,
the Municipality, failing to construct the high school, reverted ownership to the donors.
Petitioners filed an action for quieting of title and recovery of possession and ownership.
RTC ruled in favor of petitioners, but CA reversed.

ISSUE

Whether ownership was already transferred to the Municipality of Talacogon.

RULING

The donation made on April 5, 1956 by Trinidad Quijada and her brother and sisters was
subject to the condition that the donated property shall be "used solely and exclusively as
a part of the campus of the proposed Provincial High School in Talacogon." The donation
further provides that should "the proposed Provincial High School be discontinued or if the
same shall be opened but for some reason or another, the same may in the future be
closed" the donated property shall automatically revert to the donor. Such condition, not
being contrary to law, morals, good customs, public order or public policy was validly
imposed in the donation.

When the Municipality's acceptance of the donation was made known to the donor, the
former became the new owner of the donated property -- donation being a mode of
acquiring and transmitting ownership - notwithstanding the condition imposed by the
donee. The donation is perfected once the acceptance by the donee is made known to the
donor. Accordingly, ownership is immediately transferred to the latter and that ownership
will only revert to the donor if the resolutory condition is not fulfilled.

In this case, that resolutory condition is the construction of the school. It has been ruled
that when a person donates land to another on the condition that the latter would build
upon the land a school, the condition imposed is not a condition precedent or a suspensive
condition but a resolutory one. Thus, at the time of the sales made in 1962 towards 1968,
the alleged seller (Trinidad) could not have sold the lots since she had earlier transferred
ownership thereof by virtue of the deed of donation. So long as the resolutory condition
subsists and is capable of fulfillment, the donation remains effective and the donee
continues to be the owner subject only to the rights of the donor or his successors-in-
interest under the deed of donation.

Since no period was imposed by the donor on when must the donee comply with the
condition, the latter remains the owner so long as he has tried to comply with the condition
within a reasonable period. Such period, however, became irrelevant herein when the
donee-Municipality manifested through a resolution that it cannot comply with the condition
of building a school and the same was made known to the donor. Only then - when the
non-fulfillment of the resolutory condition was brought to the donor's knowledge - that
ownership of the donated property reverted to the donor as provided in the automatic
reversion clause of the deed of donation.

121 | P a g e

Você também pode gostar